PDA

View Full Version : Got a Real World Weapons or Armour Question? Mk XI



Pages : 1 2 3 4 [5] 6

warty goblin
2013-03-09, 12:11 AM
Not certain. Never had the chance to put on armour, so I don't know how much weight I could wear before it would weaken my orc-killing talents.

I know that about 60 pounds is the most you can wear before the armour starts to weigh you down, based off seeing people in plate armour. So 25 is probably quite light.

I haven't done much fighting in armor, but I have done some light hiking from time to time in a ~23lbs chain shirt, which coverage to mid-thigh and halfway down my forearms. The last time I did about three miles through woods without human-made trails in high summer, and it wasn't too bad. I was fairly tired when I got done, but I'm not in fantastic shape, and that was due to the heat as much as anything.

One thing to keep in mind with mail is that it isn't rigid, and if you don't belt it on it basically sloshes around you. This is annoying when you're just walking, but it's a downright hazard in a fight. Try to stop short and your armor will keep going, dragging you off balance. A good thick belt at the waist goes a long way towards mitigating this, and also reduces shoulder strain as well.

For a tiger of I have no idea how you'd go about coming up with a belt system that doesn't restrict breathing and movement, but I'd think something like that would be in order.

Haruspex_Pariah
2013-03-09, 01:33 AM
That's a cool picture, Raum.

So it's largely a question of terrain handling and weight support it seems. Thanks all.

Mr. Mask
2013-03-09, 01:41 AM
Yes.... there are many challenges and concerns with come with armouring a ferocious monster (forget trolls and ogres--them tigers are crazier than Tigger!).

My main concern is the weight of the armour, and how effective it would be at warding off blows and missiles. A 200 pounds of mail armour and some padding sounds like it would be excellent armour--even though it has a bigger creature to cover.

Xuc Xac
2013-03-09, 02:28 AM
You can put barding on a horse because it can handle the weight. A cat's back is too flexible and can't support much weight. A relatively straight-backed hoofed animal can support a lot of weight and animals with arched spines can support a lot. Cats would be pulled down by weight on their backs. Cats also have much, much less endurance than humans and would drop from exhaustion if they had to carry a quarter of their body weight for more than a few minutes.

Rhynn
2013-03-09, 03:45 AM
Here is a question I bet you don't often get:

How heavy is light armour, designed not to impair movement--when it is created for an 800 pound tiger to wear?

Not joking with this. I tried looking up horse armour weights for comparison, but had trouble finding a good example.

What is a tiger's surface area? Then look up the weight of mail per square inch or centimeter or whatever, and multiply... :smallamused: (Plus quilted padding, of course.)

Seriously, that's the only way to "deduce" the answer, as nothing really comparable exists, most likely. We can pull numbers like 200 out of hats, and yeah, that sounds plausible I guess, can tigers carry 200 pounds easily? Has anyone done tests with tiger carrying capacity? Go figure...

fusilier
2013-03-09, 03:53 AM
Seriously, that's the only way to "deduce" the answer, as nothing really comparable exists, most likely. We can pull numbers like 200 out of hats, and yeah, that sounds plausible I guess, can tigers carry 200 pounds easily? Has anyone done tests with tiger carrying capacity? Go figure...

Put armor on a house cat then scale up? ;-)

Mr. Mask
2013-03-09, 03:56 AM
Someone actually tried that. Heard he was an Italian lord, who really loved his Persian cats. It was more as a joke, of course... they were just housecats, after all.

There is Japanese Dog Armour we could use as an example, if anyone can find details on it.

Spiryt
2013-03-09, 04:37 AM
In my experience, it depends more on the size of the link and the weave. A 6 in 1 or more tends to be substantially heavier than the standard 4 in 1 for the same size links.

Likewise even a 4 in 1 weave would be incredible heavy with 1/4" 13 gauge wire while an 8 in 1 with 18 gauge wire and 3/4" links would be fairly light (not to mention look rather sparse)

Well, 4 in 1 is pretty much only weave seriously used in what we usually call 'mail' - so ancient Mediterranean, and medieval Middle Eastern and European armors.

At least from what we know now, it's entirely possible that there were more 6 in 1 attempts that are now lost in history.

As far as size to wire thickness ratio goes, it's of course true - so 25 pounds 'could' be overweight for mail shirt, because there were certainly more substantial shirts of such weight and heavier. Usually those probably weighted less though.

Yora
2013-03-09, 06:25 AM
It's important to remember that the mail is only one component of the full armor. Just throwing the mail over your everyday clothing would perform quite different from full gear.

Galloglaich
2013-03-09, 11:12 AM
Wheeled armored vehicles tend to be extremely limited in off-road mobility compared to tracked. This is in spite of the fact that they tend to be very lightly armored. This is why anybody uses tracks on vehicles because I promise you, dealing with tracks is a pain in the ass. The armor on most 'light armored' vehicles, while heavy in terms of weight, is usually very limited in terms of protection. The Stryker for example is only rated to be 'resistant' to 7.62mm (medium) and 14.5mm (heavy) machine guns, probably on the sides and front respectively. Even the tank-like wheeled AFV's with big guns such as the Panhard ERC (http://en.wikipedia.org/wiki/Panhard_ERC) are very lightly armored (10 mm).

http://upload.wikimedia.org/wikipedia/commons/thumb/6/69/RHP_Cote_d%27ivoire_2003.jpg/220px-RHP_Cote_d%27ivoire_2003.jpg

These vehicles can definitely be knocked out by an RPG. That doesn't mean they aren't effective, in the right kind of terrain they work pretty well, but they are limited.

Main battle tanks by contrast, have done very well against RPG's or any other type of shaped-charge warheads since the late 1970'ss when the laminate armored types like the M-1, Challenger, Leopard II, Leclerc and so on have been deployed. These are not just thick steel, but laminated / composite armor with ceramic inside that can defeat the shaped charge warheads (which essentially burn through the old type of steel armor).

By the 1990's these types of tanks were essentially invulnerable to shaped charge warheads. In fact it's almost the opposite of what Yora said, the only reliable way to knock one out was by the main gun of another tank (tanks, incidentally, can also shoot HEAT / shaped charge warheads). Various methods were developed to deal with this, the US TOW III for example has a pop-up capability which allows it to strike tanks from above.

http://upload.wikimedia.org/wikipedia/commons/thumb/2/2f/RPG-29_USGov.JPG/300px-RPG-29_USGov.JPG
In the last 5 years however the Russians developed a new tandem warhead RPG, the RPG-29v (http://en.wikipedia.org/wiki/RPG-29) which explodes twice: once to crack the ceramic and then a second time to cause damage. Some of these got into the hands of Hezbollah a few years ago and proved capable of knocking out several Israeli Merkava tanks during the last Israeli invasion of Lebanon. They also appear to have been able to disable at least one British Challenger and one M-1 Abrams in Iraq (http://en.wikipedia.org/wiki/RPG-29#Iraq_2003_onwards) but so far the effectiveness seems to have been limited since there have only been a couple of incidents.

In fact only a handful of M-1's were lost for any reason during the Iraq wars. They are just about the only US armored vehicle in which the crew is really pretty safe against all threats. The Bradley, by contrast, though it did not suffer the casualties some anticipated in the pitched battles of the Iraq wars such as 73 Easting, did prove to be overly vulnerable to IED's and for that reason was taken out of the line in favor of MRAPS. The MRAPs of course while mine resistant are cumbersome and extremely limited in mobility which basically forces them to stay on the roads (which ironically makes them more vulnerable to IED ambush)

Right now there is a big gap between the still mostly invulnerable, but very heavy (70 ton plus) not very portable Main Battle Tanks like Challenger and M-1, and smaller / lighter AFV's which can protect their crews against more than just Ak-47s and light machine guns. The US has been just using Hummers which are designed to be used only in safe rear areas, as well as MRADS and various other cobbled together anti-mine vehicles.

The Germans have probably come closest with their Marder (http://en.wikipedia.org/wiki/Marder_%28IFV%29) and later PUMA (http://en.wikipedia.org/wiki/Puma_%28IFV%29) class of APC / IFV. These are tracked so they can actually get around on rough terrain, but are about half the weight of a Main Battle Tank, and can carry some infantry. They also have armor that can stop RPG's and so on.

As for cat armor... you are in luck!

https://sphotos-a.xx.fbcdn.net/hphotos-ash3/537757_563498873668515_468787096_n.jpg


Spyrit will probably argue with me about this by pointing out the enormous variability in weights, but I think 12 lbs is more realistic / historical for a shirt of mail. Mail probably wouldn't be the ideal type of armor for a cat though. Mail is

Most people's exposure to mail is STILL butted mail which is not historical and tends to be 2-3 times as heavy as the historical mail which was always riveted. You do need to wear a belt with mail though and mail is almost always used with some kind of textile armor (a padded coat)

G

Spiryt
2013-03-09, 11:21 AM
It's important to remember that the mail is only one component of the full armor. Just throwing the mail over your everyday clothing would perform quite different from full gear.

That's as always, very debatable topic.

Depends on what we call "everyday clothing" I guess, but there's strong indication that in many places and times, mail would be worn 'just' on solid thick clothing.

Providing support and cushioning body from cold iron (http://www.pancerni.com/Szabla%20i%20kon/Artykuly/Stroj%20bojowy/stroj%20bojowy%20pancernego.php)

In any case, in case of textile padding, weight was probably mainly negligible anyway, it's bulk and thickness was real problem as far as mobility goes.

Rhynn
2013-03-09, 11:32 AM
In any case, in case of textile padding, weight was probably mainly negligible anyway, it's bulk and thickness was real problem as far as mobility goes.

Really? A modern repro gambeson will weigh 8 lbs., and if anything I'd doubt they make them thick enough. Obviously the likelihood of surviving examples is probably nil, but is there any indication what they actually weighed?

Mr. Mask
2013-03-09, 11:35 AM
That is some pretty cool looking cat armour. I doubt my cat would appreciate it from the inside, though.

Why is it you would say mail is not a good choice of armour for cats, G?

Yora
2013-03-09, 11:37 AM
By the 1990's these types of tanks were essentially invulnerable to shaped charge warheads. In fact it's almost the opposite of what Yora said, the only reliable way to knock one out was by the main gun of another tank (tanks, incidentally, can also shoot HEAT / shaped charge warheads).
No, what I was saying was that RPGs are indeed mostly useless. What I was mentioning are missiles, specifically Javelin and Hellfire.
Which of course have the significant drawback of being several times more expensive (like several hundred times).
I think there must be better ways to deal with RPGs than relying on MBTs. (Which apparently, defense companies do too, given the development of all kinds of fancy active defense gadgets.)

Thiel
2013-03-09, 01:05 PM
No, what I was saying was that RPGs are indeed mostly useless. What I was mentioning are missiles, specifically Javelin and Hellfire.
Which of course have the significant drawback of being several times more expensive (like several hundred times).
I think there must be better ways to deal with RPGs than relying on MBTs. (Which apparently, defense companies do too, given the development of all kinds of fancy active defense gadgets.)

Thing is, heavy armour will work no matter the conditions, active defences are far too likely to be hampered by environmental factors, not to mention the danger to nearby friendly infantry.

Galloglaich
2013-03-09, 02:22 PM
That is some pretty cool looking cat armour. I doubt my cat would appreciate it from the inside, though.

Why is it you would say mail is not a good choice of armour for cats, G?

Because, as someone mentioned upthread, it tends to have a bit of it's own momentum which I think would mess up a cats balance (and annoy it) and it also is noisy and a bit distracting in a tactile way because of how it moves. Again, cats being so sensitive and so on, I think it might be a problem. Doesn't necessarily bother humans once they get used to it.

I've seen masks put on Cheeta's, who were used for hunting. Dogs were definitely fitted with armor sometimes historically, for war, usually amounting to some kind of vest and maybe a partial hat. Police and military dogs today are sometimes fitted with ballistic vests. If you could get a cat to wear armor I would guess some combination of textile and plate would work best. I don't think the image I posted was 'real' cat armor, in the sense that it was meant for any serious purpose. It was probably either a 'meisterstücke' (masterpiece) of some journeyman armorer from back in the day (you see some very whimsical things along those lines), done to prove he was ready to be a master armorer. Or some modern joke.

G

Yora
2013-03-09, 05:01 PM
Does anyone have blade measurements for 15th-16th century two handed longswords?

Rhynn
2013-03-09, 07:47 PM
Does anyone have blade measurements for 15th-16th century two handed longswords?

MyArmory (http://www.myarmoury.com/features.html) can help some. The XX is the most applicable, I guess, but XVa too.

Obviously it's going to vary hugely, though. Blade lengths will be anything from 3 to 4 feet or so (3 feet of blade is often considered the lower limit for longswords, but I'm sure some two-handers of the same general types were shorter), which is enormous variation. MyArmory has a lot of very thorough reviews and descriptions of many, many swords (many or most of them reproductions with the info for the original specimen).

A lot of the two-handed longswords are 14th century, though, it seems, both on the site and in Oakeshott's typology.

Here's some examples that fit your criteria (I tried to make sure to only pick the ones that were replicas of historic swords in museums):

http://www.myarmoury.com/review_dt5140.html

http://www.myarmoury.com/review_aa_gbs2.html

http://www.myarmoury.com/review_aa_dur.html

http://www.myarmoury.com/review_cf_at1593.html

http://www.myarmoury.com/review_cf_at1520.html

http://www.myarmoury.com/review_mi_xxa1.html

http://www.myarmoury.com/review_moc_violet.html

warty goblin
2013-03-10, 12:22 AM
You can put barding on a horse because it can handle the weight. A cat's back is too flexible and can't support much weight. A relatively straight-backed hoofed animal can support a lot of weight and animals with arched spines can support a lot. Cats would be pulled down by weight on their backs. Cats also have much, much less endurance than humans and would drop from exhaustion if they had to carry a quarter of their body weight for more than a few minutes.

On the other hand, would you really need to armor the entire tiger? It's not like it's going to be passing beside its targets, it's going to hit them head on. There's maybe some sense to loading up the forward quarters, head and chest with some serious protection, and simply jumping on dudes. At 800lbs the impact alone will pretty well pole-axe a person, long enough for the tiger to get a purchase and tear off something important.

And more importantly, it would look supremely badass in a sword and sorcery kinda way.

Mr. Mask
2013-03-10, 12:47 AM
Places which are likely to hit are priority for armouring. Areas unlikely to be hit, can either be unarmoured or more lightly armoured.

It's quite likely the tiger won't be fighting one on one, however--so the sides, back, and limbs ought to be armoured. The underbelly wouldn't necessarily need to be armoured--perhaps you could hide it with a sort of armoured curtain... except they might get in the way of movement. (it seems to work for horses, but it mightn't for cats).


Even with the problem of mail causing problems with momentum, perhaps it may be preferable, still, to cutting the cat's flexibility? I think a clever armourer could work out some kind of belt-like system. The armour might have a solid frame, to try and keep the mail in check, perhaps?

Spiryt
2013-03-10, 04:26 AM
Does anyone have blade measurements for 15th-16th century two handed longswords?

Wallace Collection has plenty of late, 16th century longswords.

Dunno what exactly interests you, 15th century in particular is obviously period of huge popularity and diversity among longswords all around Europe.

They have blade lengths, usually without overall lengths though (http://wallacelive.wallacecollection.org/eMuseumPlus?service=ExternalInterface&module=collection&objectId=60975&viewType=detailView)

Kalaska'Agathas
2013-03-10, 08:06 PM
Does anyone here have any information on the Girandoni Windbüchse and other pneumatic arms, especially their performance compared to gunpowder arms?

Eorran
2013-03-10, 11:24 PM
Places which are likely to hit are priority for armouring. Areas unlikely to be hit, can either be unarmoured or more lightly armoured.

It's quite likely the tiger won't be fighting one on one, however--so the sides, back, and limbs ought to be armoured. The underbelly wouldn't necessarily need to be armoured--perhaps you could hide it with a sort of armoured curtain... except they might get in the way of movement. (it seems to work for horses, but it mightn't for cats).


Even with the problem of mail causing problems with momentum, perhaps it may be preferable, still, to cutting the cat's flexibility? I think a clever armourer could work out some kind of belt-like system. The armour might have a solid frame, to try and keep the mail in check, perhaps?

The next obvious question becomes "how many people does it take to fit armour on an unwilling 800-pound tiger, and how many do you lose in the process?"

Yora
2013-03-11, 04:28 AM
Does anyone here have any information on the Girandoni Windbüchse and other pneumatic arms, especially their performance compared to gunpowder arms?
I think we had those a few months ago. If I remember correctly, they were supposedly really quite good, being smokeless and working in wet conditions, and most likely also significantly more quite. And penetration was about similiar, I think.

Brother Oni
2013-03-11, 07:33 AM
One thing to keep in mind with mail is that it isn't rigid, and if you don't belt it on it basically sloshes around you. This is annoying when you're just walking, but it's a downright hazard in a fight. Try to stop short and your armor will keep going, dragging you off balance. A good thick belt at the waist goes a long way towards mitigating this, and also reduces shoulder strain as well.

As an expansion to this, mail is almost always worn with a belt, thus making it less carried and more worn, making it more managable.

The additional momentum warty mentions is only of real issue if you're running and have to turn suddenly. On grass with historically accurate leather shoes, it's quite amusing (for me anyway) as you just stand there and watch the heavily armoured guys sail past as if they were on ice. :smallbiggrin:


The next obvious question becomes "how many people does it take to fit armour on an unwilling 800-pound tiger, and how many do you lose in the process?"

Sounds like something that Siegfried and Roy have the resources (and inclination) to try out.

The only armoured tiger I know of is Battlecat from the He Man TV series and the original armour doesn't seem too unrealistic (then again, I don't remember him doing much fighting).


http://he-man.org/assets/images/home_news/motu_he-manbattlecat_main1_full.jpg

And for something a bit more exotic:

http://th00.deviantart.net/fs70/PRE/i/2010/018/d/4/He_Man_and_Battlecat_by_RubusTheBarbarian.jpg


Reading up on how tigers attack their prey, they tend to pounce, then bite the neck (which is different from some other large cats which just grab hold with their front paws and teeth, then use their hind legs to eviscerate their target), so Warty's earlier suggestion of the head and fore quarters is a good start with flanks and back for keeping other assailants.
I'm not so sure on the chest as the videos I've seen suggest they approach low then spring up at the last moment to knock down their prey, well within effective spear range. Since short sword/daggers are too flexible to effectively armour against when grappling with a tiger, it's probably better to keep the weight light and hope the tiger kills the target before the target can deal enough damage with the blade.

Rhynn
2013-03-11, 08:53 AM
Question time.

Self/long bows: it's pretty much all about poundage, but could a person use a bow with "too high a poundage" and still get results? I'm fairly sure a person can't get more power than the poundage of the bow allows into arrows, so being stronger than necessary for a bow would be of no advantage - you need to use the highest-poundage bow you can actually draw and loose over and over.

I'm trying to figure out whether I should alter a game system so that each long/self bow is rated for a specific Strength (representing poundage) and cannot do more damage than that allows, and whether you should be able to use a bow with a Strength higher than your own and just do less damage (possibly with penalties to accuracy?).

Spiryt
2013-03-11, 09:12 AM
Question time.

Self/long bows: it's pretty much all about poundage, but could a person use a bow with "too high a poundage" and still get results? I'm fairly sure a person can't get more power than the poundage of the bow allows into arrows, so being stronger than necessary for a bow would be of no advantage - you need to use the highest-poundage bow you can actually draw and loose over and over.

I'm trying to figure out whether I should alter a game system so that each long/self bow is rated for a specific Strength (representing poundage) and cannot do more damage than that allows, and whether you should be able to use a bow with a Strength higher than your own and just do less damage (possibly with penalties to accuracy?).

Well, in short, selfbows are definitely not 'all about poundage' - you can easily outperform a bow with two times softer one if the light weight bow is supremaly designed, and the heavy one is uneven, erroneously bending piece of branch.

As far as overdrawing bow goes, it definitely does no good, especially in selfbows.

Bow can store as much energy as it's designed to, as well as bend as far as it's designed to bend - applying more force and drawing it further will always result in some kind of damage.

Rhynn
2013-03-11, 09:39 AM
Well, in short, selfbows are definitely not 'all about poundage' - you can easily outperform a bow with two times softer one if the light weight bow is supremaly designed, and the heavy one is uneven, erroneously bending piece of branch.

Let's assume competently made bows. (And we can pretty much ignore Asian composites, at least for my case.)

What about underdrawing a heavy bow?

Yora
2013-03-11, 09:50 AM
I think the problem with underdrawing is the strain on the arm when holding the string pulled. At optimal draw length you could somewhat "lock" your shoulder and wouldn't have to keep your elbow at an angle. That should make it quite a bit easier to hold the arrow drawn, which also means it would be more steady.
If the bow is made in a way that it could be drawn farther than your arms can reach, that should not be a problem at all.
Though this is just an untested assumption on my part and I'm not super-competent with english archery terminology.

Brother Oni
2013-03-11, 10:01 AM
The problem with being overbowed (that is, using a bow that is of a too high poundage draw for you), is that your accuracy goes to pot, since you're struggling to hold the bow still.
This doesn't just affect arrow placement but the distance your arrow will fly along the same angle since you can't consistently put the same amount of power into it (an advantage crossbows have over bows).

I suppose if you could consistently under-draw an overly heavy bow (self bows do have a generally linear increase in draw poundage over distance), you'd still be accurate, but given that all self bows are rated for x poundage at y draw length, unless your arms are particularly short (or the bow is designed for somebody with the arm length of a gorilla), you're not going to be able to pull it off.

Additionally, because you're messing about with your draw length, your required arrow length is going to change. There was a question on 'could a bow injure somebody at point blank range' a while ago and the closest injury that could answer this question without actually doing it, was under-length arrows impaling your hand holding the bow on release (it looks as painful as it sounds).


I think the problem with underdrawing is the strain on the arm when holding the string pulled. At optimal draw length you could somewhat "lock" your shoulder and wouldn't have to keep your elbow at an angle. That should make it quite a bit easier to hold the arrow drawn, which also means it would be more steady.

I'd like to point out that proper form involves you drawing with your back muscles, not your arm or shoulder, but given we're talking about a hypothetical situation with a novice archer that's already over-bowed, proper form is the least of our concerns (finding consistent references would be second behind getting the right bow).

Assuming that it's a competent archer who's just picked up the wrong bow, they would probably adjust their draw length to something that they could hold comfortably (with new references), thus could probably still shoot relatively normally (they'd be much better with their own bow, or at least one of proper poundage and draw length).

warty goblin
2013-03-11, 10:44 AM
Reading up on how tigers attack their prey, they tend to pounce, then bite the neck (which is different from some other large cats which just grab hold with their front paws and teeth, then use their hind legs to eviscerate their target), so Warty's earlier suggestion of the head and fore quarters is a good start with flanks and back for keeping other assailants.
I'm not so sure on the chest as the videos I've seen suggest they approach low then spring up at the last moment to knock down their prey, well within effective spear range. Since short sword/daggers are too flexible to effectively armour against when grappling with a tiger, it's probably better to keep the weight light and hope the tiger kills the target before the target can deal enough damage with the blade.
I was operating under the assumption that getting hit in the chest with 800lbs of iron-clad tiger would knock the fight out of a person pretty effectively. I'm not speaking from direct experience here, but that's a lot of impact to absorb.

The more problematic part from the tiger's point of view is any sort of throat and head covering full helm. Something like a great helm, great bascinet or sallet with bevor rather rules the throat out as a target, and would make getting enough of a purchase to cause major head trauma rather difficult. I don't know if a tiger's bite force is enough to crush a helmet, but even if it is that's going to be pretty hard on the dentistry.

hamishspence
2013-03-11, 10:57 AM
Tigers usually go for the nape of the neck on small animals- monkeys and the like (going by Wiki).

Jaguars have a meaner bite even than tigers- and are the only big cat that commonly goes in for skull crushing.

warty goblin
2013-03-11, 12:41 PM
Tigers usually go for the nape of the neck on small animals- monkeys and the like (going by Wiki).

Jaguars have a meaner bite even than tigers- and are the only big cat that commonly goes in for skull crushing.

Interesting. I wouldn't think the nape of the neck would be a particularly appealing target on a human in a lot of ways, since its less accessible than the throat. On the other hand, I don't suppose once a tiger's knocked you over, you're going to keep it from flipping you over and biting the back of your neck either. But a lot of helmets that cover the throat also protect the back of the neck, so the point about needing to get through the steel still stands.

hamishspence
2013-03-11, 02:06 PM
I recall reading that tigers tend to attack humans from behind as well- which was why people who needed to go into the forest started wearing eyed masks on the backs of their heads- to give the tigers the illusion that they were being watched, and discourage them from attacking.

Joran
2013-03-11, 06:29 PM
Not really, only that it's ahead of its testing schedule


Like anti tank weapons SAM development isn't standing still. If the USAF want to continue to be able to carry out operations Like Desert Storm then
they need to keep up. Buying legacy fighters might be easier in the short run, but it's a bad idea in the long run.
The marines are also going to be in serious trouble without the F35B since Harrier II production stopped in 2003 and even with the acquisition of most of the former RAF and RN airframes they're going to run out of spares within a couple of years.

All my information is from articles like this, describing the shortfalls and missteps of the JSF program: JSF WaPo article (http://www.washingtonpost.com/world/national-security/f-35s-ability-to-evade-budget-cuts-illustrates-challenge-of-paring-defense-spending/2013/03/09/42a6085a-8776-11e2-98a3-b3db6b9ac586_story.html). From this article, it sounds like the entire idea of a JSF was too ambitious (70% commonality seems like it was way too high, it's now more like 30%). Coupled with some old fashioned poor oversight and you have a program that went off the rails a few years back.

Supposedly, the program is back on track now with planes being delivered, but they're going to have to go back and retrofit the planes when flaws are discovered during testing. The important thing is that the JSF is the only game in town; too many eggs have been put in the basket for the JSF to fail now. Developing alternatives will take a decade or more, time that may lead the current generation of fighters to become obsolete before the new planes come online. Air Forces like Australia are buying new F-18 Super Hornets to replace aging F-18s to fill the gap before the JSF is ready, but it really seems like the JSF will finally come to fruition.

On the plus side, the JSF isn't actively trying to kill its pilots like the F-22 XD

Thiel
2013-03-11, 07:05 PM
but they're going to have to go back and retrofit the planes when flaws are discovered during testing.
Yes? They also had to do that on the Buccaneer, Harrier, SeaVixen, Hunter, F100, F104, F86, F117, F16, F18, F22, F5, A3, A4, A6, A7, Super Etendard, Mirage II, Mirage III, Kfhir, Mig 15, Su-27, Draken, Tornado, Viggen, Tunnan.
See a pattern? It's called testing and it's the reason why preproduction runs exist.

Galloglaich
2013-03-11, 10:24 PM
Question to native German speakers or WW II historians. Does the old German word for tank panzerkampfwagen mean literally what it seems to mean, i.e. 'armored war wagon', and if so, could that possibly have anything to do with medieval war wagons like those used by the Czechs and later the Germans themselves, such as we see here:

https://commons.wikimedia.org/wiki/File:Hausbuch_Wolfegg_53r_53r1_Heerlager.jpg

G

Mathis
2013-03-11, 11:31 PM
I'm not a native German speaker, but I do speak German rather well and panzerkampfwagen does translate to armoured war/combat wagon yes. Wagen is used more to describe what in English is vehicle, although it literally means wagon. So a better translation I suppose would be armoured combat vehicle. I use combat because kampf isn't necessarily war, krieg is used if you want to describe the word war. While I am also no historian I have to assume that the word has no direct link to old war wagons other than that the word wagen has probably been used to describe a vehicle for hundreds of years, possibly dating back to old high German, the German language until roughly the end of the first millenium. So the word is probably linked because of what it describes, not necessarily because a tank and war wagon serve somewhat similar tasks on the battlefield.

Brother Oni
2013-03-12, 03:08 AM
I recall reading that tigers tend to attack humans from behind as well- which was why people who needed to go into the forest started wearing eyed masks on the backs of their heads- to give the tigers the illusion that they were being watched, and discourage them from attacking.

There's two reasons for this - tigers and other ambush predators are easily discouraged if they're seen too far away to make a strike thus they're more likely to slink off to try and find easier prey (I was watching a documentary about a troop of mixed species of monkeys when one of them spotted a hiding jaguar and they all chased it off doing nothing more than pointing and shouting their species' equivalent of "There's a leopard over there, there is!").

The second reason is that a lot of animals judge the height of other animals by how high their head/eyes are off the ground. Humans, being upright bipeds, often appear intimidatingly tall to other animals and they ignore the fact that we're lightweight spindly apes compared to the average big cat.
This is why tigers tend to attack humans from behind and the nape of the neck is easiest neck target when attacking from that angle, rather than they intentionally flip people over once they've been knocked down.


Does the old German word for tank panzerkampfwagen mean literally what it seems to mean, i.e. 'armored war wagon'...

Further to Mathis' and possibly Yora's posts, the Germans are fond of bolting words onto the end of other ones to make very long technical terms (Sonderkraftfahrzeug or Sd.Kfz for 'Special Purpose/Ordnance Vehicle' for example).
Like in other languages, the more words that get bolted on, the further away from their source etymology each individual word becomes, until you end up with nonsense, S****horpe being an excellent example. :smalltongue:

My own question for German speakers - what's the new German phrase for Main Battle Tank? I keep on coming up with 'kampfpanzer' as a shorthand for 'panzerkampfwagen', but Galloglaich mentioned that's the old term for tanks.

AMX
2013-03-12, 04:34 AM
My own question for German speakers - what's the new German phrase for Main Battle Tank? I keep on coming up with 'kampfpanzer' as a shorthand for 'panzerkampfwagen', but Galloglaich mentioned that's the old term for tanks.

"Kampfpanzer" is correct.

"Panzerkampfwagen" is the old, do-not-use-or-people-think-you-are-talking-about-WWII term.

Yora
2013-03-12, 05:32 AM
I'm not a native German speaker, but I do speak German rather well and panzerkampfwagen does translate to armoured war/combat wagon yes. Wagen is used more to describe what in English is vehicle, although it literally means wagon. So a better translation I suppose would be armoured combat vehicle. I use combat because kampf isn't necessarily war, krieg is used if you want to describe the word war. While I am also no historian I have to assume that the word has no direct link to old war wagons other than that the word wagen has probably been used to describe a vehicle for hundreds of years, possibly dating back to old high German, the German language until roughly the end of the first millenium. So the word is probably linked because of what it describes, not necessarily because a tank and war wagon serve somewhat similar tasks on the battlefield.
All correct. Wagen has been used as a term for cars for a very long time. It's only in recent generations that Auto has started to replace it, but it's still used a lot.

Kampfpanzer (combat tank) is indeed the correct term.


In D&D, bows are super expensive, which I think is based on what a great deal was made about the english and welsh military longbows which were required in proto-industrial quantities. But you can make a bundle-bow pretty much out of stuff you find lying around. What I am wondering is, how much are they actually inferior to "regular" bows made out of a single piece of optimal wood? The power of a bow is always simply the power of your arms and the aim would depend on your skill and your arrows. So I don't see them performing significantly worse.
Are there issues about durability? I guess when you make them out of freshly cut branches, the wood would eventually dry and become prone to cracking. Would they be useable for a timespan of a couple of years, or more like months?

Mathis
2013-03-12, 08:30 AM
Perhaps interestingly to some, in Norwegian, Danish and Swedish , three closely related germanic languages whose respective cultures have had a lot to do with the baltics through the years uses a word that translates literally to combat wagon. Stridsvogn, kampvogn and stridsvagn. If speaking about older tanks and actual war wagons the word krigsvogn is used, at the least in Norwegian. Same deal with German here though, to my knowledge there is no real connection other than using similar roots to the word.

Yora
2013-03-12, 08:36 AM
We do have the word Streitwagen, but in German it means a charriot.

Spiryt
2013-03-12, 08:58 AM
In D&D, bows are super expensive, which I think is based on what a great deal was made about the english and welsh military longbows which were required in proto-industrial quantities. But you can make a bundle-bow pretty much out of stuff you find lying around. What I am wondering is, how much are they actually inferior to "regular" bows made out of a single piece of optimal wood? The power of a bow is always simply the power of your arms and the aim would depend on your skill and your arrows. So I don't see them performing significantly worse.


Well, here's the main point - they will perform significantly worse.

Power of bow is not 'simply the power of your arms' - you can very easily make a bow that requires applying 1000 pounds of force, yet you will still be better off throwing your arrows by hand. :smallwink:

Making 'bundle bow' is actually not simple at all, especially gluing the pieces together without loss of performance or without all falling apart.


I guess when you make them out of freshly cut branches, the wood would eventually dry and become prone to cracking.

Bows as a rule are not made out of branches, unless it's some really damn huge branch. They are being cut out of the piece of larger piece of wood.

I'm not really knowledgeable about wood, but I don't think that any freshly cut branch has even remotely right wood structure for a bow...

Obviously, many species of woods are not very good for bows in general.

Most made of 'poor mans' piece of wood simply won't be very dynamic, accurate or convenient to use.

Yora
2013-03-12, 09:02 AM
Well, for a bundle bow you would need pieces of wood under a meter long and about thumb-thickness. Yes, those would most commonly be actually be stems of a plant and not "branches". Still sticks.:smallbiggrin:

Are there any "price comparisons" between bows and swords, say around 13th century?

warty goblin
2013-03-12, 09:31 AM
Well, for a bundle bow you would need pieces of wood under a meter long and about thumb-thickness. Yes, those would most commonly be actually be stems of a plant and not "branches". Still sticks.:smallbiggrin:


Not all woods - even from the same species or tree - are created equal. Most branches you cut of that size will be recent growth, which means it will tend towards the soft and flexible. Heartwood, cut from the trunk or old growth branches as a generality will be much less flexible, much more rigid.

What you're looking for in a bow is actually a fairly demanding set of material properties. It needs to be flexible so as not to break, but have a strong 'memory' so it wants to spring back true. Oak is fiercely rigid but is liable to split violently apart if you bent it for a bow. Willow is marvelously flexible, but has very little memory. Cut a bow from willow, and it won't spring back to its original shape when you release the string.

This is ignoring the different forces acting on the bow. The outside of the span will be under expanding force, the inside under compression. Different materials react differently to these. The reason yew was favored for longbows is that they could cut the bow out half from the heartwood, half from the sapwood. One was strong under compression, one under expansion - I can't remember which for sure but I think it was heartwood for compression. Despite being a 'self' bow, the longbow was essentially a one-piece composite.

The shape of the bow also matters. I'm even less knowledgeable about how that plays out though. So while you could take some random branches and tie them together into a bow-like object, it's probably not going to be a particularly good or durable weapon.

Spiryt
2013-03-12, 09:51 AM
Someone who knows his bows will probably correct me, but I believe that it's obviously extremely improbable for any stem or branch to actually act - bend, store energy, release it's energy etc. - like actual bow.

Bow has to be profiled, extracted, 'freed' from the piece of wood.


And selfbows in general seem r cheap, not in the same price category as swords, or more sophisticated crossbows. Although my sources are mainly about Polish/Silezian etc. teritory, where bows seemed to be marginally popular in High and Late Medieval, at least as war weapon.

Rhynn
2013-03-12, 11:54 AM
Well, for a bundle bow you would need pieces of wood under a meter long and about thumb-thickness. Yes, those would most commonly be actually be stems of a plant and not "branches". Still sticks.:smallbiggrin:

Are there any "price comparisons" between bows and swords, say around 13th century?

Out of Lisa J. Steele's Fief (which got the price example lists from a host of scholarly sources that takes up two pages of the book, but as a RPG-oriented sourcebook lacks inline citations):

Long bow 1227: 12d - 1s 6d
Long bow 1324: 6s 8d
Long bow 1347: 1s 3d
Long bow, unseasoned 1418: 1s 6d
Long bow, seasoned 1418: 2s
Long bow, yew 1475: 3s 4d
Long bow, 1480: 2s
Arrow 1300-1305: 1/4 d for 2
Arrow 1347: 1s 4d for sheaf (?)
Arrow 1480: 1½ d

By comparison:
Sword 1324: 3s 4d
Lance 1300-1305: 6d
Lance 1337: 3d
Crossbow 1277: 3s-7s
Bolts shafts 1277: 26s - 34s 4d per 1000
Bolt iron heads 1277: 14d - 16d per 1000
Habergeon 1200: 13s 4p
Habergeon 1324: 2l 10s
Helmet, war 1350: 10s
Bascinet 1350: 13s
War horse 1216: 14-50 l
War horse 1337: 28l 12s
War horse 1339: 64l 8s
War horse 1359: 36l

From prices for items for a particular tournament at Windsor:
Arms and armor 1278: 7s - 25s
Sword: 7d
Shield (unpainted wood): 5d

1 l (livre) = 12 s (sou) = 240 d (dernier)

Those prices are mostly picked out of one source or another that mentions how much an item cost someone somewhere at some point (all are in England), so there's a whole lot of circumstantial variaton (and, obviously, prices and values changed hugely over a period of 100-200 years)

Joran
2013-03-12, 12:11 PM
Yes? They also had to do that on the Buccaneer, Harrier, SeaVixen, Hunter, F100, F104, F86, F117, F16, F18, F22, F5, A3, A4, A6, A7, Super Etendard, Mirage II, Mirage III, Kfhir, Mig 15, Su-27, Draken, Tornado, Viggen, Tunnan.
See a pattern? It's called testing and it's the reason why preproduction runs exist.

Right, but for the JSF, they're mass producing the planes before testing is complete. By the time testing is finished in 2017, the U.S. military will own 365 JSFs. I was under the impression that this isn't normal in these large numbers for a pre-production run.

Yora
2013-03-12, 12:56 PM
Sword 1324: 3s 4d (724d)
Sword: 7d
This one seems to make the whole list rather pointless. A price varying by a factor of 100?

The main question is, how many bows do you get for one sword?

Spiryt
2013-03-12, 01:23 PM
There's nothing about period and place of that second sword, so we cannot tell how their price actually compares.

Currencies also varied a lot.

Anyway, variability by factor of 20 are perfectly confirmed, even greater one are probable.

According to this table:

http://www.historycy.org/index.php?act=Attach&type=post&id=10705

Unspecified bow generally cost at least 20 groshen, while swords started from at least 30.

But this is 15th century, probably later one as well, so it has to be kept in mind.

In 13th century, one certain difference would be that sword would be comparably much more expensive.

endoperez
2013-03-12, 01:36 PM
The second sword is actually grouped under the 1278 tournament, so there's a difference of about 50 years compared to the other sword.

Rhynn
2013-03-12, 02:10 PM
Sorry, the second sword (7 d) and all the Windsor tournament entries were 1278.

I actually cut out a few of the sword entries. Here's the full for the swords under "Supplies for a Tournament held at Windsor, England":

Sword (whalebone) 1278 7 d
Sword (silvered whalebone) 1278 25 s 7 d
Sword (silvered whalebone w. gilded hilt) 1278 28 s 13 d

Given that "arms and armor" for the tournament is 7 s - 25 d, and would probably include swords, the 7 d sword doesn't seem outrageous. But since this is a tournament, we might not be talking proper swords anyway. (The other entries under this header include a leather helmet and a gilded leather helmet.)


This one seems to make the whole list rather pointless. A price varying by a factor of 100?

EDIT:
Whoah, major reading fail on my part, ignore some of the following (leaving it up since I wrote it). The sword price goes from 7d to 3s 4d. That's 7d to 40d. That's a multiplier of 5.5 or so. So uh no, that price did not change by a factor of 100, but see below for one that did...
/EDIT

I assume you mean the warhorse? Yes, that's an enormous variation, but warhorses are definitely something where you would see a giant variation. The leap from 10-40 s (a perfectly understandable range) in 1160-1172 to 14-50 l in 1216 may, for instance, indicate a shift from English palfreys to destriers imported from the continent; and many of these are going to be the prices paid in some specific transaction, with no context.

Also, even in the same year, you would probably have seen giant variation between prices specific knights and lords paid for warhorses. Think about it - what's the cheapest you can get a working car for? What's the most expensive? Having a very expensive warhorse was a prestige thing for some lords.

Prices varied enormously. I suppose it's possible there's a typo at work in the case of the first warhorse, but given the 40 s price that sort of supports that range, I doubt it, though.

You should get a load of the regional variations. An ox in England sometime in the 14th century cost 13 s 1 d. In 1348-9, it was as low as 9 s 4 d (in Waltham, England). Meanwhile in 1306 Siena, Italy a price of 16 l 10 s is recorded. For 1326-50 in Pistoia, Italy, the average is 19 l 8 s. By 1376-1400, the average is 48 l 6 s!

Money was not convenient and of a set value in the Medieval period. Indeed, they wouldn't have been using cash money at all, most likely - in the Medieval period, particularly for this sort of purchases, it would have been "money of account", i.e. a value that you can compare other things against. Trade was mostly done in debt if you were a single individual buying and selling for yourself.


The main question is, how many bows do you get for one sword?

I'd just go with ½. That is, a long bow costs twice what a sword does. 1324 is the one where we know the price of a sword and don't have cause to think it might be some kind of "tournament" sword, and that's the ratio there.

It's bound to be inaccurate as a generalisation, but unless you can get much, much more examples of prices of items from one period, for several periods, you can't do better than a generalisation...

Fortinbras
2013-03-14, 11:50 AM
Two questions.

1. In the book "Early Carolingian Warfare" Bernard Bachrach claims that the early Frankish "phalanx" relied primarily fighting in the Roman way (big shield, javelin, short sword) and that there don't seem to be much in the way of records referring to Carolingian infantry, like the men who fought for Charles Martel, using spears in close combat. Bachrach speculates that this was because spears were liable to break and this resulted in a loss of cohesion as some men discarded their broken weapons and pushed forward to use swords while other men continued to fight at a distance with spears. Bachrach also wonders if the Franks didn't use "hedgehog" like formations because they weren't facing heavy cavalry. This led me to wonder if there was such thing as a sort of "general purpose" spear, one that could serve as a javelin when engaging infantry or planted in the ground to ward of cavalry.

2. Does anybody have any good information about the specific layout of a Roman army barracks. Illustrations and diagrams would be really helpful, and at the moment I'm more interested in detailed information than focus on any particular time period.

Thanks

Storm Bringer
2013-03-14, 01:32 PM
Two questions.

1. In the book "Early Carolingian Warfare" Bernard Bachrach claims that the early Frankish "phalanx" relied primarily fighting in the Roman way (big shield, javelin, short sword) and that there don't seem to be much in the way of records referring to Carolingian infantry, like the men who fought for Charles Martel, using spears in close combat. Bachrach speculates that this was because spears were liable to break and this resulted in a loss of cohesion as some men discarded their broken weapons and pushed forward to use swords while other men continued to fight at a distance with spears. Bachrach also wonders if the Franks didn't use "hedgehog" like formations because they weren't facing heavy cavalry. This led me to wonder if there was such thing as a sort of "general purpose" spear, one that could serve as a javelin when engaging infantry or planted in the ground to ward of cavalry.


it appears so, yes, in that i am sure i have heard references to spears that were used both as throwing and as melee weapons.


2. Does anybody have any good information about the specific layout of a Roman army barracks. Illustrations and diagrams would be really helpful, and at the moment I'm more interested in detailed information than focus on any particular time period.

Thanks


a quick google shearch shows a few good hits.... (http://www.google.de/search?q=roman+barracks+plan&hl=en&source=lnms&tbm=isch&sa=X&ei=GxRCUbv_M5GLswb7uYGYCg&ved=0CAcQ_AUoAQ&biw=1311&bih=597)

on a specific note, have you tried looking up some of the old fort remains in the north of england? Here (http://www.english-lakes.com/hardknott_roman_fort.htm)is a good air picture of the old auxilary fort at Hardknott, apprantly home to aprox 500 cavalry form the Dalmatian area (in the Balkans, now part of Croatia). Here (http://viewfinder.english-heritage.org.uk/search/detail.aspx?uid=75705)is Housesteads, one of the forts on Hadrian's Wall, the defensive line between roman england and the Scottish Picts to the north.

in both images, you may assume that most of the empty space inside the walls was occuiped by wooden structures that have not survived.

Here (http://www.cpat.org.uk/educate/leaflets/romans/fort.gif)is a diagram of a generic roman fort,

Galloglaich
2013-03-14, 09:09 PM
Sorry, the second sword (7 d) and all the Windsor tournament entries were 1278.

I actually cut out a few of the sword entries. Here's the full for the swords under "Supplies for a Tournament held at Windsor, England":

Sword (whalebone) 1278 7 d
Sword (silvered whalebone) 1278 25 s 7 d
Sword (silvered whalebone w. gilded hilt) 1278 28 s 13 d


Whalebone swords aren't really "swords", they are flexible sparring weapons used for tournaments, the medieval equivalent of foam rubber and duct tape, or maybe rataan. The 'whalebone' was actually baleen, the mouth fibers of filter feeder whales.

G

Rhynn
2013-03-15, 04:22 AM
Whalebone swords aren't really "swords", they are flexible sparring weapons used for tournaments, the medieval equivalent of foam rubber and duct tape, or maybe rataan. The 'whalebone' was actually baleen, the mouth fibers of filter feeder whales.

G

Thank you! I had no idea - never heard of that. That explains that, then, and takes the 7 d figure off the table.

Matthew
2013-03-15, 05:18 AM
Prices varied enormously. I suppose it's possible there's a typo at work in the case of the first warhorse, but given the 40 s price that sort of supports that range, I doubt it, though.

When looking at this stuff you have to be aware of how English coinage changed in the period 1200-1500. There is some "super inflation" going on in what had previously been a very stable currency, especially compared to France. Differences in price c. 1275 to 1350 might be explained by this. War horses, on the other hand, have always seemed to vary enormously in price, but we do have war records for the cost of replacing horses, a burden taken on by the king, which makes for a better idea of the "average" cost. If you can be bothered to wade through this thread it has some interesting information: [AD&D] Campaign Economics (http://www.dragonsfoot.org/forums/viewtopic.php?f=42&t=29651)

Rhynn
2013-03-15, 08:07 AM
When looking at this stuff you have to be aware of how English coinage changed in the period 1200-1500. There is some "super inflation" going on in what had previously been a very stable currency, especially compared to France. Differences in price c. 1275 to 1350 might be explained by this. War horses, on the other hand, have always seemed to vary enormously in price, but we do have war records for the cost of replacing horses, a burden taken on by the king, which makes for a better idea of the "average" cost. If you can be bothered to wade through this thread it has some interesting information: [AD&D] Campaign Economics (http://www.dragonsfoot.org/forums/viewtopic.php?f=42&t=29651)

I'm generally aware of the inflation effect (it's very obvious in some cases, like the Italian prices; and the general trend seemed to be prices going up as time passed), yeah, but thank you for the link! Bookmarked for sure. And yeah, some of the warhorse prices seem to be averages for certain periods.

Galloglaich
2013-03-15, 08:40 AM
Sword prices definitely went down over time, as their quality generally went up. In the 8th Century a sword could cost as much as several villages, by the 12th they were common enough that peasants were ordered to bring them to muster in some rural levies. By the 15th Century I have a price of 1/2 mark for a sword (120 pence I think, or 20 kreuzer) and 1 mark for a crossbow in Poland.

Of course with gold inlay and special workmanship it could be much much more, but that 1/2 mark probably got you a pretty nice weapon.

I agree the horse seems to have potentially been the most expensive bit of kit. And also that the currency values seem to have fluctuated enormously, often due to devaluing.

G

Kurien
2013-03-17, 09:43 AM
Looking for and examining fantasy artwork featuring knights and other heavy armour wearers, I often see armour which seems improbable. If I ignore the actual amount of the body that is covered, I still wonder how comfortable the armour may be, how it is donned and so forth. For some depictions, one might think that the artist(s) have no clue how armour works.

I'm currently interested in the sophisticated steel plate harnesses of 15th century Europe. Could someone explain to me how the various components of these armours were put on, the straps, cords, fasteners that kept them attached, the appropriate weight distribution, etc?

Pictures would be most helpful, as I wish to use the information to draw and design better fantasy armours, ones that are practical but still look cool.

Thanks a bunch! :smallsmile:

Rhynn
2013-03-17, 09:53 AM
Later harness usually went over an arming coat/doublet/jacket, gambeson, or aketon, which would probably have attachment points for straps, and might even have had mail in places that wouldn't be covered by the harness plates.

Diagram (http://25.media.tumblr.com/tumblr_m19g1fs8YP1r49ix5o4_1280.gif) (gif picture) that does a decent job at showing off a lot of the straps.

Some of the more obscure points... the greaves (knee to ankle) are two-sided (not a given), and consist of two halves that are strapped together at the sides of the calf/shin. The lower cannons on the arms (elbow to wrist) are similar, as is the breastplate. (Again, all of these would sometimes be one-sided; the back might be "open.")

Another diagram (http://www.southtower.on.ca/armour/images/DiagramArmour4.jpg) (jpg picture), similar armor.

If you want to see completely awesome-looking and realistic fantasy armor, check out Mark Smylie's Artesia comic (with the exception of Artesia's bare ass in the first few issues), or even his D&D 3.X illustrations (in Complete Warrior and Faiths & Pantheons, at least).

Realistic (http://www.erzo.org/shannon/images-rpg/artesia2-2.jpg) and cool (http://artesiaonline.com/blog/wp-content/uploads/background/8.jpg) (jpg pictures).

endoperez
2013-03-17, 02:01 PM
Looking for and examining fantasy artwork featuring knights and other heavy armour wearers, I often see armour which seems improbable. If I ignore the actual amount of the body that is covered, I still wonder how comfortable the armour may be, how it is donned and so forth. For some depictions, one might think that the artist(s) have no clue how armour works.

I'm currently interested in the sophisticated steel plate harnesses of 15th century Europe. Could someone explain to me how the various components of these armours were put on, the straps, cords, fasteners that kept them attached, the appropriate weight distribution, etc?

Pictures would be most helpful, as I wish to use the information to draw and design better fantasy armours, ones that are practical but still look cool.

Thanks a bunch! :smallsmile:

Depending on how detailed you want to go...

Here's a video of armor being put on:

http://www.youtube.com/watch?v=XP3LiwNWrk0

And here's a video of a blacksmith making the armour:

http://www.youtube.com/watch?v=ZGhfH31ytdU

Galloglaich
2013-03-18, 03:51 PM
I have a question for everybody

http://upload.wikimedia.org/wikipedia/commons/d/d3/Blackdeath2.gif

During the initial outbreak of the Black Death, in 1347-1350, the amount of people killed varied enormously from region to region. Some towns like Florence lost up to 80% of their population or maybe more, while other nearby cities such as Milan and Venice seem to have suffered far fewer deaths and recovered much more quickly.

Other areas which seem to have been at least partly spared included Bruges, Ghent, and several other towns in Flanders, and the city of Krakow as well as several other towns in Silesia and Poland.

My question is why. I am not able to find much out in English and I suspect a lot of the good information is better known to natives of these different areas, Northern Italy, Belgium, Poland. And any other places I haven't heard of. So I'm particularly interested in the perspective of people from these areas in Europe.

Relative success in Milan and Venice have been attributed to strict quarantine efforts, but not that much detail seems to be available about this. Answers about Flanders and Poland seem to be much more vague, probably for the usual reasons that so little is available in English in general about these areas. It has been alleged by some authors that Flanders only suffereed about a ten percent mortality from the Black Death, based on tax records. Others claim it was as high as 27% which is close to the European average. I'm not sure which verion to believe. In Poland, some people have attributed the Jewish population in Krakow somehow having an effect on their experience of the plague, but plague did hit hard there later in the 15th Century so I'm not sure if that works as a reason. Others have asserted that Poland established quarantine on their borders, if that was true, I'd be really interested to see some details and / or evidence.

I'm interested to learn more about those outbreaks though, and any hard numbers on death rates in different towns or regions.

G

Spiryt
2013-03-18, 04:55 PM
I"ve heard about many different theories - dunno if anybody can now tell for sure what had really happened.

Supposedly, plague could only reach the regions with black rat being prevalent - because brown rat parasites don't attack people.

About people running from to the forests.

Weird theories about Jews being very rigorous about hygiene and this fact somehow helping overall population. :smallconfused:

Also about population being somehow resistant, either by facing the plague somehow earlier in unrecorded history, or by some random genetic trait (?)


Every hypothesis has quite large holes in it, obviously.


http://img841.imageshack.us/img841/2039/pestilencespreading1347.png

fusilier
2013-03-18, 06:07 PM
I vaguely remember from one of my history courses that in Eastern Europe the plague didn't travel as rapidly because the communications to those areas were slower. Which either made it run out before it could spread, or appear to be less damaging because the researchers looked at too short a time frame (i.e. the plague took longer to spread in those areas). Not sure about either of those.

For Italy, the Encyclopedia of the Italian Renaissance, doesn't have much to say about it. It does note that thirty percent of the population of Venice died in the Plague outbreak of 1575-77. When talking about the affect that plague had on the arts, it does have this to say:


Yet the black rat and its plague-bearing flea could find a more hospitable environment in the hovels of the poor than in the stone-built houses of wealthy patrons (who, moreover, were often able to remove themselves from areas where plague had broken out). For this reason, perhaps, it is difficult to find, outside Tuscany, evidence of cultural change which could be attributed to plague, and in the Italy of the 15c. and 16c. the main effect of the disease in art is to be found only in the frequent portrayal of the plague saints, Rocco and Sebastion.

So, perhaps, some cities, with more stone-buildings, etc., may have had better sanitation/living conditions, and that helped mitigate the effect of plague?

The Encyclopedia . . . does give a general description of the response to plague:

Preventative measures included the boarding up of infected families, the isolation of sufferers in plague hospitals, the burning of infected clothing, but none worked or mitigated the feeling of hopelessness

Maybe the cities that did better were more efficient with their response?

Daily Life in Renaissance Italy, doesn't have too much to add directly in specific terms of why some cities did better than others. It mentions that the statistics are imperfect, but gives a more detailed description of how the disease spreads and the complex factors involved in plague outbreak.


The lessons of all these facts are several. For one, humans themselves were not a major source of contagion. Rather, it was usually sick rats on the move that brought the disease to an uninfected, vulnerable rat population, from which it spread to humans only when most of the local rats took sick and died. That is why ships, often rat infested, were so dangerous and quarantine so useful. This pattern also shows why locking stricken families in houses was a mistake, for it was the house, with its dying rats and ravenous fleas, that was dangerous, not the inhabitants, even if already infected. Those who fled town . . . thus took the prudent course. At the same time, some of the other plague remedies, such as burning the clothing of the dead or sitting near hot fires[?!], drove off fleas and probably did some good.

Brother Oni
2013-03-18, 07:20 PM
The historical aspect's been covered, let me try the epidemiological approach.

There's a couple of articles from the Medical History Supplemental that might interest you:

Epidemiology of the Black Death and Successive Waves of Plague. (http://www.ncbi.nlm.nih.gov/pmc/articles/PMC2630035/)
The Language of Plague and its Regional Perspectives: The Case of Medieval Germany. (http://www.ncbi.nlm.nih.gov/pmc/articles/PMC2630034/)
Pestilential complexities: understanding medieval plague. (http://www.ncbi.nlm.nih.gov/pmc/articles/PMC2632864/)
Universal and Particular: The Language of Plague, 1348–1500. (http://www.ncbi.nlm.nih.gov/pmc/articles/PMC2630032/)

The first and last articles have some numbers and case studies if you're interested in hard numbers and first hand data.

A quick scan of the articles indicate a number of potential theories for the different spread (this is not exhaustive):

Y. Pestis not being the same causative agent in all three major pandemics (the famous Black Death was the second one, the Justinian plagues (http://en.wikipedia.org/wiki/Plague_of_Justinian) of the 6th Century killed nearly the same number of people although it didn't spread as fast, while the third one barely killed 3% of the population).
Feral cat culls (which resulted in an increased in the local rodent population).
Different strains being sensitive to temperature (epidemic virulence of the bubonic strain is only maintained within 10-25°C, plus the winter of 1348 was especially wet and mild).


The articles are mostly taken up with highlighting discrepancies in the data with the theories (not to mention outright arguing) - I can try and do a TL:DR summary if you like, but I think think they're fairly readable for the layman.

fusilier
2013-03-19, 03:08 AM
The historical aspect's been covered, let me try the epidemiological approach.

There's a couple of articles from the Medical History Supplemental that might interest you:

Epidemiology of the Black Death and Successive Waves of Plague. (http://www.ncbi.nlm.nih.gov/pmc/articles/PMC2630035/)
The Language of Plague and its Regional Perspectives: The Case of Medieval Germany. (http://www.ncbi.nlm.nih.gov/pmc/articles/PMC2630034/)
Pestilential complexities: understanding medieval plague. (http://www.ncbi.nlm.nih.gov/pmc/articles/PMC2632864/)
Universal and Particular: The Language of Plague, 1348–1500. (http://www.ncbi.nlm.nih.gov/pmc/articles/PMC2630032/)

The first and last articles have some numbers and case studies if you're interested in hard numbers and first hand data.

A quick scan of the articles indicate a number of potential theories for the different spread (this is not exhaustive):

Y. Pestis not being the same causative agent in all three major pandemics (the famous Black Death was the second one, the Justinian plagues (http://en.wikipedia.org/wiki/Plague_of_Justinian) of the 6th Century killed nearly the same number of people although it didn't spread as fast, while the third one barely killed 3% of the population).
Feral cat culls (which resulted in an increased in the local rodent population).
Different strains being sensitive to temperature (epidemic virulence of the bubonic strain is only maintained within 10-25°C, plus the winter of 1348 was especially wet and mild).


The articles are mostly taken up with highlighting discrepancies in the data with the theories (not to mention outright arguing) - I can try and do a TL:DR summary if you like, but I think think they're fairly readable for the layman.

That is interesting. I'll try to read some of those when I get the chance. As for the cat theory, it's been around for a while and I've never cared too much for it [cat-lover propaganda ;-) ].* The issue is actually somewhat complicated, because the rat flea doesn't actually like to live on humans, it only does so when the rats are dying. So, paradoxically, if the rat population is carrying infected fleas, but the rats themselves are healthy, then a mass killing of the rats would cause the fleas to "jump ship" to other species. But, in the short version given in my previous post, migrating rats seem to pass the infected fleas onto local, uninfected rat populations. So if a community didn't have much of rat population to begin with, plague might be less likely.

*For a couple of reasons: 1) I've never seen any real study about the effectiveness of cats controlling rat populations, especially in medieval conditions. 2) I believe certain dog breeds (like terriers) are better at rat control. 3) If plague is present in the rat population, cats (and dogs) will merely serve as conduit for the fleas to pass on to humans, through their usually acceptable interaction with humans. While I'll admit that it's possible that the culling of feral cats *may* have had an adverse effect before plague arrived. The culling of them (and again dogs) during an outbreak of plague may actually have been beneficial.

Brother Oni
2013-03-19, 03:50 AM
While I'll admit that it's possible that the culling of feral cats *may* have had an adverse effect before plague arrived. The culling of them (and again dogs) during an outbreak of plague may actually have been beneficial.

I believe the first paper discusses this theory at some length and I see some merit in your arguments, but we would need to check any records on the number of animals killed during these culls before dismissing it out of hand.

That said, it should be remembered that plague kills rats as well, so a massive explosion in the rat population would result in better peneration of human settlements before they become infected, die and the fleas start spreading it to other things (given the general standard of living, there's no reason to suggest that the fleas couldn't jump directly to people after the death of their rodent host).

One of the theories (it's not Y.pestis) is based on some modelling of the spread and the flea vector via rodents just doesn't spread fast enough to match historical accounts.
Additionally, one of the papers comments that if there were so many rats about, why do none of the contemporary accounts mention them - admittedly they may have been otherwise distracted with plague, but the fact remains that no surviving account mentions the required large numbers of rat and their carcasses.

It's not an easy question to answer and the best we'll probably get is 'it's most likely this', given that the causes are still being debated by more informed academics than us.

Spiryt
2013-03-19, 08:13 AM
I vaguely remember from one of my history courses that in Eastern Europe the plague didn't travel as rapidly because the communications to those areas were slower. Which either made it run out before it could spread, or appear to be less damaging because the researchers looked at too short a time frame (i.e. the plague took longer to spread in those areas). Not sure about either of those.


Vast areas Scandinavia and Rus were generally even less populated, urbanized and connected with roads/etc. than Polish states, and some parts of them were absolutely wiped out, while Poland territories were relatively plague free.

That's why it's weird.

Galloglaich
2013-03-19, 09:48 AM
First, thanks to everybody for responding. I'm going to read through those links later.

Here is what I have on it


Vast areas Scandinavia and Rus were generally even less populated, urbanized and connected with roads/etc. than Polish states, and some parts of them were absolutely wiped out, while Poland territories were relatively plague free.

That's why it's weird.

Yes I think this is one of the myths that keeps cropping up, and I think this one at least can be discounted - circa 1350, Poland was probably significantly more urbanized and connected to international commercial trade networks than say, France or most of England was. Krakow was on a major trade nexus.

Some of the other things that usually come up

The cats
I'm inclined to agree with fusilier on that one for the reason he cites, though I'm willing to learn otherwise. I suspect the thing about Medieval people being crazy and killing all their cats is a bit dubious, no doubt some monks and religious fanatics felt that way, but the culls were probably due to out of control populations of feral cats just like we have today. I have seen documents where cats were clearly popular pets in this era... including one manuscript where a monk complains his cat peed on his book.

The black / brown rats
This could be a factor but I'd like to learn more - I agree the speed of the spread of the disease seems to bely the spread of rats but rats can and do explode suddenly in population, and we know how devastating invasive species can be. There was however in the initial outbreak clearly a pneumonic component to the plague (i.e. transmission via breath) which does bypass the rats entirely.

http://en.wikipedia.org/wiki/Pneumonic_plague

Hygeine / Jewish population
It's true that Jewish people were more tolerated in Poland at that time than in most of the rest of Europe, and Jews did have special hygeine rules, but there are two problems here. Jews were also in hundreds of German, Czech, and other central European towns. And hygeine was actually quite good in Medieval Europe contrary to popular myth. People bathed regularly and washed (often boiled) their clothes routinely. Most towns east of the Rhine had good sewer systems and protected water supplies which distributed in fountains throughout the town, industries like abbatoirs and skinners had to be kept outside of the town gates. Paris and London in fact were two of the only big cities which lacked some kind of sewer system.

The average city in central Europe in 1350 was probably substantially cleaner, less crowded, and more tidy than most cities were in 1650 or even 1850. People defintely bathed more and washed their clothes more.

Krakow did have a lot of jewish butchers so they may have possibly have had some effect on food preparation, there seem to have been a lot of laws put in place trying to restrict when and how they sold meat to gentiles. But the christian butchers guilds also had pretty strict guidelines on how meat was handled and prepared, again by and large much better than the norm in the 19th Century for example.

Boarding people up and quarantining
This apparently did happen in both Milan and Venice, it's documented (bricking people in actually, ala Edgar Allen Poe) in Milan and Venice seems to have pioneered using special islands to quarantine people. They even found one of the islands recently (http://news.nationalgeographic.com/news/2007/08/070829-venice-plague.html). Within a year or two of the initial outbreak standard quarantine measures were widely established in many places, usually requiring visiting ships for example to wait for 30 or 40 days in quarantine before coming ashore.

Poland sealed her borders
This is possible, though it seems like it would have been a major organizational challenge especially since her border with the areas controlled by the Tartars was under near constant attack.

Famine
I think this is possible. It may help explain why some pockets in Flanders and Lombardy seem to have escaped, and Poland as well. All three were areas where they had particularly good agricultural techniques in practice, and / or very fertile soil. Peasants in these zones were also fairly prosperous which would contribute to their health. There was also a clear and well understood correlation between famine and plague, this was a known quantity during the 15th Century. In fact plague seemed to routinely break out during wars and especially sieges, so much so that they took steps to outlaw scorched earth tactics after they cause famine which led to war, such as famously in the Hunger War between Poland and the Teutonic Order in 1414 which led to famine immediately followed by 6 years of devastating plague http://en.wikipedia.org/wiki/Hunger_War

But this is all very general. What I'm hoping to find, and please post here if you come across any, are examples of other places which seem to have had low mortality rates, and more details about how the places which did seem to mitigate the effects of the original and most devastating onset of the bubonic plague managed to accomplish that. I'm also interested in examples from the era of Justinians plague or earlier ones such as in Athens and so on, if there is any documentation or hard evidence available.

Venice and Milan seem to have largely avoided or at least mitigated the initial outbreak of plague in their area in 1348, but there is very little information as to how this happened, how the local authorities manged it, other than the general anecdote about walling people up in Milan and putting infected people on Islands in Venice. This is a good clue but there are no sources, no numbers. There are a lot of articles about devastating plague outbreaks in Venice in the 16th and 17th Centuries, but I can find very little about what happened the first time, during the Black Death.

Virtually nothing seems to be available about Flanders or the Flemish cities, other than one interesting power point presentation I found. And nothing about Poland except these vague ideas we discussed already.

If anyone runs across more details on these things, especially localized as opposed to general information, please post in this thread. I'm also interested in the biological warfare angle if anyone has anything on that.

G

Galloglaich
2013-03-19, 09:56 AM
Every hypothesis has quite large holes in it, obviously.


http://img841.imageshack.us/img841/2039/pestilencespreading1347.png

What does your map here say about the towns indicated in blue? Those seem to be some of the worst hit from my info... ...?

The zones in green were the relatively plague free zones obviously.

G

Spiryt
2013-03-19, 10:45 AM
Here's full resolution

http://upload.wikimedia.org/wikipedia/commons/2/29/Pestilence_spreading_1347-1351_europe.png

It means town centers riots, apparently, so yeah.

Brother Oni
2013-03-19, 12:27 PM
There was however in the initial outbreak clearly a pneumonic component to the plague (i.e. transmission via breath) which does bypass the rats entirely.


The problem is, pneumonic plague isn't very contagious - its risk of transmission is 0.25-0.55 (on average, how many people an infected person will pass it onto), compared to very infectious diseases like influenza (1.8-10), measles (10-15) or pertussis (16-18).

Even so, the worst epidemic of pneumonic plague (Manchuria 1911) had a fatality rate of 0.3% of those affected and that was in fairly optimal conditions for its spread (the people were travelling on train cars during winter).



I'm also interested in the biological warfare angle if anyone has anything on that.

Are you talking from a historical perspective or a modern one?

I don't know of any specific medieval use except by launching infected bodies over castle/city walls during a siege and hoping something catches.
More recently, Unit 731 of the IJA air dropped infected fleas over Changde during WW2, which caused some outbreaks.

From a modern perspective, plague is a poor bioweapon compared to others available (anthrax for example).
There's a paper on its possible use as a bioweapon from 2000:
Plague as a biological weapon: medical and public health management. Working Group on Civilian Biodefense. (http://www.ncbi.nlm.nih.gov/pubmed/10807389)

Its conclusions are based on an aerosolised form of the disease (thus the pneumonic variant initially) and estimates that symptoms would start within 1-6 days from exposure, with death occurring within 2 to 4 days after onset of symptoms.
What isn't mentioned, is that Y.pestis is fairly vulnerable to antibiotics (although strains with some resistance have been noted in the wild), so correct diagnosis and early treatment (it would initially appear to be pneumonia) would significantly reduce its effectiveness.

Did you want me to go further into the bioweapons aspect, or is this enough?

Edit: And I'm now probably on some government watchlist somewhere....

Octopusapult
2013-03-19, 12:37 PM
I've got a question. Does the Duom actually exist? And if so, when was it ever used?

And if it doesn't exist, is there a practical application for it? To me it seems like it just wouldn't be combat effective.

(Duom can be found in the D&D 3.0 Arms & Equipment Guide)

Brother Oni
2013-03-19, 12:51 PM
I've got a question. Does the Duom actually exist? And if so, when was it ever used?

And if it doesn't exist, is there a practical application for it? To me it seems like it just wouldn't be combat effective.

(Duom can be found in the D&D 3.0 Arms & Equipment Guide)

From the description of it, it sounds very much like a Chinese halberd variant:


"The duom is a longspear with a standard spear- head, as well as two blades curved so that they point backward along the shaft."

http://www.wle.com/media/W324-T.jpg

http://users.telenet.be/Philip/dnd/Pics/duom.jpg


It could very well just be a ceremonial weapon of some sort.

hamishspence
2013-03-19, 01:54 PM
The problem is, pneumonic plague isn't very contagious - its risk of transmission is 0.25-0.55 (on average, how many people an infected person will pass it onto), compared to very infectious diseases like influenza (1.8-10), measles (10-15) or pertussis (16-18).

Even so, the worst epidemic of pneumonic plague (Manchuria 1911) had a fatality rate of 0.3% of those affected and that was in fairly optimal conditions for its spread (the people were travelling on train cars during winter).


Now, yes- but might it not have mutated between the Middle Ages and 1911?

Rhynn
2013-03-19, 02:35 PM
Just because it has a pneumonic component doesn't mean it's a pneumonic plague, as such. The bubonic, pneumonic, and and septicemic plague are all Y. pestis (http://en.wikipedia.org/wiki/Yersinia_pestis), and the Black Death was probably all of them, not a single disease across all locations. And yes, it may have been a much more virulent form than what we have now (nevermind the far worse hygiene and ability to even understand how diseases were transmitted - much of it was "bad air" and "wrath of God" back then).

Brother Oni
2013-03-19, 05:12 PM
Now, yes- but might it not have mutated between the Middle Ages and 1911?

Y. pestis doesn't mutate in the way I think you're suggesting as it's a bacterium, not a virus. All it tends to do is change its surface antigens to evade the host's immune response and thus while the coating is different, the core organism is essentially identical unlike a much simpler virus.


Just because it has a pneumonic component doesn't mean it's a pneumonic plague, as such.

True, probably got a bit of a terminology wire cross there.

While transmission of bubonic plague can happen via inhaled droplets (I assume that's what's meant by pneumonic component), this is classified as a secondary case of pneumonic plague as the primary disease state is the bubonic version caused by a flea bite.

As you've pointed out, all three variants of plague are caused by the same organism, the different disease states are just called different things as they have different symptoms and vectors of transmission.

Aux-Ash
2013-03-19, 05:22 PM
An interesting discussion. Looking over my materials on pathogens to freshen things up I noticed that one of the possible reservoirs (which in microbiological terms refers to a host which can be infected without suffering any symtoms or negative effects) of Y. pestis is domestic livestock. So you might want to look up how livestock was kept in the regions that were hit less hard.

I believe I've read many times that here in Sweden, particuarily in the countryside, livestock was frequently brought inside the homes during winter to keep the warmth. Which would bring a natural source of the bacteria right next to the people dying from them (and fleas acting as vectors, transmitting between livestock and people).

If it varies how close to their livestock people lived, that could help explain it. Though it will likely only be one out of many combined explanations (if they didn't try to treat plaguevictims with arsenic and mercury in these areas that'll certainly help too).

Octopusapult
2013-03-19, 08:15 PM
With all the talk of biological weapons, this thread is going to land us all on some FBI lists.

Galloglaich
2013-03-19, 08:20 PM
Just because it has a pneumonic component doesn't mean it's a pneumonic plague, as such. The bubonic, pneumonic, and and septicemic plague are all Y. pestis, and the Black Death was probably all of them, not a single disease across all locations. And yes, it may have been a much more virulent form than what we have now

My understanding is that in epidemiology, all kinds of pathogens increase in both virulence and mortality / lethality when conditions for transmission are very easy, this has been well documented with Cholera I believe.


(nevermind the far worse hygiene and ability to even understand how diseases were transmitted - much of it was "bad air" and "wrath of God" back then).

While it's true that medieval sanitation isn't as good as today with chlorinated water in everyone's house, sewerage treatment plants and garbage trucks and incinerators and so on, and Medieval medicine was certainly very crude compared to today... if you will forgive my saying this it is a persistent and myth that people in the Medieval period were particularly dirty. For some reason we tend to think of the conditions during the 19th Century, which was a time of overcrowding and mass poverty, and project backwards (at least until we get to the Romans and Greeks because we think of them as clean, they look clean on TV shows unlike Medieval people who usually look like filthy cavemen). We also of course project specifically based on English and to a lesser extent, French history (through the lens of Monty Python). But history doesn't work that way. It's not one steady march to smart phones and

In a nutshell, each house in a medieval town or village had a privy in the backyard, just like many did in the US in rural areas right up to the 1960's. Towns had water piped in from legally (and militarily) protected water sources, much as the Romans had so famously done with their aqueduct systems. Refuse was carted out of the town, messy businesses like slaughterhouses and tanners were forced to set up outside the town walls. Towns had sewers or stone gutters where waste water was drained out into a nearby waterway (virtually every town was built on a water way because that was the principle mode of transportation for anything heavy). Important to note! This normally was NOT the same waterway they got their drinking water from. I know the details on this for Danzig where there were two protected water sources used both for drinking water and for their water mills, and one brackish stream which was connected to the Baltic Sea which was both their sewer and principle navigation waterway. But from what I understand the pattern was the same in hundred of Medieval towns. Just like during Roman times.

It is really important to note, London and Paris were both unusual by Medieval standards in not having real public sanitation systems. They were also unusually large cities for their day - the principle reason towns in Central Europe remained small was precisely because of sanitation.

http://4.bp.blogspot.com/-I8b71fvoh_I/Tc1o78VawrI/AAAAAAAAAwY/S1gDHRd--B0/s1600/5-12-bock1597kunstfueralle.jpg
As for personal hygiene, for one thing, those Roman bath never went away. Check out a few of the period images here (https://www.google.com/search?q=medieval+bathing&oe=utf-8&aq=t&rls=org.mozilla:en-US:official&client=firefox-a&um=1&ie=UTF-8&hl=en&tbm=isch&source=og&sa=N&tab=wi&ei=FABJUd_yNKW62gXztYDQAg&biw=1366&bih=622&sei=FwBJUYSOMerk2QXpxYHgAQ)

Many towns had laws actually providing money for even poor people to bathe at least once a week.

Bath-houses are ubiquitous in the towns. For example, according to surviving documents in the 15th Century there are 12 public bath-houses in Kraków, which is a small city by the standards of the day (source 1). From the end
of the thirteenth century the city of Lübeck had a bathhouse in almost every street; in Ulm, at the close of the Middle Ages, there are eleven public bath-houses, in Nuremberg twelve, in Frankfort at least fifteen, and Vienna twenty-nine (source 2). The first record of a bath-house in Danzig was from the Teutonic Order in 1386 though archeology shows they existed for at least three centuries before that (source 3).

(Source 1) (Zamoyski, Adam. The Polish Way: A Thousand-Year History of the Poles and their Culture, New York: Hippocrene, (1987), page 58

(Source 2) Geschichte des deutschen Volkes seit dem Ausgang des Mittelalters, “History of the German People at the close of the Middle Ages”, Johannes Janssen, 1896 (translated by M.A. Mitchell and A.M. Christie in two volumes), page 34

(Source 3) Lübecker Kolloquium zur Stadtarchaologie im Hanseraum IV: Die
Infrastruktur Schmidt - Roemhild; (October 2004) ISBN: 3795012651,
The Archeology of Gdansk, Henryk Paner, page 279

Even in rural areas, people bathed routinely. In the Scandinavian, Baltic and Slavic zones it was commonplace to have a sauna or a banya (steam bath) in every farmstead.

Much of this gradually changed in the 16th and 17th Centuries mainly I think due to the religious fanaticism of the reformation and counter-reformation, and probably due to the onset of syphilis which made bath-houses and the kind of frolic which took place there seem dangerous and amoral. But that change took place much more in the West than in Central or Northern Europe where it never really went away, even in the Victorian era. In fact Americans and English people are still rather shocked by public bathing in Scandinavia today.

When I was stationed in Germany, they still had many public baths which date back to the Medieval era, or even to Roman times (and some more recently from the 19th Century), are still used and many still feature co-ed public bathing. Dozens of towns in Germany are named for their baths (baden).

http://jurgenfauth.com/wp-content/uploads/kaiserfriedrichtherme-01.jpg http://farm3.staticflickr.com/2395/2152434214_8f84011f65.jpg
I think one of the above images is Wiesbaden and the other is Baden baden.

Even "bad air" was also clearly not entirely a crazy thought considering there was apparently a 'pneumonic' component to the Bubonic plague. Not to mention the transmission of other airborne pathogens. But more relevant, they followed the doctrine that stinking things like garbage and sewerage were bad for you and could cause rot. Which ... isn't entirely scientific by today's standards but does kind of work. Like a lot of Medieval thinking.


An interesting discussion. Looking over my materials on pathogens to freshen things up I noticed that one of the possible reservoirs (which in microbiological terms refers to a host which can be infected without suffering any symtoms or negative effects) of Y. pestis is domestic livestock. So you might want to look up how livestock was kept in the regions that were hit less hard.

I believe I've read many times that here in Sweden, particuarily in the countryside, livestock was frequently brought inside the homes during winter to keep the warmth. Which would bring a natural source of the bacteria right next to the people dying from them (and fleas acting as vectors, transmitting between livestock and people).

If it varies how close to their livestock people lived, that could help explain it. Though it will likely only be one out of many combined explanations (if they didn't try to treat plaguevictims with arsenic and mercury in these areas that'll certainly help too).

Here, I think you may have hit on something really important. One type of the town regulations I was referring to above had to do with the management of animals. I wonder if this might have had something to do with it.

G

Galloglaich
2013-03-19, 08:32 PM
With all the talk of biological weapons, this thread is going to land us all on some FBI lists.

I was referring to the original Mongol introduction of the Black Death into Caffa during a siege in 1346, ala

http://wwwnc.cdc.gov/eid/article/8/9/01-0536_article.htm

G

Rhynn
2013-03-19, 09:25 PM
if you will forgive my saying this it is a persistent and myth that people in the Medieval period were particularly dirty.

:smallconfused: It's got nothing to do with the myths about people not washing, etc.

They had worse hygiene, plain and simple.


While it's true that medieval sanitation isn't as good as today with chlorinated water in everyone's house, sewerage treatment plants and garbage trucks and incinerators and so on

And that's just a start. Heck, Ignaz Semmelweis was derided for proposing that you shouldn't deliver babies without washing your hands first - in the 19th century.

(Obviously, 18th or 19th century urban hygiene probably lost out to average medieval rural hygiene, but I was only comparing to actual modern hygiene...)

I get fighting urban legends (or Victorian legends), but when they're not even brought up?

Galloglaich
2013-03-19, 10:27 PM
And that's just a start. Heck, Ignaz Semmelweis was derided for proposing that you shouldn't deliver babies without washing your hands first - in the 19th century.

What did I say about projecting backward from the 19th Century? These things don't come out of thin air. To be a medieval physician (i.e. to receive a doctorate or masters degree in medicine), you had to study three auctore, 'authorities': Hyppocrates (http://en.wikipedia.org/wiki/Hippocrates), Galen (http://en.wikipedia.org/wiki/Galen), and Avicenna (http://en.wikipedia.org/wiki/Avicenna). All three of them specifically recommended washing your hands (with boiled water or wine) before any medical procedure and also boiling bandages and surgical instruments before use.

http://cid.oxfordjournals.org/content/47/5/730.2.full

I'm not in any way saying that Medieval medicine was sophisticated or usually effective, or wasn't based on crazy theories about the humours and bloodletting and the effects of astrology on health... but they did know something about antiseptics before they came back into vogue in the 19th Century.

The origin of alcohol, al-kohl, the arabic term for distilled spirits, was first attributed to wine by a famous alchemist named Paracelsus (http://books.google.com/books?id=3f-kPJ17_TYC&pg=PA229&lpg=PA229&dq=wine+antiseptic+galen&source=bl&ots=KlFmKw6RJ4&sig=zqjy1oH97SNbiHgJsVCAZifaSiw&hl=en&sa=X&ei=oiNJUdyyOoa32wWP8IDgDw&ved=0CDQQ6AEwAQ#v=onepage&q=wine%20antiseptic%20galen&f=false) in the 16th Century to describe the (by then well known) antiseptic qualities of wine.

The 12th Century Arab knight Usamah Ibn Munqidh described how a Frank (i.e. European) taught him how to treat wounds by using 'sharp' vinegar.

http://en.wikipedia.org/wiki/Medieval_medicine_of_Western_Europe#Medicine

They also incidentally did have anesthesia (mainly based on opium, just like today)

Mind you, I'm definitely NOT saying medieval doctors were good, I'm saying basic hygiene was actually pretty good in the Medieval period, contrary to the common myth. It got a lot worse in the Early Modern period.

There are a lot of similar myths, like witch burnings ... of which there were very few in the medieval period, they almost all happened in the 16th and 17th Century. Or the idea that the church was opposed to science (or that it had any coherent, unified policy at all)



(Obviously, 18th or 19th century urban hygiene probably lost out to average medieval rural hygiene, but I was only comparing to actual modern hygiene...)

I'm comparing 18th or 19th Century urban hygiene to medieval urban hygiene. Believe it or not, in many places a big chunk of the population lived in cities. In the 14th Century both Flanders and Lombardy were about 40% urban. More than North America in the early 19th Century.


I get fighting urban legends (or Victorian legends), but when they're not even brought up?

The world is a lot weirder place than the Victorian and modern TV placeholders would have you believe. What is common knowledge and popular opinion on these things is basically a cartoon. Not even a good cartoon...

G

Brother Oni
2013-03-20, 03:21 AM
My understanding is that in epidemiology, all kinds of pathogens increase in both virulence and mortality / lethality when conditions for transmission are very easy, this has been well documented with Cholera I believe.

You're correct, but it should be remembered that epidemiology tends to view disease effects at the population level rather than the individual.

The increase in mortality with conditions optimal for transmission isn't because the disease has become more deadly, it's because it's infected and killed more of the vulnerable population (young, elderly, people who are immunocompromised or have otherwise weakened immune systems, etc).

hamishspence
2013-03-20, 03:29 AM
Y. pestis doesn't mutate in the way I think you're suggesting as it's a bacterium, not a virus.

True- but all life is capable of mutating. And bacteria, with a short generation, will have more opportunity for mutation in the same timeframe.

Brother Oni
2013-03-20, 06:15 AM
True- but all life is capable of mutating. And bacteria, with a short generation, will have more opportunity for mutation in the same timeframe.

On a general level this is true, but it's less applicable to Y. pestis as it requires a host to propogate itself and thus can't undergo antigenic drift or shift as easily as a free living organism.

There's an epidemiological hypothesis of 'old' and 'young' diseases - the overall goal of any living organism is to ensure its survival, thus for a disease to kill its host quickly before it has had time to spread, makes relatively little sense.

Pathogenic organisms that have had more time to adapt to their host (old diseases) have managed to mitigate their symptoms and tend not to kill off their hosts quickly and generally just lurk endemically to the population (eg. the common cold, tuberculosis).
More lethal diseases haven't cottoned on to this trick yet (or the host hasn't figured out a way to defend itself effectively) and just kill their host in their mad dash to propogate themselves (eg. ebola, rabies)

Humans are a bit of a special case as evolutionarily speaking, we're still very new so a large number of diseases haven't had sufficient time to adapt to us; rabies has been known since ~2000BC, but it's still >99% fatal untreated (and very hard to treat even before the patient becomes symptomatic).

Galloglaich
2013-03-20, 10:28 AM
This book I read

http://www.amazon.com/Parasite-Rex-New-Epilogue-Dangerous/dp/074320011X

...seemed to have a different take, claiming that cholera and other diseases actually varied in their virulence dramatically (for the same kind of people) from places where transmission was difficult, like Arizona where there is chlorinated water and so on, to places where it was easy like Haiti or Nepal.

But it sounds like you work in the field so I'll defer to your experience.

G

Spiryt
2013-03-20, 11:28 AM
:smallconfused: It's got nothing to do with the myths about people not washing, etc.

They had worse hygiene, plain and simple.



That's generally true, obviously, although we have to remember that from quite some time, we observe too 'much' hygiene is so called first world, which is also not good thing at all.

Too much sterile conditions, water, cosmetics, etc. also does human body no good.

http://www.sciencedaily.com/releases/2012/10/121003082734.htm


Anyway, I found a mention, that author of this book

http://books.google.pl/books/about/A_Distant_Mirror.html?id=BmRoOIwLWhsC&redir_esc=y


apparently claims that few chronicles from 14th century praise Kazimierz the Great for establishing effective controls of travelers of western border of Poland.

That could be some clue, if somebody has this book, then can check what kind of sources author found. I have it in my local library, so I may be able to take a look soon.

Brother Oni
2013-03-20, 01:24 PM
But it sounds like you work in the field so I'll defer to your experience.


I don't actually work in epidemiology, I just have a healthy (or unhealthy, depending on your point of view) interest in diseases and remember most of what I was taught from university. :smalltongue:

Without being able to read the book in more detail, I can't say whether it agrees or denies with the hypothesis, but the brief bit on the 'laws of virulence' where it's nasty enough to spread but not too nasty to wipe out all its hosts seems to fit.
Since the book appears to have been written after I left university, I would say that the hypothesis has been updated.

Galloglaich
2013-03-20, 03:13 PM
Essentially one of the arguments of the book is that pathogens as well as parasites fluctuate in their virulence depending on the conditions ... when it's easy to transmit, they get more lethal.

I don't have any training in the field (beyond what I learned as a medic in the Army) so I don't really have a good way to judge the academic bona fide's of the book, though it was an excellent read.

G

Aux-Ash
2013-03-20, 05:50 PM
This book I read

...seemed to have a different take, claiming that cholera and other diseases actually varied in their virulence dramatically (for the same kind of people) from places where transmission was difficult, like Arizona where there is chlorinated water and so on, to places where it was easy like Haiti or Nepal.


It's a bit more tricky than that. Most bacteria cause symtoms primarily by depletion of resources or by poisons produced, either as a result of production or defence. V. cholerae secrete a poison when distressed and this is what causes the disease. In a more hostile enviroment, such as chlorinated water, it will be more likely to go on the defensive than not. Thus the dosage is generally much higher and it is spurred into a higher reproduction to survive. Leading to a more active and more aggressive strain spreading.
The bacteria itself only cares about us in the sense that our intestines are an hostile enviroment. It does not actually intend to end up there (it does however intend to get out, hence the toxin). It just tries to survive.

Y. pestis is different in that unlike V. cholera it actually "wants" to get into our lymphnodes. Like M. tuberculosis the disease makes use of our immunesystem to grow. The symtoms are a byproduct of it killing the phagocytic cells as it grows (which contains quite a few strong poisons). It does not cause the disease by defending itself but by reproducing. As such it is neither less nor more infectious.


Essentially one of the arguments of the book is that pathogens as well as parasites fluctuate in their virulence depending on the conditions ... when it's easy to transmit, they get more lethal

The individual strains? No.
But the more people infected the faster it spreads since people carry the infection to other people. So with an easy transmission, it will be more lethal if you look at it's effect on a population.

My books suggest though that untreated plague is just about the worst thing there is. It has sky high lethality (75% for bubonic, 85 for septic and 90 for pulmonary), the lowest infection dose there is (a single cell is enough to cause an outbreak of the disease, compare to 1 000 000 for Salmonella) and a really short incubation time (3-5 days for bubonic and 1-2 days for pulmonary, which is to say that at it's longest comparable to a cold).
With such a short incubation time you could basically get infected and go through the disease twice before the body has managed to develop an immunity to it. With such a low infection dosage it means that even in suboptimal conditions it can still spread easily.

Resistant Y. pestis is the nr 1 on WHO's nightmare list, and for a very good reason.

Yora
2013-03-20, 07:25 PM
Essentially one of the arguments of the book is that pathogens as well as parasites fluctuate in their virulence depending on the conditions ... when it's easy to transmit, they get more lethal.
It's the other way round. When transmission is difficult, the pathogen can not afford losing its host too quickly. The host has to stay alive (and preferably in a condition where he mingles with others) to increase the chance of spreading to other individuals. If a host dies, so does the pathogen, and if it has not been transmited by that time, the species goes extinct.

I like the concept of the egoistic gene (from a book by Richard Dawkins), which explains that evolution does not care at all for the wellbeing of the individuum or the survival of it's offspring. It is not "survival of the fittest" that drives evolution, but simply what genes happen to continue to be replicated and which genes die out together with their carriers.
Evolution has created a lot of organisms that do things that do not help them at all or even harm them, but that doesn't matter as long as the gene responsible for that trait is continuusly replicated into the organisms offspring before it dies. (Like make insects that have no digestive system and just starve shortly after hatching, but some manage to mate before that.)

If a pathogen causes its host to do something that leads to it's own extinction, it goes extinct. As long as it gets spread around between hosts, it does not "care" in any way what it does to it.

f you will forgive my saying this it is a persistent and myth that people in the Medieval period were particularly dirty.
Also, being fully submerged in water is not synonymous with cleaning. I would think most of us have less than a bath a year. Because we shower instead. I don't think I bathed in over 5 years.

Galloglaich
2013-03-20, 10:53 PM
Also, being fully submerged in water is not synonymous with cleaning. I would think most of us have less than a bath a year. Because we shower instead. I don't think I bathed in over 5 years.

I don't really see your point. They cleaned themselves in their baths and used soap. Soap was one of the hottest export products from several cities, notably Venice which had a big industry for it, but it goes back to the Hallstadt culture. Galen described a recipe for soap in the second century AD and said the best soap came from he Germanic tribes, the second best from Gaul (source: Partington, James Riddick; Bert S Hall (1999). A History of Greek Fire and Gun Powder. JHU Press. p. 307.)

Soap making was a major state industry in Venice by 1302

http://books.google.com/books?id=vvkj1wC8aTcC&pg=PA13&lpg=PA13&dq=soap+industry+venice&source=bl&ots=zQL_0_hSjf&sig=PLP0jNXZDTl8sq3uIH_gHITGphI&hl=en&sa=X&ei=zYNKUavBJIvOqAHThYDwCg&ved=0CGgQ6AEwCQ#v=onepage&q=soap%20industry%20venice&f=false

They also combed their hair, trimmed their nose hair, cleaned their teeth, removed wax from their ears, and typically washed their hands before every meal (which is something many people don't do today in the "Industrialized West" even though they should). Most of this 'everyone is filthy / some lovely muck over here Dennis' medieval caveman stuff is 100% b.s.

G

Galloglaich
2013-03-20, 11:05 PM
The individual strains? No.
But the more people infected the faster it spreads since people carry the infection to other people. So with an easy transmission, it will be more lethal if you look at it's effect on a population.

Maybe this is what the book said and I misunderstood it. If I get the time I'll look up the section and transcribe what it says if it seems to be different.


Resistant Y. pestis is the nr 1 on WHO's nightmare list, and for a very good reason.

My buddy who works for the CDC says antibiotics are going to be done for in 20 years, I hope he's wrong.

G

Kalmageddon
2013-03-21, 07:35 AM
Since there are so many competent people giving advice in this thread I'd like to request assistence for my current campaign by providing a link (http://www.giantitp.com/forums/showthread.php?t=276986) to the thread I opened in another section, since this thread is not for giving game mechanic advices.

Thanks in advance to anyone willing to help!

Brother Oni
2013-03-21, 08:09 AM
Resistant Y. pestis is the nr 1 on WHO's nightmare list, and for a very good reason.

I'm not dismissing the WHO's concerns out of hand, but they're more focused on diseases in the wild.
To be honest, there are a large number of other diseases that I would be concerned with more, primarily the ones that can be weaponised - while this does include Y. pestis, the only really viable strain that can weaponised is the pneumonic version, which has a comparatively low rate of transmission that no amount of tinkering can really remedy without it being more worthwhile to use a different organism.

I can't really think of any regime capable and crazy enough to try and use the far more effective bubonic plague by launching infected fleas via missiles and hoping.


It's the other way round. When transmission is difficult, the pathogen can not afford losing its host too quickly. The host has to stay alive (and preferably in a condition where he mingles with others) to increase the chance of spreading to other individuals. If a host dies, so does the pathogen, and if it has not been transmited by that time, the species goes extinct.

I think that you're assigning too much intent by the organism to the process.

A unicellular organism isn't going to be aware of the big picture - all it knows is conditions are currently good for it to reproduce (which is usually inside the host).
As Aux-Ash and I've pointed out, increased mortality rates of diseases are measured on a population scale, not individual.


My buddy who works for the CDC says antibiotics are going to be done for in 20 years, I hope he's wrong.

Two major caveats to that - it's estimated that it's going to be 20 years before diseases that are completely resistant to the current antibiotic of last resort (vanomycin) start turning up.
The other is that current antibiotics (which are all derived from penicillin) all essentially work in the same way (I'll spare you the molecular biology, but if you're interested: beta-lactam ring (http://en.wikipedia.org/wiki/Beta-lactam_antibiotic)).

We're developing new classes of antibiotics (linezolid (http://en.wikipedia.org/wiki/Linezolid), among others, is starting to replace vancomycin as the antibiotic of last resort) which function in a fundamentally different way and science isn't just sitting there twiddling their thumbs while micro-organisms figure out a way to become resistant:link (http://phys.org/news190207645.html).

So while your buddy is right to be concerned, it's not all doom and gloom: we're working on it.

Edit: Is this the point where I shout 'FOR SCIENCE!'? :smalltongue:

paddyfool
2013-03-21, 09:28 AM
On virulence: one reason that it's altered by ease of disease spread is that ease of spread may also affect the dose. For instance, where it's difficult to get cholera (e.g. if drinking water supplies are kept clean, but it's still possible to get infected via a contaminated drinking utensil etc.), if you do catch it you'll likely be infected by only a small population of Vibrio cholerae bacteria, giving your immune system time to adjust and fight back. But if cholera is all over the place, you're more at risk of getting infected by a big load of cholera at once, and that's rather harder to deal with.

On antibiotic resistance: it depends very much on which bugs you're dealing with. For some (e.g. Syphilis, Streptococcus etc.), the oldest drugs we've got still work. For others (e.g. Gonorrhea, Staphylococcus etc.), it seems like they evolve resistance to new drug classes practically every year. So antibiotics are likely to remain very useful against some bugs, but may be of diminishing use against others without finding new classes.


Two major caveats to that - it's estimated that it's going to be 20 years before diseases that are completely resistant to the current antibiotic of last resort (vanomycin) start turning up.

Actually, high-level resistance to Vancomycin has been around since at least 2002. (http://en.wikipedia.org/wiki/Vancomycin-resistant_Staphylococcus_aureus) It's not widespread yet, but given the way antibiotics are used around the world today, it's going to be.


The other is that current antibiotics (which are all derived from penicillin) all essentially work in the same way (I'll spare you the molecular biology, but if you're interested: beta-lactam ring).

Not true. Three major classes of current antibiotics do target the beta-lactam ring (penicillins, cephalosporins and carbapenems), but others target a wide variety of other targets - the bacterial 30s and 50s ribosomal subunits; the bacterial DNA gyrase; the folate synthesis pathway; and various others. See the mechanism of action column in this table, for instance. (http://en.wikipedia.org/wiki/Antibiotic_classes)

eulmanis12
2013-03-21, 09:30 AM
I'm not dismissing the WHO's concerns out of hand, but they're more focused on diseases in the wild.
To be honest, there are a large number of other diseases that I would be concerned with more, primarily the ones that can be weaponised - while this does include Y. pestis, the only really viable strain that can weaponised is the pneumonic version, which has a comparatively low rate of transmission that no amount of tinkering can really remedy without it being more worthwhile to use a different organism.

I can't really think of any regime capable and crazy enough to try and use the far more effective bubonic plague by launching infected fleas via missiles and hoping.



I think that you're assigning too much intent by the organism to the process.

A unicellular organism isn't going to be aware of the big picture - all it knows is conditions are currently good for it to reproduce (which is usually inside the host).
As Aux-Ash and I've pointed out, increased mortality rates of diseases are measured on a population scale, not individual.



Two major caveats to that - it's estimated that it's going to be 20 years before diseases that are completely resistant to the current antibiotic of last resort (vanomycin) start turning up.
The other is that current antibiotics (which are all derived from penicillin) all essentially work in the same way (I'll spare you the molecular biology, but if you're interested: beta-lactam ring (http://en.wikipedia.org/wiki/Beta-lactam_antibiotic)).

We're developing new classes of antibiotics (linezolid (http://en.wikipedia.org/wiki/Linezolid), among others, is starting to replace vancomycin as the antibiotic of last resort) which function in a fundamentally different way and science isn't just sitting there twiddling their thumbs while micro-organisms figure out a way to become resistant:link (http://phys.org/news190207645.html).

So while your buddy is right to be concerned, it's not all doom and gloom: we're working on it.

Edit: Is this the point where I shout 'FOR SCIENCE!'? :smalltongue:

no, its the part where you shout ,

FOOOOOOOORRRRRR!!!!!!!!!! SCIIIEEEENNNNNNNNNCEEEEEE!!!!!!!!!!!!:smallbiggrin :

Brother Oni
2013-03-21, 11:05 AM
Hmm, seems I need to brush up on my epidemiology and anti-biotic knowledge. While I have some excuse for VRSA (they had only just synthesised it in the lab when I was at uni), I have no such reason for forgetting about all the various antibiotic classes. :smallredface:

Yora
2013-03-21, 12:11 PM
I'm not dismissing the WHO's concerns out of hand, but they're more focused on diseases in the wild.
To be honest, there are a large number of other diseases that I would be concerned with more, primarily the ones that can be weaponised - while this does include Y. pestis, the only really viable strain that can weaponised is the pneumonic version, which has a comparatively low rate of transmission that no amount of tinkering can really remedy without it being more worthwhile to use a different organism.
That was my point. An organisms evolves in completely random ways with no plan or destination at all. If a virus evolves to a form where the host dies before there is a good chance to spread it, it goes extinct.

Though I admit that "mortality among the infected" and "mortality among the population" are two very different things.
A super-lethal virus generally makes people a lot more alert and cautious, increasing the chance that it is contained quickly. Like you have with Ebola where you have mortality rates for infected that frequently go above 80%, but outbreaks are usually limited to a few dozen or at most hundred infections.
A virus that kills only 1 in 1000 infected will face much less obstacles. Like flu, which leads to millions of infections every year and therefore results in hundreds of thousands of death, even though the risk for infected is only in the 0.1% range.

That a disease with high chance of death on infection also spready through a very large population is rare. Plague outbreaks being among them.

hamishspence
2013-03-21, 02:45 PM
Flu pandemics I think also combined high (or at least moderate) lethality with high spread rate.

Galloglaich
2013-03-21, 03:17 PM
Flu pandemics I think also combined high (or at least moderate) lethality with high spread rate.

Cholera can be up to 50-60% mortality untreated. Or if the antibiotics don't work...

G

Wardog
2013-03-22, 12:07 PM
I can't really think of any regime capable and crazy enough to try and use the far more effective bubonic plague by launching infected fleas via missiles and hoping.
Is this the point where I shout "They all said I was mad! But I'll show them!! I'll destroy them all!!! BWAHAHAHAHA!!!!! :smalltongue:[/QUOTE]

Yora
2013-03-23, 04:26 AM
In Dungeons & Dragons, there are bows that deal additional damage if the user has a high strength, but have a penalty to aim if the users strength is not high enough.
Is that reasonably realistic?

Spiryt
2013-03-23, 07:49 AM
The main problem here is that drawing heavy bow is also - or even mainly - matter of skill, familiarity and conditioning, so training in general.

After all, any reasonably strong man can pull and hold ~120 pounds. Doing it in bow stance while aiming and so on is very different matter.

But seeing that such overall 'strength level' is very simple abstraction in the first place, I would say that in D&D bow system is just alright.

Galloglaich
2013-03-26, 05:19 PM
I have a question.

A buddy was asking me the other day about 'Continental' longbow use. I know the Burgundians adapted the English style longbow... or maybe they had their own which they further militarized according to the 15th Century ordinances. I think the French used them as well on a small scale though they never did get well established with them (unlike the Burgundians)

I've also seen evidence of their fairly widespread use in Italy and in Central Europe. And I know the longbow (of some kind) was used in Scandinavia back to the neolithic era. I am even aware that there is a debate as to whether the longbow was a special weapon of the British Isles or not.

But the longbow is not my area of expertise and I know very little about the 100 years war which I think is probably the main focus of the documented use of the longbow in Continental Europe by the English or anyone else. Haven't been able to land a copy of The Great Warbow yet.

Could anyone here provide me with some good sources for the use of longbows on the Continent?

G

Spiryt
2013-03-26, 05:30 PM
http://www.arcus-lucznictwo.pl/index.php?id=49

In polish only, sadly, but has a lot of pics. :smallwink:

Generally Thorsberg and Nydam finds are pretty damn priceless, not that much about them in the Internet though.



Anyway, bow was naturally prevalent tool of hunt, and war as well during Dark Ages, so there obviously had to be a lot of them, even if few had prevailed, sadly.


I am even aware that there is a debate as to whether the longbow was a special weapon of the British Isles or not.

It depends how we define 'longbow', I guess, if as 'any long self bow' then it absolutely is not special to British Isles in any possible way.

Galloglaich
2013-03-26, 07:37 PM
It depends how we define 'longbow', I guess, if as 'any long self bow' then it absolutely is not special to British Isles in any possible way.

Right, and the definition of what is meant by a 'longbow' or an 'English Warbow' gets slippery and is still endlessly debated. I gather it changed over time. But you could say very broadly that there is a thesis that

1) There were 'ordinary' bows which as we can see today from hunters, can range from the 40-70 lbs draw range, and be of various lengths / sizes, which are sufficient to kill big game like deer and can also be shot in lofted shots at a pretty good distance (at least a couple of hundred yards for the best of them) which were therefore (arguably) suitible for military use.

2) There were simultaneously some rarer, stronger 'longbows' made out of special wood like heart of yew which had a heavier draw in the 80-120 lbs draw range, which were considered 'special' compared to other bows, and had a greater range and velocity than 'ordinary' bows. This may include the Nydam bows and other weapons from the Bronze Age back to the Neolithic which were found both in the British Isles and Scandinavia, among other places, and also includes the Welsh bows which made an impression on the English during their 12th -13th Century anexation of Wales.

3) These weapons and / or native English weapons became an increasingly important part of the English army during the 14th, 15th, and 16th Century and played a decisive role in some battles in France such as at Crecy, Poitiers, and most famously, Agincourt. By then both the culture of archery and the weapon itself was further consciously developed by the English into a longbow which some people call the 'English Warbow', based on finds from the 16th Century shipwreck of the Mary Rose, may have been up to 120-160 lbs draw, very formidable weapons shooting heavy arrows to a long range with substantial power. In it's own way this weapon was a (maybe slightly inferior in certain respects) rival to the famous composite bow of the Central Asian steppe and Anatolia.

In stage 3, and also possibly going back to stage 2, the Burgundians were using some kind of variant of these weapons, possibly some French, Italians and others were also using them... and this is what I'm hoping someone here can help me find sources for.

G

Matthew
2013-03-26, 08:43 PM
Galloglaich, have you read the 2011 article "The development of the longbow in late medieval England and 'technological determinism'" by Clifford J. Rogers? It is a good read; shoot me an email if not.

fusilier
2013-03-26, 09:14 PM
In stage 3, and also possibly going back to stage 2, the Burgundians were using some kind of variant of these weapons, possibly some French, Italians and others were also using them... and this is what I'm hoping someone here can help me find sources for.

G

English longbowmen were employed as mercenaries during the 14th century in Italy, and into the mid 15th century . . . by which time they may have been "naturalized" Italians, but they were very few in number. I'm not aware that the use of long bows in Italy ever resulted in attempts to emulate it among the native populations. "Saracen" style re-curve(?) bows, on the other hand, had influence in Southern Italy and Sicily, and the Venetians were impressed enough that they trained local militia in it for a while in the late 15th and early 16th centuries. Although, I think that was primarily a feudal thing in the overseas territories. Intended primarily for shipboard use, they did find their way into battles on the terrafirma during that period. Saracen influences on southern Italy are often reported, but rarely described in detail, prior to the hiring of Turkish troops after the siege of Otranto. Venice's use of them is well known, but stradiots were also used pretty heavily by the Kingdom of Naples beginning in the late 15th century.

Despite the prevalence of crossbows, bows may have been popular for hunting among the nobility, but not much is seen in the way of long bows.

On this webpage you can see some late 15th century images of Italian bowmen (towards, the bottom although they are from religious scenes):
http://www.greatestbattles.iblogger.org/Italy/Italy-MidToLate15thCentury.htm

Spiryt
2013-03-27, 04:06 AM
2) There were simultaneously some rarer, stronger 'longbows' made out of special wood like heart of yew which had a heavier draw in the 80-120 lbs draw range, which were considered 'special' compared to other bows, and had a greater range and velocity than 'ordinary' bows. This may include the Nydam bows and other weapons from the Bronze Age back to the Neolithic which were found both in the British Isles and Scandinavia, among other places, and also includes the Welsh bows which made an impression on the English during their 12th -13th Century anexation of Wales.



Most hedeby bows are yew, so are Thorsberg and nydam ones, most pieces of bows found in Poland are yew as well.

Pretty much everybody was making bows out of yew if he could only find decent one it seems, because it's very good and 'easy' wood for bow - in the sense that due to natural composite structure etc. 'D' cross-sectioned bow is very foolproof - even if someone doesn't know what he's doing that well, it's hard to get completely useless bow.


Elm, another popular bow material, has some problems with resisting compression, for example, that's why elm bow bellies need some more careful designs.

So there's nothing really that special about yew wood, in fact it had gone endangered in most of Europe because of it's mass use.

It's really hard to tell anything for sure these days, but I generally hold opinion that any 'impression' made on some people was generally matter of bow culture, so strong skilled archers, capable of using very heavy bows accurately.

Although it's also entirely possible that there was some strong bowyer culture as well - so art of sculpting particularly dynamic bows out of available wood.

Then, theoretically, bows of the Welsh could have been really good, despite Wales being low and humid - which conditions don't produce good bow wood.

Straybow
2013-03-27, 07:21 AM
no, its the part where you shout

Please don't, you're breaking the frame :(

Deepbluediver
2013-03-27, 12:49 PM
QUESTION(s)
sorry, wanted to make sure this got seen


What benefit does adding studs to armor confer? In D&D, studded leather armor is listed as a whole seperate type, with better AC, lower max Dex, and a higher armor check penalty.

What would the real world implications of studded leather armor (D&D style) be, besides making it heavier?


Also, Hide armor is listed seperately as well, as medium instead of light armor. If I managed to make armor out of some animal with thicker skin (rhino or elephant, maybe...or dragon I guess) or stacked multiple layers of regular leather together, would it be more effective? Would it even be possible?


Bonus question- in D&D/gaming terms, what would you classify Far-east asian (http://en.wikipedia.org/wiki/Japanese_armour) style armor, made from wood and leather, as?


Additional Explanation
Core D&D lists 12 different varieties of armor. See them here (http://www.d20srd.org/srd/equipment/armor.htm).
I'm aware that in real-life, armor comes in near-infinite number of styles and combinations, but I'm brainstorming ways to simplify and pare down this list somewhat, and I'd like to know what sort of things I can dispose of, replace, or combine and not leave out major real-world categories.


Thanks for your help!

warty goblin
2013-03-27, 01:04 PM
QUESTION(s)
sorry, wanted to make sure this got seen


What benefit does adding studs to armor confer? In D&D, studded leather armor is listed as a whole seperate type, with better AC, lower max Dex, and a higher armor check penalty.

Studded leather armor really wasn't a thing anybody wore. I'm not even clear whether people used leather armor all that much. I've seen claims that hoplite panalopy was heavy leather, but also that it was multiple layers of linen glued together. So far as I can tell, neither side seems to have much in the way of physical or period textual evidence.



What would the real world implications of studded leather armor (D&D style) be, besides making it heavier?

You could get cute little rivet shaped punctures when somebody thwonked you with a hammer?


Also, Hide armor is listed seperately as well, as medium instead of light armor. If I managed to make armor out of some animal with thicker skin (rhino or elephant, maybe...or dragon I guess) or stacked multiple layers of regular leather together, would it be more effective? Would it even be possible?
I suppose it could be referencing untanned animal hide, which is distinct from leather, which has been tanned. Rawhide because it's uncured, has a tendency to rot when it gets wet, but can be pretty strong and hard. If you wet your rawhide and form it up, it dries in shape and can be used to produce extremely tight bindings. I've seen thick rawhide referenced as the material for cuir boilli, but again am not 100% certain of that.

Spiryt
2013-03-27, 01:13 PM
What benefit does adding studs to armor confer? In D&D, studded leather armor is listed as a whole seperate type, with better AC, lower max Dex, and a higher armor check penalty.

What would the real world implications of studded leather armor (D&D style) be, besides making it heavier?

Adding a lot of studs would make everything stiffer, heavier and probably generally harder to get trough, but to have any effect, studs would have to be very close to each other, and any sort of 'piercing' threat would still get
trough.

That's why such armor doesn't really have sense, and there's little evidence of any people wasting more or less precious metal for such things.




Also, Hide armor is listed seperately as well, as medium instead of light armor. If I managed to make armor out of some animal with thicker skin (rhino or elephant, maybe...or dragon I guess) or stacked multiple layers of regular leather together, would it be more effective? Would it even be possible?


Armor made of multiple layers of leather is definitely possible, that's the whole point of buff coat, for example.

http://www.freha.pl/index.php?act=attach&type=post&id=17635

It would be more effective, obviously. Single layer of even thick buffalo leather isn't really much of protection.


Bonus question- in D&D/gaming terms, what would you classify Far-east asian (http://en.wikipedia.org/wiki/Japanese_armour) style armor, made from wood and leather, as?

Japanese armor was Lamellar, and usually made from iron plates, like most lamellars, actually.

It would be any medium/heavy armor, depending on degree of coverage, I guess, in 3.5 terms.


Additional Explanation
Core D&D lists 12 different varieties of armor. See them here (http://www.d20srd.org/srd/equipment/armor.htm).
I'm aware that in real-life, armor comes in near-infinite number of styles and combinations, but I'm brainstorming ways to simplify and pare down this list somewhat, and I'd like to know what sort of things I can dispose of, replace, or combine and not leave out major real-world categories.

As far as 'D&D terms' go, I'm really not sure, they're a mess and not really realistic in any way, so for purposes of real armors it would be best to ditch the system altogether.

Galloglaich
2013-03-27, 07:13 PM
Most hedeby bows are yew, so are Thorsberg and nydam ones, most pieces of bows found in Poland are yew as well.

Pretty much everybody was making bows out of yew if he could only find decent one it seems, because it's very good and 'easy' wood for bow - in the sense that due to natural composite structure etc. 'D' cross-sectioned bow is very foolproof - even if someone doesn't know what he's doing that well, it's hard to get completely useless bow.


Elm, another popular bow material, has some problems with resisting compression, for example, that's why elm bow bellies need some more careful designs.

So there's nothing really that special about yew wood, in fact it had gone endangered in most of Europe because of it's mass use.

It's really hard to tell anything for sure these days, but I generally hold opinion that any 'impression' made on some people was generally matter of bow culture, so strong skilled archers, capable of using very heavy bows accurately.

Although it's also entirely possible that there was some strong bowyer culture as well - so art of sculpting particularly dynamic bows out of available wood.

Then, theoretically, bows of the Welsh could have been really good, despite Wales being low and humid - which conditions don't produce good bow wood.

Very well put. In the case of England it was clearly the culture of the archer which was key, since shooting these bows required some buildup of strength and a lot of practice for effective battlefield use. Though that was also true of other weapons (like the battlefield grade crossbow, contrary to what you hear on just about every History Channel or BBC show on Agincourt)

My friend in Bruges gave me some good leads on the Burgundians but I fear I may have to buy a book, my sagging bookshelf will suffer further. If I do find useful tidbits I'll post them here as a followup.

G

AgentPaper
2013-03-27, 08:54 PM
This may be stretching the bounds of "real world" a bit, but bear with me.

Say that there was a race of small humanoids, roughly 3-4 feet in height on average, with a slim build. In DnD terms, I think this roughly corresponds to a gnome. However, unlike gnomes, they also have wings, allowing them to fly. Their wing strength allows them to fly carrying roughly half their body weight in equipment for a period of roughly ten minutes.

Basically, my question is, what effects would this slim build have on combat tactics for the race, assuming it normally fought human-sized opponents, and a roughly medieval technology level (ie: no guns). Additionally, we're talking about massed combat here, at least 1000 people fighting, and usually much more.

My assumption would be that they would generally prefer to use bows and pikes. Bows because they allow them to stay away from enemies and pepper them with arrows, and pikes because a massed pike formation relies (relatively) less on body strength compared to, say, a sword and shield. Is this true? Or would both tactics be just as ineffective as any other tactic? Or is there some other tactic that might work better?

fusilier
2013-03-27, 09:20 PM
Adding a lot of studs would make everything stiffer, heavier and probably generally harder to get trough, but to have any effect, studs would have to be very close to each other, and any sort of 'piercing' threat would still get trough.

I tend to assume that the studs are the rivets on a coat of plates. :-)

Matthew
2013-03-27, 09:38 PM
Studded leather armor really wasn't a thing anybody wore. I'm not even clear whether people used leather armor all that much. I've seen claims that hoplite panalopy was heavy leather, but also that it was multiple layers of linen glued together. So far as I can tell, neither side seems to have much in the way of physical or period textual evidence.

I thought it was pretty cut and dried (pun intended) that linothorax was linen, and not leather. The early hoplite panoply was obviously bronze, but certainly before that leather was used for shields and even helmets, as attested in the Illiiad.

Raum
2013-03-27, 11:00 PM
What benefit does adding studs to armor confer? It looks 'pretty'. We still do it today (http://www.youtube.com/watch?v=amicrtFYgVQ), if not to armor. :smallwink: Though, as others have mentioned, I'm not sure there's any evidence of historical studded armors.

Galloglaich
2013-03-28, 12:21 AM
This may be stretching the bounds of "real world" a bit, but bear with me.

Say that there was a race of small humanoids, roughly 3-4 feet in height on average, with a slim build. In DnD terms, I think this roughly corresponds to a gnome. However, unlike gnomes, they also have wings, allowing them to fly. Their wing strength allows them to fly carrying roughly half their body weight in equipment for a period of roughly ten minutes.

Basically, my question is, what effects would this slim build have on combat tactics for the race, assuming it normally fought human-sized opponents, and a roughly medieval technology level (ie: no guns). Additionally, we're talking about massed combat here, at least 1000 people fighting, and usually much more.

My assumption would be that they would generally prefer to use bows and pikes. Bows because they allow them to stay away from enemies and pepper them with arrows, and pikes because a massed pike formation relies (relatively) less on body strength compared to, say, a sword and shield. Is this true? Or would both tactics be just as ineffective as any other tactic? Or is there some other tactic that might work better?

I would say javelins. Not well represented in RPG's or genre literature, but very effective and WIDELY used in real life. Why bother with mechanical power (and encumberance) of a bow when you can use the momentum of swooping down to add to the power of your javelin throw. No reason to stand and fight with pikes either if you can just inflict damage with javelins and darts, though a light lance might be useful in the manner of light cavalry.

G

AgentPaper
2013-03-28, 01:12 AM
I would say javelins. Not well represented in RPG's or genre literature, but very effective and WIDELY used in real life. Why bother with mechanical power (and encumberance) of a bow when you can use the momentum of swooping down to add to the power of your javelin throw. No reason to stand and fight with pikes either if you can just inflict damage with javelins and darts, though a light lance might be useful in the manner of light cavalry.

G

Interesting. So basically like a thousand tiny Stukas raining pointy death on the enemy. I like it, and I think it can be represented fairly well in the game I'm using this for (Crusader Kings 2). I was also already planning on giving them Light Cavalry. Not sure if I'm going to give them actual flying mounts, though having them ride giant eagles would be pretty sweet...

Regardless, they still (I assume) need some kind of dedicated melee unit, since they can't just fly forever. Am I correct in my assumption that they would fare better as Pikemen than otherwise?

cucchulainnn
2013-03-28, 01:43 AM
i would imagine rodeleros would also be very useful. they where armoured soldiers with steel shields who would slip under the opposing pikes and break up their formations by fighting inside the pikemens guard.

Ooo wait you said with wings, never mind, just have them dive bomb em with javelins or some thing.

Octopusapult
2013-03-28, 01:44 AM
How would you guys scale relatively realistic armor ranging from Light to Heavy then? Since Studded Leather is essentially heavier leather armor with less effectiveness, what other armors are actually useless?

Hjolnai
2013-03-28, 03:41 AM
In terms of medieval equipment, you could probably set a scale up like this:
Thin cloth (10 layers)
Thicker cloth (20 layers)
Partial steel armour + cloth (a helmet of some sort, often a few other pieces)
Mail over cloth or under cloth (both happened)
Coat of plates/brigandine/similar over cloth
Coat of plates over mail over cloth
Plate harness over mail over cloth
Plate over coat of plates over mail over cloth (Yes, this happened)
High-quality plate ("proofed") over mail.

The last five don't apply until late 13th century or so, with the better plate not until later than that.

Of course, any system is going to be a simplification, and you could drop several of these categories easily enough.

Spiryt
2013-03-28, 04:39 AM
How would you guys scale relatively realistic armor ranging from Light to Heavy then? Since Studded Leather is essentially heavier leather armor with less effectiveness, what other armors are actually useless?

The problem here is that, generally, scaling armors from 'light' to 'heavy' isn't very feasible.

"Cloth armor" is frequently and ubiquitously set as very light armor, while cloth jacks from later medieval we know about, could easily be some ~ 25 layers of heavy linen fabric, often backed with some leather/hide on outermost layers.

Something like that is obviously actually going to be rather heavy.

In general, weight obviously cannot depend entirely, or even mostly on 'type'.

Mail, plate, coat of plates, scale etc. is just 'material' - while scale shall be pretty much always generally heavier than mail, there will be naturally huge difference between scale out of say:

- 3mm thick plates about 3x1.5 cm in size.

and

- 2mm thick plates about 5x2 cm in size.

The former will, obviously be substantially, potentially even by multiplicity.

Same thing with plate - we have some early Renaissance and 17th century plate suits covering quite a lot of body that were pretty magnificently light.

Polish hussar armors consisting out of breastplate, backplate, helmet, vambrance and some thigh protection frequently weighted just ~ 28 pounds.

On the other hand, 16th century heavy cavalry 3/4 armors, would have a bit greater coverage, but could weight as much as 90 pounds... - those were really thick, massive and interlacing pieces, made in attempt to protect against long firearms bullets.

http://3.bp.blogspot.com/-Pwd5jM23urI/UCjbcEdz_TI/AAAAAAAABQw/YingbnsmkZg/s1600/SDC15011.JPG


So while some 'weight hierarchy' can be probably established as roughly :

- cloth
-mail
-plate/coat of plates
-lammellars/brigandines etc.
-scales

It will ultimately depend on particular piece.

Deepbluediver
2013-03-28, 08:54 AM
Thanks for all your feedback everyone! I'm not usually much for realism in games, but if I can avoid making people cry with my inaccuracies, I think it makes for a better situation. :smalltongue:
I've also been reading up on wikipedia to better understand the minute differences in armor-styles.


In terms of medieval equipment, you could probably set a scale up like this:
Thin cloth (10 layers)
Thicker cloth (20 layers)
Partial steel armour + cloth (a helmet of some sort, often a few other pieces)
Mail over cloth or under cloth (both happened)
Coat of plates/brigandine/similar over cloth
Coat of plates over mail over cloth
Plate harness over mail over cloth
Plate over coat of plates over mail over cloth (Yes, this happened)
High-quality plate ("proofed") over mail.

The last five don't apply until late 13th century or so, with the better plate not until later than that. Of course, any system is going to be a simplification, and you could drop several of these categories easily enough.


The problem here is that, generally, scaling armors from 'light' to 'heavy' isn't very feasible.

I'm aware that armor was made and worn in a huge number of styles and combinations, depending on the era, level of technology, culture, resources available and the goal of the wearer. Representing every single possibility in a tabletop RPG might be possible, but is it really necessary? What I was thinking about was would be the smallest number of categories I could use and have a good representation of real-world styles. Because it's not just about weight (though the light-heavy thing might be misleading) but also about how restrictive armor is, since D&D tends to give an armor penalty to dexterity-based skills and things like swimming.


So if I wanted to do something like this, how far off would I be?

Lightweight armors
-Primarily Cloth and thin leather armor- not too heavy, not to restrictive, but minimal overall protection
-Reinforced/multi-layered leather armor-better protection than cloth, still not too restrictive

Medium Armors
-Scale Mail and Lamellar armors- better protection than leather, more weight and restrictiveness though.
-Chainmail- same similar level of protection as Scale mail, but less weight and less restrictive (or should it be vice versa with scalemail?)
-possibly something like "Reinforced leather" or multilayered leather, or should it have the same stats as chainmail, and just be listed as an alternate variety?

Heavy Armors
-Splint Mail/Banded Mail/Half Plate- better protection tha scale and chain mail, etc etc etc
-Full Plate- best protection in terms of armor and/or damage reduction, but most expensive & restrictive, & weighs the most



For the sake of simplicity, it's assumed that various cloth or leather under-armor/padding are worn with the various types and are included.

I'm also aware that how restricting armor is can be a relative thing. I've seen the youtube videos of people doing cartwheels in full-plate, but there's also a reason track-&-field athletes and gymnasts tend to run around just-shy of nekkid. :smallbiggrin:


Edits: crossed out things that I want to change

Spiryt
2013-03-28, 09:02 AM
Leather armor that weren't multi-layered or 'reinforced' by having a form of many, very thick scales, for example, wasn't really worth much, and we have little evidence of it's use as actual armor.

RPG players are hell bent on their leather though, so some leather could have to stay, I guess.

Deepbluediver
2013-03-28, 09:07 AM
Leather armor that weren't multi-layered or 'reinforced' by having a form of many, very thick scales, for example, wasn't really worth much, and we have little evidence of it's use as actual armor.

RPG players are hell bent on their leather though, so some leather could have to stay, I guess.

I think it's because, in D&D, druids are prohibited from wearing metal armor, so people like some alternatives in non-light categories. But when I think about it, once a druid learns their shapeshifting, armor is kind of pointless anyway. :smallconfused:

So yeah, I got no problems with taking it out.

Octopusapult
2013-03-28, 12:54 PM
Leather armor that weren't multi-layered or 'reinforced' by having a form of many, very thick scales, for example, wasn't really worth much, and we have little evidence of it's use as actual armor.

RPG players are hell bent on their leather though, so some leather could have to stay, I guess.

It has applications in the modern world. Leather jackets keep you from taking road rash when you fall off your motorcycle.

AgentPaper
2013-03-28, 01:24 PM
I'd point out that armor doesn't need to be able to stop a sword to be useful, either. If it can be the difference between a 1 inch deep cut and a tear in your fancy leather jacket, that's still pretty damn useful.

Octopusapult
2013-03-28, 01:34 PM
I'd point out that armor doesn't need to be able to stop a sword to be useful, either. If it can be the difference between a 1 inch deep cut and a tear in your fancy leather jacket, that's still pretty damn useful.

Truth. The goalie masks of old didn't do much but stop the cuts when a puck would hit them in the face. The bruises and breaks were still very much real. But you'd believe they'd still rather wear those than nothing...

Straybow
2013-03-28, 03:55 PM
What benefit does adding studs to armor confer? In D&D, studded leather armor is listed as a whole seperate type, with better AC, lower max Dex, and a higher armor check penalty.

What would the real world implications of studded leather armor (D&D style) be, besides making it heavier? I searched but couldn't find it... there have been other references in this or previous threads about some coat-of-plates that look like leather with "studs" that are actually rivets for holding mid-sized plates on the inside. Gygax and Arneson probably looked at pictures in a book or encyclopedia that didn't explain what it was.


Also, Hide armor is listed seperately as well, as medium instead of light armor. If I managed to make armor out of some animal with thicker skin (rhino or elephant, maybe...or dragon I guess) or stacked multiple layers of regular leather together, would it be more effective? Would it even be possible? Spiryt mentioned buffalo, but that isn't much thicker than cowhide. Rhino and elephant skin on the animal is thick enough that it can stop pistol bullets and shotgun pellets, but neither stops a sharp arrow or spear head. If you want thickness, leather glues well and there is little reason to think that a single thickness is somehow better.

Historically, the major use of animal skin is rawhide covered wooden shields. The rawhide glued to the wood acts compositely to prevent splits in the wood from propagating and holds the split pieces together. A shield less than ½" thick was proof of almost all spears, javelins, arrows, and bolts. However, shields of this sort were expendable. A few minutes of heavy blows from swords and axes is all they could stand. A warrier would actually have a companion who carried spare shields and spears.


Bonus question- in D&D/gaming terms, what would you classify Far-east asian (http://en.wikipedia.org/wiki/Japanese_armour) style armor, made from wood and leather, as? Wood with glued rawhide is effective but not lasting protection. If you carried around a sack full of spare parts to replace pieces damaged after every encounter its effective encumbrance goes way up. =)

There is some debate about Greeks using laminated leather armor. We don't really have surviving examples. It is possible that cured leather, heavy linen, and rawhide could be made into an effective composite.

For game purposes we need to posit a waterproof glue to make it feasible as a lasting piece of equipment that doesn't fall apart in the rain or sea spray. But even then "lasting" just means you can wear it for days or weeks of travel. It can only stop a few hand weapon hits before it becomes significantly weakened, so unless used sword-and-board style it will only last a few encounters. This is complicated by the poor combat mechanics in D&D, in that you don't know if the character is actually hit, if the armor absorbs damage, etc.

The main things that leather armor could do is allow a wearer to do cinematic stuntwork. It doesn't stop a sword, it allows the character to drop and tumble, bounce off walls, rolls with blows, etc without bruising and scraping themselves to bits.

Now, back to studded leather. If a cured leather cuirass could be covered with lightweight iron or bronze discs it could turn an edge or arrow just enough to add an increment of protection. It would look a bit like scale or lamellar armor but the metal is lighter. Again, this would be "ablative" armor in that when a disc takes a heavy blow it has to be replaced, but spares would be relatively light and the bent and broken ones can be reworked by a smith.

Straybow
2013-03-28, 04:02 PM
This may be stretching the bounds of "real world" a bit, but bear with me.

Say that there was a race of small humanoids, roughly 3-4 feet in height on average, with a slim build. In DnD terms, I think this roughly corresponds to a gnome. However, unlike gnomes, they also have wings, allowing them to fly. Their wing strength allows them to fly carrying roughly half their body weight in equipment for a period of roughly ten minutes.

Basically, my question is, what effects would this slim build have on combat tactics for the race, assuming it normally fought human-sized opponents, and a roughly medieval technology level (ie: no guns). Additionally, we're talking about massed combat here, at least 1000 people fighting, and usually much more.

My assumption would be that they would generally prefer to use bows and pikes. Bows because they allow them to stay away from enemies and pepper them with arrows, and pikes because a massed pike formation relies (relatively) less on body strength compared to, say, a sword and shield. Is this true? Or would both tactics be just as ineffective as any other tactic? Or is there some other tactic that might work better? Their strength is in movement. They don't engage, they rely on bows to attack from the air. They use light bows that are only effective for about thirty yards, shooting "indian style" half-draw to the center of mass, and arrows with poison when it comes to a real fight.

Pikes are slow and only of use in formation and in standing ground. This is not what they'd ever want to do. If they need armor penetration they have larger individuals carry a couple of javelins that can be dropped from above. They can also carry sacks of caltrops which they can drop on troops already in movement.

They wear a bleastplate without a back to save weight, and since they only anticipate ranged combat it could be wood with rawhide covering. It isn't much lighter than metal but it is cheaper and can be ditched when escape is called for since their endurance isn't great.

In "real world" terms, the biggest issue is that they'd need a wingspan of about 15 feet. A thirty-pound Andean condor has a 10-foot wingspan, and these guys are bigger and able to carry 50% of their weight...

AgentPaper
2013-03-28, 05:04 PM
In "real world" terms, the biggest issue is that they'd need a wingspan of about 15 feet. A thirty-pound Andean condor has a 10-foot wingspan, and these guys are bigger and able to carry 50% of their weight...

Yeah, the wings and flying bit isn't realistic, but in this case it isn't really meant to be, simply because a realistic model isn't feasible or especially interesting.

Anyways, thanks for all the help. I think I'll have their army composition be something like this:

1) Light Infantry: Carries Javelins, which it uses as the start of battles to deal as much damage as possible. When they run out of ammo, or if they're forced to fight hand to hand somehow, they also carry a small sword. Ideally this is used only as a backup weapon, and their preferred tactic is to retreat and resume skirmishing if at all possible. As far as armor goes, they wear a light breastplate made of thin wood plates held together with leather, a metal cap, and a light wooden buckler used more to block enemy arrows than in actual blocking in melee.

2) Archers: Carries a small bow, which they use throughout battle to pepper the enemy with arrows. Their arrows are exceptionally thin, almost needlelike, and are used mainly to deliver a deadly poison, rather than causing bodily wounds. They aren't issued melee weapons by standard issue, but most carry some sort of knife, dagger, or other small blade to defend themselves in desperate circumstances. They wear metal caps similar to the Light Infantry, but no other armor other than their standard uniforms.

3) Pikemen: Carry Pikes about 3-4 meters long, and have light metal armor, including a metal cap, breastplate, and shoulder guards. The weight of their spears and armor means they can't fly above the enemy for sustained periods like others, but they are able to "sprint", using their wings to effectively take very large strides, allowing them to move much faster for a short time.

These are the professional soldiers of the army, and while they like to brag about how how their superior skill allows them to fight larger opponents, in truth they serve mostly to fend off enemy cavalry (the only troops fast enough to catch them reliably), or as at best a delaying force against enemy troops to allow the rest of the army to regroup and resupply before resuming skirmish tactics.

4) Light Cavalry: Ride War Eagles, and are equipped similarly to Pikemen (and in fact most start out as pikemen before earning their 'wings'), using a slightly shorter and lighter Lance, but the same light metal armor. In battle, they are mostly held in reserve until enemy morale and formations are broken down, at which point they charge in to bravely pursue the enemy as they run away. Occasionally, they will attack targets of opportunity during the main battle, but being made up mostly of nobility, they tend to see themselves as too valuable to risk attacking head-on, with some notable exceptions of varying success.


Edit: Now that I've gotten them figured out, I feel like I should also expand on some of the other races more. I've already got plans for the Minotaur to act sort of like hoplites/macedon, and Centaur are pretty easy to pin down as the equivalent of Mongols. Any other historical or theoretical army tactics/compositions that I could riff off of for other races?


Edit: Edit: Further, does anyone have any recommendations on books to read that would discuss medieval warfare, especially during the High Middle Age? Partly to be able to provide a more detailed and realistic depiction of that kind of warfare, but also to serve as inspiration for less realistic things, like how armies of ogres and minotaurs and so on might operate.

Matthew
2013-03-28, 07:31 PM
I searched but couldn't find it... there have been other references in this or previous threads about some coat-of-plates that look like leather with "studs" that are actually rivets for holding mid-sized plates on the inside. Gygax and Arneson probably looked at pictures in a book or encyclopedia that didn't explain what it was.

"Studded" armour first appears in Chain Mail, is ignored for D&D, and then reappears for AD&D. The context in which it first appears is apparently complex because the table is imported from an early zine, so may not have anything to do with Gygax, certainly not with Arneson. However, Gygax provides two descriptions in the DMG for "studded leather":

"Studded Leather is leather armor to which have been fastened metal studding as additional protection, usually including an outer coat of fairly close-set studs (small plates)." (DMG, p. 27)

"LEATHER ARMOUR is cuir bouli, consisting of coat, leggings, boots, and gauntlets. STUDDED LEATHER adds protective plates set in the leather and an extra layer of protection at shoulder area." (DMG, p. 165)

The latter is preceded by reference to the text he was probably using:

"Note: If you are unfamiliar with medieval armour types, you might find Charles Ffoulkes' ARMOUR AND WEAPONS (http://archive.org/details/armourweapons00ffouuoft) (Oxford 1909) a short and useful text. The armour types I have selected are fitted into a game system."

... which of course does feature "trelliced" and "ringed" armour types.

Straybow
2013-03-29, 02:08 AM
Yeah, haven't had the manuals for decades now. What I got along the way was that the conjecture wasn't supported. Whether from Ffoulkes or Gygax. We don't even really know that cuir bouli was used that way, and tests have shown that it is completely ineffective against arrows. The wax is a lubricant and it aids penetration of the leather.

"Ring mail" is another thing that seems to have no historical basis. I've speculated privately that Japanese style mail, with the loose grid structure rather than tight interlocking structure, could be positioned as "ring mail."

In a way, 3.5e did well to introduce mail shirt. It's good to recognize that physical coverage ought to be part of the "formula," and changing coverage changes the "armor class." If I were to still use that horrible system I'd allow a significant number of add-ons and doubles that improve the defense of a given type of armor.

For example, banded as the traditional lorica segmentata covers only the torso and shoulders. Add faulds and codpiece for an increment. Arm protection for another. Upper leg and knee cops for another. For mail add chausses. Replace the coif with a mantle to double the shoulders.

kardar233
2013-03-29, 02:59 AM
I've got a character set in a campaign that's Iron Kingdoms with Full Metal Alchemist magic (also looking for help in this (http://www.giantitp.com/forums/showthread.php?t=277739) thread), who is a primarily martial character. However, as she uses a massive multitude of weapon styles, I'm taking advantage of the magic system to be able to use whatever weapon set seems right for the task.

Basically, I've decided that she wears a pair of cestus/bracer combinations that she reshapes into shields and weaponry. Now, I'm not expecting to use anything significantly out there like double shields, but she needs to be able to use nearly much any weapon style.

So, the cestus/bracers need to be made up of enough of whatever materials are necessary to make a huge range of prospective weapons and shields. The issue I'm having is figuring out how much of what materials she'd need. I'm guessing she could make just about any pair of weapons or weapon and shield out of a combination of steel, wood and some leather, but I feel like she'd need plenty else.

I'm guessing that the outer limit of how much steel she'd need is when making a pair of hand-and-a-half swords, which based on the numbers I've seen should need about four pounds of steel each. I imagine that the top end of the amount of wood she'd use is with a large shield and a polearm or spear of some sort, but I'm getting conflicting numbers as to the weight of a large shield like an aspis or scutum (or those big Norse round shields I don't know the name of) so that's also a problem, as I'd need to figure out how much and what kind of wood I'd need. I also don't know what other materials might be necessary.

Luckily, she's extremely strong so carting around a pair of fifteen-plus pound cestus isn't going to be too much of a problem.

Can you give me a hand here?

Wardog
2013-03-29, 05:06 AM
3) Pikemen: Carry Pikes about 3-4 meters long, and have light metal armor, including a metal cap, breastplate, and shoulder guards. The weight of their spears and armor means they can't fly above the enemy for sustained periods like others, but they are able to "sprint", using their wings to effectively take very large strides, allowing them to move much faster for a short time.


Is it feasable for 3-4' high people to wield 3-4m (10 - 13') pikes?

Thiel
2013-03-29, 05:07 AM
This may be stretching the bounds of "real world" a bit, but bear with me.

Say that there was a race of small humanoids, roughly 3-4 feet in height on average, with a slim build. In DnD terms, I think this roughly corresponds to a gnome. However, unlike gnomes, they also have wings, allowing them to fly. Their wing strength allows them to fly carrying roughly half their body weight in equipment for a period of roughly ten minutes.

Basically, my question is, what effects would this slim build have on combat tactics for the race, assuming it normally fought human-sized opponents, and a roughly medieval technology level (ie: no guns). Additionally, we're talking about massed combat here, at least 1000 people fighting, and usually much more.

My assumption would be that they would generally prefer to use bows and pikes. Bows because they allow them to stay away from enemies and pepper them with arrows, and pikes because a massed pike formation relies (relatively) less on body strength compared to, say, a sword and shield. Is this true? Or would both tactics be just as ineffective as any other tactic? Or is there some other tactic that might work better?
One thing to remember is that these guys are going to be extremely strong. Tear people limb from limb strong. They need to be if they're going to be able to fly since a humanoid chest is supremely ill suited as an achor point for wing muscles. Birds have big chests in order to give their muscles a better angle to work with. Humanoids do not so they'll need much more powerful muscles.

endoperez
2013-03-29, 05:48 AM
QUESTION(s)
sorry, wanted to make sure this got seen


What benefit does adding studs to armor confer? In D&D, studded leather armor is listed as a whole seperate type, with better AC, lower max Dex, and a higher armor check penalty.

What would the real world implications of studded leather armor (D&D style) be, besides making it heavier?

One explanation that might explain the "studded armor" thing is a layman seeing an armor with studs in it, and not realizing that the studs are holding something else in place under the leather or cloth.

http://0-media-cdn.foolz.us/ffuuka/board/tg/image/1337/60/1337602743889.jpg

In case the image doesn't work, search for images of "coat of plates" and "brigandine". They are metal plates affixed to leather or cloth, and often UNDER it - only the studs keeping them in places are visible on the outside...

JustSomeGuy
2013-03-29, 12:03 PM
One thing to remember is that these guys are going to be extremely strong. Tear people limb from limb strong. They need to be if they're going to be able to fly since a humanoid chest is supremely ill suited as an achor point for wing muscles. Birds have big chests in order to give their muscles a better angle to work with. Humanoids do not so they'll need much more powerful muscles.

Unless they're using the wings for the tearing, that isn't particularly relevant; because you can easily lift your bodyweight on 1 leg, can you do it on 1 arm? The shoulder girdle might well be affected, if it is making room for another one which the wings are attatched to, but fantasy doesn't seem to have a problem giving snakes arms/wings, or turning horse necks into human torsos, or humanoids a second pair of arms, or any other unfeasible musculoskeletal adaptations because COOL.

Silver Swift
2013-03-29, 05:48 PM
Could you apply a sapphire or diamond coating to a metal blade to get a sword that wouldn't go blunt (like they do with eye surgery scalpels)?

More generally, are there any ways for modern technology to significantly improve upon mediaeval swords or where the materials and designs used during the time more or less optimal for their purpose?

Rhynn
2013-03-29, 05:53 PM
Could you apply a sapphire or diamond coating to a metal blade to get a sword that wouldn't go blunt (like they do with eye surgery scalpels)?

Probably not a good idea. You do not want your sword to be razor-sharp, like a scalpel. Swords are only "moderately sharp" - probably on kitchen knife level. You can half-sword a longsword with no ricasso bare-handed (holding the blade in one hand), although that requires knowing how to do it.

Silver Swift
2013-03-29, 06:00 PM
Probably not a good idea. You do not want your sword to be razor-sharp, like a scalpel. Swords are only "moderately sharp" - probably on kitchen knife level. You can half-sword a longsword with no ricasso bare-handed (holding the blade in one hand), although that requires knowing how to do it.

Yes, but isn't that because most materials can't hold a razor sharp edge for very long? If you have a material that could actually hold such an edge without deforming then that would give you an advantage because all the force you are exerting is focussed on a smaller surface, right?

Spiryt
2013-03-29, 06:00 PM
Could you apply a sapphire or diamond coating to a metal blade to get a sword that wouldn't go blunt (like they do with eye surgery scalpels)?

More generally, are there any ways for modern technology to significantly improve upon mediaeval swords or where the materials and designs used during the time more or less optimal for their purpose?

I don't think that such diamond coating would last long in sword applications... It's all about more or less violent contacts, after all.

Modern technology could certainly 'improve' medieval swords, if only because modern steel alloys technology has so much more to offer.

The thing is that sword has no actual practical use in battle/skirmish/duel anymore, so there's no practical info/push on what aspects of sword in particular would have to be improved....

Should it pierce stuff easier? Dull less easily? Should those be of more piercing designs, or cutting ones? Long or short?

It doesn't really matter, because fighting is done with automatic guns, planes, satellites, etc.

As far as "medieval" use goes, even best experts can't really claim to be sure on how exactly swords worked and what little details about them were sought after, and which were only incidental features.

So even if somebody, from whatever reason, made 'ideal' sword for medieval application, it's very probable that he would made some few rather bad mistakes, that would leave the sword less than desireable for battlefield.

Even though precision of material and execution would surpass medieval one greatly.

Deepbluediver
2013-03-29, 06:03 PM
Probably not a good idea. You do not want your sword to be razor-sharp, like a scalpel. Swords are only "moderately sharp" - probably on kitchen knife level. You can half-sword a longsword with no ricasso bare-handed (holding the blade in one hand), although that requires knowing how to do it.

My understanding was that you don't want a sword razor-sharp because it will dull too quickly.

Then I read that the edge-to-edge combat you see in movies wasn't actually how they did things, because smacking sword-blades like that would actually chip or break the weapon in short order.

I'm assuming that anything crystaline would chip and shatter even quicker, because while it might be hard, it's very brittle. In other words, I think it works for tools or knives where can precisely control what you are cutting, but it wouldn't stand up to the wear-and-tear of a combat situation.

And depending on how good your kitchen knives are, I'm not sure you want a sword even that sharp. One time when I was in a rush, I nearly took my fingertip off one time with my best knife.
I own a couple of (reproduction) swords, and they are probably less sharp than the dullest of my knives. I still wouldn't want to be hit with one; you should see what they do to pumpkins the day after Halloween. :smallamused:


I remember trying to find out if you could make a sword out of titanium, and I found several lengthy debates on different websites on if it would be any better than just good steel. (apparently, "amateur weapon smith" is a real hobby)
Medieval weapons had several millenia of development behind them, and despite technology being more advances, it's mostly advanced in different directions.

You might have better luck in terms of armor. I'm not sure how well a ballistic vest would stand up to a sword or mace, but it's probably lighter than a metal breastplate.

AgentPaper
2013-03-29, 06:05 PM
I don't think that such diamond coating would last long in sword applications... It's all about more or less violent contacts, after all.

Modern technology could certainly 'improve' medieval swords, if only because modern steel alloys technology has so much more to offer.

The thing is that sword has no actual practical use in battle/skirmish/duel anymore, so there's no practical info/push on what aspects of sword in particular would have to be improved....

Should it pierce stuff easier? Dull less easily? Should those be of more piercing designs, or cutting ones? Long or short?

It doesn't really matter, because fighting is done with automatic guns, planes, satellites, etc.

So basically, what's needed here is context. Are these swords being sent into the past or something to be used by medieval soldiers? Or in a modern world without gunpowder or some other reason for no guns? What kind of armor would they be contending with? And who is wielding them, untrained recruits? Professional soldiers? Bio-enhanced super-soldiers? Aliens?

Having a uber-sharp blade is bad if you're an average joe, but less so if you're a space marine or jedi.

Edit: Also, regardless of whether it's realistically feasible, this article (http://www.newscientist.com/article/dn16610-diamond-no-longer-natures-hardest-material.html) might interest you. It covers two new materials, wurtzite boron nitride and Lonsdaleite, which have been shown to be theoretically much stronger than Diamond.

Silver Swift
2013-03-29, 06:27 PM
So basically, what's needed here is context. Are these swords being sent into the past or something to be used by medieval soldiers? Or in a modern world without gunpowder or some other reason for no guns? What kind of armor would they be contending with? And who is wielding them, untrained recruits? Professional soldiers? Bio-enhanced super-soldiers? Aliens?

Having a uber-sharp blade is bad if you're an average joe, but less so if you're a space marine or jedi.

The motivation behind the question was pure curiosity, resulting from a discussion about diamond swords and finding out about that diamond scalpel thing.

But I can make up some context to make the question more concrete: say time travelling to early Roman empire and creating an elite group of soldiers armed with future weapons (thinking up some nonsense reason for why they don't take guns with them is left as an exercise to the reader :smalltongue: ) to counter Roman legions and stop the Roman empire from getting of the ground.

warty goblin
2013-03-29, 06:34 PM
Could you apply a sapphire or diamond coating to a metal blade to get a sword that wouldn't go blunt (like they do with eye surgery scalpels)?

The better question is why would you really want to? What stops a sword cutting somebody isn't so much its lack of edge so much as armor, and diamond-coating your edge won't let you chop through metallic armor anyways. It will make your edge fragment when you do hit somebody in armor though.


More generally, are there any ways for modern technology to significantly improve upon mediaeval swords or where the materials and designs used during the time more or less optimal for their purpose?

Better steel for the most part. The impression I've gotten looking at people's comments on how replica swords handle though suggests that those truest to the originals' designs perform the best. Despite what we sometimes seem to think, intelligence doesn't begin in the modern era, and period things tend to be extremely well engineered for their purposes given the materials people were working with.

Spiryt
2013-03-29, 06:49 PM
But I can make up some context to make the question more concrete: say time travelling to early Roman empire and creating an elite group of soldiers armed with future weapons (thinking up some nonsense reason for why they don't take guns with them is left as an exercise to the reader :smalltongue: ) to counter Roman legions and stop the Roman empire from getting of the ground.

If they, from whatever weird reason had to stick to roughly period weapons, instead of producing some guns and vehicles, making technology really decisive would be tricky.

Doesn't matter how much sharper, more resilient, springy etc. is your sword if that Roman bastards wield their shields and swords so brutally compared to you.

If 'future' weapon would mean full access to modern metallurgy etc. then actually well fitted plate(ish) armors with a lot of aluminium/titanium use in proper places could make difference.

Shields from some really tough polymers instead of humble plywood as well.

But that leaves the realm of 'real world' really quick. :smalltongue::smallwink:

AgentPaper
2013-03-29, 06:54 PM
Weapons probably wouldn't change much, aside from possibly slightly stronger/more durable blades, though a diamond tip just on the point of the sword could improve it's ability to pierce armor. Spears and arrows especially would benefit from this.

A modern Compound bow would probably also be pretty useful, allowing you to keep shooting for longer and aim more reliably, though the training of the archer is still far more important, and such training hasn't really improved, and in fact is probably far worse compared to, say, a british longbowman's training.

A bigger improvement would be in the soldier's armor, both in construction and material, probably either plastic or some kind of carbon fiber. Combine in some of the more flexible stuff and clear plastic for visors and shields, and you've got a soldiers that's pretty much invincible to anything the Romans can cook up short of siege weaponry.

Honestly though, if you're going to be using time travel, you'd have a much easier time simply going back to the middle ages and grabbing a bunch of existing soldiers, who would be already trained, already equipped, and already experienced in combat. Hand them a few new toys to sweeten the deal, and you've got yourself an unstoppable army.

Finally, regardless of how you get your soldiers, hands-down the biggest improvement would be in the logistics you could supply, using trucks, mechanized farming and mining, and factories to keep your army fed, watered, and fully equipped. Not to mention improved medicine to keep them all alive and free of disease. Far more soldiers died of disease and infection than ever died in actual battle, after all.

Rhynn
2013-03-29, 07:21 PM
Yes, but isn't that because most materials can't hold a razor sharp edge for very long? If you have a material that could actually hold such an edge without deforming then that would give you an advantage because all the force you are exerting is focussed on a smaller surface, right?

No, you don't want the edge to be razor-sharp. A coating isn't going to help you when the edge contacts something and gets chipped - a razor-sharp sword edge would get nicks in it very easily, weakening the sword. And as I tried to imply, it would limit the uses of the sword.

It just doesn't offer a substantial advantage. Normal swords are perfectly capable of cutting people into pieces, and they're not going to dull so fast as to be a problem. Scalpels probably may - tiny blades that need to be ultra-sharp, cutting into and around hard materials - sure, you want to protect their edge as much as possible. Swords? You just need a whetstone and a chance every now and then to use it.


Honestly though, if you're going to be using time travel, you'd have a much easier time simply going back to the middle ages and grabbing a bunch of existing soldiers, who would be already trained, already equipped, and already experienced in combat. Hand them a few new toys to sweeten the deal, and you've got yourself an unstoppable army.

Why would medieval soldiers be superior to Roman Republic/Empire legions? As far as personal training and discipline go, I have a hard time believing that medieval soldiers were superior to Marian Roman legions, and I suspect they were probably largely inferior. Aside from mercenaries, knights were close to the only "professional" soldiers, and they weren't full-time. Most "soldiers" were "weekend warriors" (well, not even weekly training in most cases, probably), levies, etc. Roman legionares were veterans who served for decades and saw a lot of action.

I actually think the Roman legions might have been fairly devastating in the medieval period. I'm not even convinced that medieval strategists and generals would have been superior. (Heck, I wouldn't be suprised if Tacitus and other Roman authors were considered essential reading for military commanders in the medieval period - I'm not expert on the subject, though.)

Just because something is later in time doesn't mean it's better.

Deepbluediver
2013-03-29, 07:43 PM
Why would medieval soldiers be superior to Roman Republic/Empire legions? As far as personal training and discipline go, I have a hard time believing that medieval soldiers were superior to Marian Roman legions, and I suspect they were probably largely inferior. Aside from mercenaries, knights were close to the only "professional" soldiers, and they weren't full-time. Most "soldiers" were "weekend warriors" (well, not even weekly training in most cases, probably), levies, etc. Roman legionares were veterans who served for decades and saw a lot of action.

I actually think the Roman legions might have been fairly devastating in the medieval period. I'm not even convinced that medieval strategists and generals would have been superior. (Heck, I wouldn't be suprised if Tacitus and other Roman authors were considered essential reading for military commanders in the medieval period - I'm not expert on the subject, though.)

Just because something is later in time doesn't mean it's better.

At it's best, the Roman army was very good, and they would win against almost any less well drilled or better-practiced force. Some people forget that at it's height, the Roman empire included much of Europe, not just the meditaranian. I don't know what, if anything, of Roman military strategy might have been left over in Europe after the Roman regulars withdrew.

Roman infantry, to my knowledge, liked to carry those large shields into battle, backed up with short throwing lances and swords.

For actual armies (as opposed to just conscripted hordes) Medieval tactics tended to have lighter armored infantry with smaller shields but with longer spears and lances (2-handed versions), I think. You might be able to find some historical examples of these two types of forces clashing, but I don't know which one was "better".

Rome had cavalry, as far as I know, but I don't think there where of the same sort as the heavily armored knights that are stereotypical of medieval Europe. The heavy cavalry charge was a typical Medieval battle tactic, but I don't know if the Romans had anything similar.

And the last major area, where I'm totalling ignorant, is whether or not the Roman army frequently employed archers. The infantry could defend itself better against archers with those heavy shields and tight formations, but I don't know if Rome would counter with it's own bowmen if if they simply marched over anyone in their way.

AgentPaper
2013-03-29, 07:52 PM
Why would medieval soldiers be superior to Roman Republic/Empire legions? As far as personal training and discipline go, I have a hard time believing that medieval soldiers were superior to Marian Roman legions, and I suspect they were probably largely inferior. Aside from mercenaries, knights were close to the only "professional" soldiers, and they weren't full-time. Most "soldiers" were "weekend warriors" (well, not even weekly training in most cases, probably), levies, etc. Roman legionares were veterans who served for decades and saw a lot of action.

I actually think the Roman legions might have been fairly devastating in the medieval period. I'm not even convinced that medieval strategists and generals would have been superior. (Heck, I wouldn't be suprised if Tacitus and other Roman authors were considered essential reading for military commanders in the medieval period - I'm not expert on the subject, though.)

Just because something is later in time doesn't mean it's better.

The Romans were great and all, but let's not forget that there's a reason they're not around anymore.

Roman legions would not, in fact, have been devastating in the medieval period, and we know this because they were around in the medieval period. Or at least, they could have been, if Byzantium hadn't stopped using them already, and probably not because they didn't like their stinky cheese rations.

TuggyNE
2013-03-29, 08:38 PM
And the last major area, where I'm totalling ignorant, is whether or not the Roman army frequently employed archers. The infantry could defend itself better against archers with those heavy shields and tight formations, but I don't know if Rome would counter with it's own bowmen if if they simply marched over anyone in their way.

If I recall correctly (I am not an expert by a very far shot), Rome used auxiliaries for archery, and they weren't necessarily a major component of their tactics most of the time. Still, they weren't unknown or unused.

Rhynn
2013-03-30, 01:57 AM
The Romans were great and all, but let's not forget that there's a reason they're not around anymore.

Roman legions would not, in fact, have been devastating in the medieval period, and we know this because they were around in the medieval period. Or at least, they could have been, if Byzantium hadn't stopped using them already, and probably not because they didn't like their stinky cheese rations.

Byzantium was not a gigantic empire and I would think lacked the resources to train and arm full-time armies on the scale that Rome did. The fall of the Roman Empire has little to do with the fighting capability of its legions. There's no reason to think they abandoned the Marian legion model due to it being inferior or outdated, especially when it's obvious they lacked the imperial wealth to support it. The Roman Empire was driven by conquest, like most empires are, and eventually it gets too hard to conquer anything else, administration gets too hard, and then there's a collapse. The fall of any empire is obviously complex and has no single reason; division (between West and East - which became the Byzantine Empire over a century before the Roman Empire fell), corruption, etc. By the 4th century, the Empire's legions had shrunk considerable (from their strength on paper; i.e. undermanned legions were fielded).

Incidentally, the Byzantine Empire did continue to use the legions until the 7th century, when the adoption of the theme system (for reasons of needing more, rather than better, armies) decentralized the Byzantine military, turning it into a more or less feudal system with "yeoman" soldiers (or yeoman-serfs; the soldiers bound themselves and their children to farming land they did not own and to military service).

So without some indication that the Marian legions were actually found inferior and replaced for those reasons, rather than for reasons of administration and finance, I'm not inclined to swallow that they were in any way inferior - not when they had armor generally superior to medieval armies (most of those armies would have little more than a helmet and maybe a gambeson), and weapons no worse than those of medieval armies.

Why exactly would medieval soldiers in Roman times be any kind of super-unit?


Roman infantry, to my knowledge, liked to carry those large shields into battle, backed up with short throwing lances and swords.

It all depends on the period. Pre-Marian, you had the velites (skirmishers with javelins, etc., the poorest and least experienced troops), hastati (younger or poorer men, lighter armor, pilums, short swords, shields), principes (better armored and more experienced, but same basic thing), and triarii (same thing again). Post-Marian Reforms is the typical "Roman legion" we think of, with mail (lorica hamata) and later the iconic lorica segmentata, scutum shields, pilums throwing spears, and gladius swords.


Rome had cavalry, as far as I know, but I don't think there where of the same sort as the heavily armored knights that are stereotypical of medieval Europe. The heavy cavalry charge was a typical Medieval battle tactic, but I don't know if the Romans had anything similar.

Rome used both equites (wealthier citizens; more or less the origin of European knights) and cavalry auxiliaries. AFAIK they did not use heavy cavalry, but they certainly met it - Parthian (and Scythian, Sarmatian, etc.) cataphracts.


And the last major area, where I'm totalling ignorant, is whether or not the Roman army frequently employed archers. The infantry could defend itself better against archers with those heavy shields and tight formations, but I don't know if Rome would counter with it's own bowmen if if they simply marched over anyone in their way.

I can't speak to Roman strategy, really, but they certainly had archer auxiliaries, and probably slingers. (Really more comparable to archers in range than to javelin skirmishers.)

AgentPaper
2013-03-30, 03:38 AM
Why exactly would medieval soldiers in Roman times be any kind of super-unit?

Because we're talking about early roman republic here, not late empire praetorians. They're still using bronze weapons and armor at this point.

If you're going to try and butt heads against the full might of the Roman Empire at it's height, then things would certainly get trickier, but I think you'd still do well with a later medieval army or two, if you were willing to fit them all out with some decent armor.

Rhynn
2013-03-30, 05:24 AM
Because we're talking about early roman republic here, not late empire praetorians. They're still using bronze weapons and armor at this point.

Peregrine Crow said "early Roman empire." I'm not sure how you interpret that to be Republic at all, and certainly don't see how you interpret that to mean bronze.

What are you basing this on? The Roman Republic began in the 6th C BC, some 600 years after the Iron Age began in Europe. The manipular legions (the hastati-principe-triarii model) of 4th C BC onward already had iron armor for certain. Before that, there were no legions - they used hoplite phalanxes. But they probably had iron for several centuries already.

And why are bronze weapons and armor worse, especially substantially worse? Bronze was replaced by iron because iron was much easier to obtain and process than tin (required for bronze), especially as time went on.

I'm not sure how Praetorians (the Imperial bodyguard founded in the 3rd C BC) enter into any of this. I wouldn't really consider them in a general discussion about Roman legions, manipular or Marian. They seem to me to have nothing to do with the effectiveness of the Roman legions.

And how does any of that make medieval soldiers better? A soldier's quality is a matter of training and discipline, and it seems clear that in Europe, the training and discipline of the Roman legions was rarely equalled until professional armies returned post-Renaissance. (Well, there were obviously exceptions, like the Swiss pikemen.)


If you're going to try and butt heads against the full might of the Roman Empire at it's height, then things would certainly get trickier, but I think you'd still do well with a later medieval army or two, if you were willing to fit them all out with some decent armor.

Late Medieval, maybe. Firearms and knights in plate harness are going to be be a bit of a game-changer, probably. But "medieval" is a pretty broad term (something like an 800-year span).

If we want to get specific, I can certainly concede that, for instance, 14th C mounted knights in full harness and with lances could give a Marian legion cohort serious trouble (like I understand the eastern cataphracts gave the legions). If you want to specify your argument further, that's certainly going to affect my objections to it - you started out with a non-specific blanket statement.

AgentPaper
2013-03-30, 07:55 AM
Peregrine Crow said "early Roman empire." I'm not sure how you interpret that to be Republic at all, and certainly don't see how you interpret that to mean bronze.

He further explained that the idea was to stop it "before it got off the ground". So, some time before they took full control of the Italian peninsula, at least.


What are you basing this on? The Roman Republic began in the 6th C BC, some 600 years after the Iron Age began in Europe. The manipular legions (the hastati-principe-triarii model) of 4th C BC onward already had iron armor for certain. Before that, there were no legions - they used hoplite phalanxes. But they probably had iron for several centuries already.

I may have been misinformed, but when I learned about the romans, it was stated explicitly that early on a lot of their stuff was still bronze, and it wasn't until later on that they really standardized the equipment and had enough iron for full iron/steel armor.


And why are bronze weapons and armor worse, especially substantially worse? Bronze was replaced by iron because iron was much easier to obtain and process than tin (required for bronze), especially as time went on.

Well, that's kinda the point. If you could make a ton of bronze, sure it'd be great, but before you start using iron you can't fully armor your whole army, because you just can't make enough of the stuff. So, you end up with less, and less thick, armor.


And how does any of that make medieval soldiers better? A soldier's quality is a matter of training and discipline, and it seems clear that in Europe, the training and discipline of the Roman legions was rarely equalled until professional armies returned post-Renaissance. (Well, there were obviously exceptions, like the Swiss pikemen.)

I gotta call bullhokey on this. You're talking like all the armies of the medieval period were just big hordes of untrained peasants. Medieval armies certainly weren't as standardized as roman ones, but they were still professional soldiers.


Late Medieval, maybe. Firearms and knights in plate harness are going to be be a bit of a game-changer, probably. But "medieval" is a pretty broad term (something like an 800-year span).

If we want to get specific, I can certainly concede that, for instance, 14th C mounted knights in full harness and with lances could give a Marian legion cohort serious trouble (like I understand the eastern cataphracts gave the legions). If you want to specify your argument further, that's certainly going to affect my objections to it - you started out with a non-specific blanket statement.

Well, yes, I would assume that if you're going to pick an army from history, you'd pick one of the better ones.

Hawkfrost000
2013-03-30, 03:15 PM
Then I read that the edge-to-edge combat you see in movies wasn't actually how they did things, because smacking sword-blades like that would actually chip or break the weapon in short order.

Sorry to just drop in, but this really really bothers me.

Every modern fencing master and every ancient fencing manual i have ever met or read says you parry with the edge. Specifically the true edge (The edge that points away from your body when you hold the sword out.) because you have enormous leverage with that action compared to using the flat or false edge of the blade.

Someone who parried with the flat of his sword would preserve their sword, but find themselves very, very dead.

DM

Traab
2013-03-30, 04:36 PM
The only time I would think a diamond coated sword would be useful would be while slaughtering unarmored peasants. The clean slice would prevent chipping, and the coating would help prevent dulling. I admit to not being any kind of expert, but when I picture weapons going through armor, its either a pierce or a hack. Neither would be served that well by coating an entire weapon for reasons that have been gone over. You want a thick enough edge to chop through the armor without deforming, you arent trying to slice the armor. You have to find the right balance between enough of a point or edge to focus the force being applied to as small an area as possible while having it be thick enough to not break itself under the pressure.

Am I at all right in this school of thought? Or am I so far off base that its scary?

Rhynn
2013-03-31, 12:31 AM
Sorry to just drop in, but this really really bothers me.

Every modern fencing master and every ancient fencing manual i have ever met or read says you parry with the edge. Specifically the true edge (The edge that points away from your body when you hold the sword out.) because you have enormous leverage with that action compared to using the flat or false edge of the blade.

Someone who parried with the flat of his sword would preserve their sword, but find themselves very, very dead.

DM

What do you think about John Clements' stance (http://www.thearma.org/essays/edgemyth.htm) on this?

My understanding is that while some manuals specify edge (and some flat), most manuals are actually completely agnostic on this, not specifying whether you parry with edge or flat. (Because the manuals were meant for students of the master or school, not for mail-order courses or for reference 600 years later, they left out a lot of "obvious" things.)

It doesn't help, of course, that most people have a completely twisted idea of what parrying is. Edge-on-edge is really "blocking," and not AFAIK a preferred technique. (Obviously, when you have to, you do anything to stay alive - including blocking, whether with edge or flat.) Longsword and rapier parries, for instance, are offensive counter-actions that meet an opponent's weapon, force it aside, and deliver a strike back.


The only time I would think a diamond coated sword would be useful would be while slaughtering unarmored peasants. The clean slice would prevent chipping, and the coating would help prevent dulling. I admit to not being any kind of expert, but when I picture weapons going through armor, its either a pierce or a hack. Neither would be served that well by coating an entire weapon for reasons that have been gone over. You want a thick enough edge to chop through the armor without deforming, you arent trying to slice the armor. You have to find the right balance between enough of a point or edge to focus the force being applied to as small an area as possible while having it be thick enough to not break itself under the pressure.

Am I at all right in this school of thought? Or am I so far off base that its scary?

Weapons did not, as a rule, go through armor. You went around armor. You might cause bruising (even serious bruising and maybe trauma to bones) through mail or plate, and it is possible to pierce mail (but not very deeply, unless it's modern riveted repro/SCA junkmail), but it was not what you would try to do in a fight, because the chances were so bad.

A top-heavy weapon like an axe or mace might dent plate (I'm doubtful) and bruise through mail (much more likely), but an axe isn't going to hack through armor.

Injuries at medieval gravefields from battles are mostly to the legs, where byrnies and hauberks did not reach - that is, wounds are where there was no armor. And look at any harnischfechten manual's illustrations - you won't see a single one (or I haven't, anyway; I'd be amazed to be proven wrong) where a longsword (a weapon evolved to deal with plate harness) is used to go through armor. I recommend looking at those illustrations anyway - they are liable to really change how you think about armored medieval combat.

And wear on your weapon is almost never going to be an issue in the middle of combat, unless you're using a really old weapon that's not been kept in shape.

TuggyNE
2013-03-31, 01:22 AM
What do you think about John Clements' stance (http://www.thearma.org/essays/edgemyth.htm) on this?

Not that I'm the one addressed, but I am distressed to discover that I simply cannot follow his argument at all; he may well be correct, but his quotes, reasonings, and illustrations prove nothing to my limited understanding. :smallfrown: (If anything, some of the illustrations look an awful lot like they're contradicting what he's saying.)

Hawkfrost000
2013-03-31, 02:09 AM
What do you think about John Clements' stance (http://www.thearma.org/essays/edgemyth.htm) on this?

My understanding is that while some manuals specify edge (and some flat), most manuals are actually completely agnostic on this, not specifying whether you parry with edge or flat. (Because the manuals were meant for students of the master or school, not for mail-order courses or for reference 600 years later, they left out a lot of "obvious" things.)

It doesn't help, of course, that most people have a completely twisted idea of what parrying is. Edge-on-edge is really "blocking," and not AFAIK a preferred technique. (Obviously, when you have to, you do anything to stay alive - including blocking, whether with edge or flat.) Longsword and rapier parries, for instance, are offensive counter-actions that meet an opponent's weapon, force it aside, and deliver a strike back.

Emphasis mine.

I was a tad unclear with my original post, i was typing while quite annoyed. It is a bit of a sore spot for me, i had this debate with someone in the flesh and eventually convinced him to try it with real swords (blunted of course) and he still insisted i was wrong, even when my parries defended myself 9/10 times (he did a weird feint thing once) and his only defended himself 3/10 times and only then when he was over-anticipating and providing an absurd amount of resistant force...

Anyway, my definition of parry is similar to yours. I agree that the ridiculous "blocks" are just that, but for several reasons beyond the edge alignment. Mostly to do with the fact that it is rediculously easy to make very nasty and powerful plays from a heavy "bind" of the swords like that. Fiore and Lichtenhauer (sp?) both detailed them.

But Fiore's Parare are parries as i would use the word, primarily defensive but can easily be turned into an offensive action. These would be delivered with the True Edge of the sword, how they would be received (to create a edge on edge alignment) would depend on the opponent and his reaction.


Not that I'm the one addressed, but I am distressed to discover that I simply cannot follow his argument at all; he may well be correct, but his quotes, reasonings, and illustrations prove nothing to my limited understanding. :smallfrown: (If anything, some of the illustrations look an awful lot like they're contradicting what he's saying.)

His argument is a tad unclear, he has a very clear thesis (edge on edge is bunk) then he gives a bunch of evidence but never really ties it back to his thesis.

DM

Yora
2013-03-31, 11:15 AM
There is a major difference between "always do this" and "if you get an opportunity, you could do this, so train how it would be done".

The idea that edge on edge impact results in better levarage seem more than doubtful in regards to mechanical physics, and I would dare to say bogus.
Maybe you could avoid the other blade sliding down your blade, because they are biting into each other and become wedged, but that way you'd ruin a sword every training session.

fusilier
2013-03-31, 01:11 PM
I've had only a tiny amount of sword training; in Spanish renaissance sword drill (and I can't even tell you the author, the person who taught me stated that he had to study different manuals to piece together a coherent system). Anyway, the blocks are done edge to edge, only the defense against a thrust is done with the flat -- deflecting the opponent's sword to the side. In all cases the action is "natural", attempting to block with the flat would mean rotating the wrist into an awkward and probably "weak" position. So this may be the "mechanical" leverage, referenced above?

As for chipping the edge of a sword -- it happened in battle. I vaguely remember being told that most samurai replaced their swords after each major battle, the edge was just that chipped. I think Biringuccio's book mentions repairing chipped sword edges (although I may be wrong on that, I know he mentions a kind of solder for filling cracks on swords).

Yora
2013-03-31, 03:30 PM
Was there are specific handgun model that was called "the great equalizer", and who made the term popular?

paddyfool
2013-03-31, 03:42 PM
NB on the Romans: one major innovation they'd have had to deal with in fighting a medieval army would be the fairly humble invention of stirrups (http://en.wikipedia.org/wiki/Stirrup#History) (perhaps the key feature, although there were others, such as the cantled saddle, in making cavalry more effective after about the 6th century AD in Europe, although horse-breeding might also have steadily led to better and better warhorse stock once this started getting going as a major part of military action).

Frozen_Feet
2013-03-31, 03:44 PM
I believe the origin of such phrases comes from original Colt revolver, the first greatly succesful revolver model. ("God made men, Colt made them equal.") I'm not well-learned on early revolver models, so that's all I can say.

EDIT: Bronze weapons were superior to early iron weapons, and continued to be at least equal throughout middle ages. The reason bronze was abandoned was not inferiority, it was cost and availability - specific countries had monopoly on tin, and eventually they started to run out, making costs prohibitive and necessitating moving to iron, which corroded more easily and was harder to melt and work.

fusilier
2013-03-31, 03:54 PM
I believe the origin of such phrases comes from original Colt revolver, the first greatly succesful revolver model. ("God made men, Colt made them equal.") I'm not well-learned on early revolver models, so that's all I can say.

EDIT: Bronze weapons were superior to early iron weapons, and continued to be at least equal throughout middle ages. The reason bronze was abandoned was not inferiority, it was cost and availability - specific countries had monopoly on tin, and eventually they started to run out, making costs prohibitive and necessitating moving to iron, which corroded more easily and was harder to melt and work.

I think you are correct about the Colt revolver, the original being the Colt-paterson: http://en.wikipedia.org/wiki/Colt_paterson

On the Iron vs. Bronze -- I know that was true of early artillery, but I didn't think bronze was superior to iron for edge weapons?

Spiryt
2013-03-31, 04:21 PM
On the Iron vs. Bronze -- I know that was true of early artillery, but I didn't think bronze was superior to iron for edge weapons?

I don't know a thing about metalurgy in general, but I would think that as far as edge goes, it could indeed be easily superior - at least well made weapon grade 'bronze' alloy can be work hardened and shaped to pretty impressive edge, while iron not so much.

Steel edge would be different matter, obviously, but AFAIU, especially in the beginning of iron weapons history, effective carbonization of iron edge was a bit tricky.

warty goblin
2013-03-31, 06:23 PM
I don't know a thing about metalurgy in general, but I would think that as far as edge goes, it could indeed be easily superior - at least well made weapon grade 'bronze' alloy can be work hardened and shaped to pretty impressive edge, while iron not so much.

Steel edge would be different matter, obviously, but AFAIU, especially in the beginning of iron weapons history, effective carbonization of iron edge was a bit tricky.

One thing that's interesting to note in bronze age weaponry is the amount of design effort that seems to have gone into making an effectively stiff blade, particularly on the bigger swords (http://www.bronze-age-swords.com/British_and_European.htm). It can be a little hard to pick out because of the mirror-finish on those blades, but many of them have very pronounced raised portions running down the central spine of the blade.

Interestingly this same thing can be seen in early iron weapons forged in the same designs. Whether this was later abandoned because iron-working progressed to the point where it was no longer necessary, or people found other ways of increasing their blades' stiffness that were easier to execute in iron I have no idea. I'd imagine forging a raised section into an iron blade would be extremely difficult however.

Another interesting detail is that the edges of many bronze swords are forged in by hamming the metal out thin. This apparently work-hardens the bronze enough to allow it to hold a better edge. It's visually incredibly striking. (http://www.myarmoury.com/talk/files/p1020010_201.jpg)

I love the contrast in this photo (http://www.myarmoury.com/talk/files/p1020008_170.jpg). The gladius is just so much less elegant in proportion and construction than the bronze weapon. The bronze sword is so delicate and precise looking by comparison with the big, sloppy looking iron blade.

Also, an interesting page about Mycenian bronze armor (http://www.salimbeti.com/micenei/armour1.htm). Of particular interest (at least to me) is the laminar cuirass a bit down the page. It's pretty much unlike any other armor I've ever seen, and certainly has the most impressive neck protection.


On a related note, I'd love a really good dedicated bronze age RPG.

Rhynn
2013-03-31, 06:34 PM
I don't know a thing about metalurgy in general, but I would think that as far as edge goes, it could indeed be easily superior - at least well made weapon grade 'bronze' alloy can be work hardened and shaped to pretty impressive edge, while iron not so much.

Steel edge would be different matter, obviously, but AFAIU, especially in the beginning of iron weapons history, effective carbonization of iron edge was a bit tricky.

Yeah, my understanding was that iron replaced bronze for reasons of availability; but that only steel was actually superior to bronze. (Of course, what is "iron" and what is "steel" is a bit vague anyway, since AFAIK all smelted and forged iron is going to have carbon in it, and some "steels" have less carbon than some "irons," etc. ...)

There is, IIRC, a completely erroneous scene in Mika Waltari's novel The Egyptian (Sinuhe egyptiläinen) where an iron sword is demonstrated and cleaves right through a bronze sword. I seriously doubt that would work. Maybe you could cut gold or silver with an iron sword?

The differences in design between bronze, iron, and steel weapons are interesting. The metallurgy had a big effect on what shape of blade (and how long of a blade) you could have.

warty goblin: I just get a myArmoury.com banner on those direct image links. :smallfrown: They don't allow direct linking of images.

And a Bronze Age RPG would be pretty cool. I guess there's Mazes & Minotaurs (http://mazesandminotaurs.free.fr/revised.html), but that may not be exactly what you're thinking. RuneQuest's world of Glorantha is kind of a mix of ancient cultures with bronze as the main metal, and lately even the West of Glorantha has gotten a new look (http://moondesignpublications.com/blog/jeff/seshnela-art-direction) to replace the overdone "medieval European knights" thing with something rarely seen in RPGs. I think any edition of RuneQuest would work well for a Bronze Age RPG.

warty goblin
2013-03-31, 07:04 PM
Yeah, my understanding was that iron replaced bronze for reasons of availability; but that only steel was actually superior to bronze. (Of course, what is "iron" and what is "steel" is a bit vague anyway, since AFAIK all smelted and forged iron is going to have carbon in it, and some "steels" have less carbon than some "irons," etc. ...)

Another issue with iron that only recently came to my attention is getting the ore pure. Unless you can build a kiln capable of actually melting the iron and making crucible/Wootz steel, apparently you're likely to have a lot of slag in the metal. This is a bad thing, and makes it weak and brittle, since your sword/whatever is basically full of little bits of rock. Bronze, since it melts at a lower temperature, is, I assume, easier to get relatively free of contaminants.

Traditional crucible steel is also very hard to work with due to the micro-structure of the metal when it comes out of the kilm. Here's some cool (http://www.youtube.com/watch?v=nXbLyVpWsVM)videos (http://www.youtube.com/watch?v=jwSXYq5uCUY&playnext=1&list=PL4oqKmVgardBn9wCrzAnGBlFSB6fUT5FS&feature=results_main)about people trying to make and work with it. Eight hours of hammering just to get it into a bar? Roll those craft checks people!

(The guy from the second link also has some very interesting videos about pattern-welding Viking style blades that are well worth a watch. He goes into a lot of detail, and the level of craftsmenship that goes into such a weapon is staggering.)


There is, IIRC, a completely erroneous scene in Mika Waltari's novel The Egyptian (Sinuhe egyptiläinen) where an iron sword is demonstrated and cleaves right through a bronze sword. I seriously doubt that would work. Maybe you could cut gold or silver with an iron sword?
I very much doubt you could cleave a sword of just about any metal with another sword. Even soft metal isn't exactly easy to cut. Shatter it if the blade is excessively brittle, or else crumple and bend it out of all usefulness I could see, but straight up cut? Probably not.

(I've seen some pictures of copper swords, which I can't imagine worked all that well. Copper is either excessively soft, or if you work harden it to the point where it could maybe take an edge, very brittle. I'd have no trouble believing that a bronze or iron sword could wreck a copper blade in short order)


The differences in design between bronze, iron, and steel weapons are interesting. The metallurgy had a big effect on what shape of blade (and how long of a blade) you could have.
I had figured bronze would have forced much shorter blades at first, but apparently they go up to 90cm or so. While it certainly isn't a longsword, that's still a plenty big weapon, although Wikipedia (http://en.wikipedia.org/wiki/Bronze_Age_sword) claims its right at the limit of what you can do in bronze. Certainly it's about as long as most as most iron weapons reached for a long while.

I've also read that bronze swords tended to be as long as people could cast them, which is another limit. Since iron blades are forged you can keep hammering them longer, but with bronze you need to get it to fill the mold. This is apparently quite difficult (http://www.bronze-age-craft.com/swordcasting.htm).


warty goblin: I just get a myArmoury.com banner on those direct image links. :smallfrown: They don't allow direct linking of images.
Sorry, they worked fine when I checked them. Here's the link to the thread (http://www.myarmoury.com/talk/viewtopic.php?t=18446&postdays=0&postorder=asc&start=0), which also contains a lot of other photos that make me drool.


And a Bronze Age RPG would be pretty cool. I guess there's Mazes & Minotaurs (http://mazesandminotaurs.free.fr/revised.html), but that may not be exactly what you're thinking. RuneQuest's world of Glorantha is kind of a mix of ancient cultures with bronze as the main metal, and lately even the West of Glorantha has gotten a new look (http://moondesignpublications.com/blog/jeff/seshnela-art-direction) to replace the overdone "medieval European knights" thing with something rarely seen in RPGs. I think any edition of RuneQuest would work well for a Bronze Age RPG.
I'm not familiar with RuneQuest, but will take a look. Regardless, Mycenaean palace civilization would be a really cool setting, although I'd imagine it would be a bit of a head-screw for most people. If all people write are property lists, you can't have wizards and spell books.

Rhynn
2013-03-31, 09:27 PM
Another issue with iron that only recently came to my attention is getting the ore pure. Unless you can build a kiln capable of actually melting the iron and making crucible/Wootz steel, apparently you're likely to have a lot of slag in the metal. This is a bad thing, and makes it weak and brittle, since your sword/whatever is basically full of little bits of rock. Bronze, since it melts at a lower temperature, is, I assume, easier to get relatively free of contaminants.

Yeah, my understanding was that early ironwork was very brittle, even in weapons and armor. Cast/wrought iron blades and armor (breastplates) would be pretty shoddy. Mail was less affected, as I understand - in fact, by what I've read, even medieval mail was rarely made from steel, because iron was sufficient.


I had figured bronze would have forced much shorter blades at first, but apparently they go up to 90cm or so. While it certainly isn't a longsword, that's still a plenty big weapon, although Wikipedia (http://en.wikipedia.org/wiki/Bronze_Age_sword) claims its right at the limit of what you can do in bronze. Certainly it's about as long as most as most iron weapons reached for a long while.

Huh, that is longer than I had thought. Not a long rapier or longsword (is that blade or blade + tang length?), but definitely a "full-sized" sword.


I'm not familiar with RuneQuest, but will take a look. Regardless, Mycenaean palace civilization would be a really cool setting, although I'd imagine it would be a bit of a head-screw for most people. If all people write are property lists, you can't have wizards and spell books.

RuneQuest 6 (http://www.thedesignmechanism.com/runequest.php) is the newest and best "new era" RuneQuest (Chaosium's RQ2 and Avalon Hill's RQ3 are classics but hard to come by, and RQ2 is very Glorantha-tied.) RQ6 is written very much in a "decide setting details for yourself." Mechanics are given, but how they interact with the world is up to you.

Incidentally, no spellbooks are needed - sorcerers learn their spells from somewhere, but this can be by contact with demons, from revelation or enlightenment, or from studying the 10' tall glyphs carved into a smoothed mountain-side... all up to the GM.

warty goblin
2013-03-31, 10:22 PM
Yeah, my understanding was that early ironwork was very brittle, even in weapons and armor. Cast/wrought iron blades and armor (breastplates) would be pretty shoddy. Mail was less affected, as I understand - in fact, by what I've read, even medieval mail was rarely made from steel, because iron was sufficient.

One interesting angle to take with the development of metal weapons and armor is the challenge of obtaining large pieces of metal. Since bronze is cast, you can make very large ingots, which have been found in shipwrecks dating from the late bronze age. However it is apparently very hard to forge bronze, this paper (http://www.academia.edu/1200494/THE_FUNCTION_OF_BRONZE_AGE_SHIELDS) suggests ~200 rounds of hammering and annealing to produce the disk for a Yethold shield (http://en.wikipedia.org/wiki/Yetholm-type_shields) for example. Casting bronze in sheets I suspect is quite difficult as well, since it would be challenging to get the metal to fill the mold evenly.

Iron apparently suffered from similar problems in terms of getting big enough chunks of it to make things. Thus chainmail, a substantially less obvious development than simply strapping metal plates to oneself. Also pattern welded blades, which, gorgeous aesthetics aside, are very much the long way 'round to making a sword.

There's an interesting comparison to be made I think between pattern welded swords and bronze swords. The first uses different types of metal to get a hard edge and soft, strong core. The second uses a homogeneous material, but hardens the edges through hammering to get the same effect.


Huh, that is longer than I had thought. Not a long rapier or longsword (is that blade or blade + tang length?), but definitely a "full-sized" sword.
Counting tang I suspect. There are bronze swords with hilts riveted on to a separately cast blade. My limited understanding is that these were earlier weapons, and the technique was abandoned when metalsmiths learned how to make reasonably long single-piece weapons. he larger weapons were cast in one piece, since the riveted design is fairly obviously structurally weaker than a single piece blade. There is indirect textual evidence for this from the Iliad. In Book III during their duel, Menelaus clonks Paris over the head with his 'silver studded' sword, only for the weapon to break at the hilt. At this point, like any good warrior king, Menelaus starts hauling Paris about by his helmet crest. I've always liked that, as not only does it make Paris look even more ridiculous, but suggests a very versatile approach to combat, employing multiple techniques.


RuneQuest 6 (http://www.thedesignmechanism.com/runequest.php) is the newest and best "new era" RuneQuest (Chaosium's RQ2 and Avalon Hill's RQ3 are classics but hard to come by, and RQ2 is very Glorantha-tied.) RQ6 is written very much in a "decide setting details for yourself." Mechanics are given, but how they interact with the world is up to you.

Incidentally, no spellbooks are needed - sorcerers learn their spells from somewhere, but this can be by contact with demons, from revelation or enlightenment, or from studying the 10' tall glyphs carved into a smoothed mountain-side... all up to the GM.
Interesting, I'll have to see if I can rustle up a copy.

Matthew
2013-03-31, 10:55 PM
NB on the Romans: one major innovation they'd have had to deal with in fighting a medieval army would be the fairly humble invention of stirrups (http://en.wikipedia.org/wiki/Stirrup#History) (perhaps the key feature, although there were others, such as the cantled saddle, in making cavalry more effective after about the 6th century AD in Europe, although horse-breeding might also have steadily led to better and better warhorse stock once this started getting going as a major part of military action).

The Great Stirrup Controversy is quite a fun read on this subject. It probably was not as big a gap in effectiveness as might be thought, but certainly was an improvement.

Discussing "the Romans" is always fraught with peril because of the very different way their armies were recruited, organised, trained and equipped over a thousand or so years of history. Probably organisation was the key feature for the success of the Roman army, and taking them out of time and place would be a big problem, since they would be cut off from their logistics. If we assume that is not a problem for whatever reasons and just concentrate on the battlefield tactical effectiveness of a Roman army versus, for example, the Anglo-Danes or the Normans in 1066 (ignoring also numerical superiority), the latter armies would probably have the edge in state of the art equipment, but probably had less of it than the Romans (this is totally arguable, of course, since we have no idea if the army was really fully equipped or if not in what proportion). If we take, for example, one of the famous Roman armies, such as that of Scipio Africanus, Marius, Pompey, Caesar, Augustus or Trajan, my money would be on the Romans, but it would be far from a whitewash.

On the subject of Roman equipment, in the early to middle Republican period (6th to 3rd centuries BC) they would have probably have been equipped similarly to the Greeks, which means largely iron weapons and bronze or linen armour.

Galloglaich
2013-03-31, 11:38 PM
Some interesting activity here of late! I'll try to chime in with a few points.

Ok so we can start with the later Republic / early Imperial Roman Army. I say late Republic since the Marian Reform was in 100 BC which was still during the Republican period.

http://www.dougherring.com/FestHistory/01%20Roman%20Legion.jpg

Equipment of Roman Armies from 100 BC
Front-line Legionnaires would have short sleeved iron mail shirts (lorica hamata), bronze or brass helmets, and shields, as well as probably some textile armor underneath. But basically armor protection only on their torso. They were armed with javelins (pilum) as their principle weapon backed up by the short cut and thrust sword (gladius). They would be supported by cavalry armored with scale or mail armor, also principally on the torso, and armed with javelins and light lances. These would be considered very light cavalry by Medieval standards. Further support would be supplied by light armed troops called velites (http://en.wikipedia.org/wiki/Velites) who ran back and forth protected mainly by light wicker shields and threw darts and javelins, though these were largely phased out after the Marian reforms.

In addition the Marian era Roman army had excellent engineering capabilities and good torsion spring artillery ranging from man portable ballistas to much larger weapons used for sieges.

http://www.profimedia.si/photo/detail-of-the-battle-of-hastings-from/profimedia-0014763572.jpg
Equipment of 'Medieval' Armies 7th- 11th Century
This is an era still dominated by infantry in Europe, most armies were similar to the barbarian armies the Romans faced during the time of Marius. Javelins were the primary weapon, as well as other missiles like thrown axes. The spear and shield were the close-combat weapon, backed up by swords and more axes, as well as knives like the sax, and protected primarily by their shields. In the 7th and 8th Century personal body armor was still pretty rare in Barbarian Europe but toward the end of this period mail armor was becoming much more ubiquitous and most soldiers had it, as well as helmets. For example at Hastings and Clontarf and most of the principle battles of the 11th Century the current consensus is that many if not most of the fighters were armored..

Roman armies in this period used infantry which was 'lighter' in the sense that they were equipped much like the Visigoths, Franks and Lombards who they fought, with longer swords and sometimes less armor. Their cavalry however had become heavier in the Eastern (Parthian / Sassanid) style, wearing head to toe armor and riding armored horses which superficially resembled a 13th Century French knight (not without reason, the Franks included a lot of Iranian cavalry who settled as nobility in France). They also had cavalry archers though this was mostly in the East.

http://upload.wikimedia.org/wikipedia/commons/thumb/a/a6/Knight-Iran.JPG/407px-Knight-Iran.JPG
Note for example this 6th Century Sassanid fresco depicting one of their Kings. Hist kit could be right out of a monty python movie.

There is, incidentally, no reason to assume that the Byzantine Army was any weaker than the Western Roman army was in this period, in fact to the contrary since under Justinian they briefly reconquered and retook control over much of the Western Roman Empire

http://upload.wikimedia.org/wikipedia/commons/9/9b/Justinian555AD.png

http://en.wikipedia.org/wiki/Justinian_I#War_in_Italy.2C_second_phase.2C_541.E2 .80.93554

http://img382.imageshack.us/img382/7584/53manasseschronicleaz4ns5.jpg

Equipment of Medieval Armies 12th - 14th Century
By this era, medieval armies were notably different from the 'early medieval' or migration era armies which the Romans were familiar with. The main difference was the dominance and power of the heavy cavalry. The invading Crusader armies entering Anatolia and the Middle East in the 11th Century were a shock to the Byzantines for several reasons. They had developed better armor, either due to advances in iron working technology, armor design, or to the textile component or for some other reason we don't know, but the Byzantines reported shock at the effectiveness of the 'Frankish' mail panoply.

The European barbarians had improved technology and tactics for heavy cavalry considerably. This may have hinged mainly on changes to saddle design as well as horse breeding. Or it may have been better stirrups or some other reason. Whatever the cause, the "Frankish" (i.e. European) heavy cavalry caused a major shock to the Byzantines, and more so to the Turks, both of whom noted that in spite of the first Crusade being often (though not always) a poorly organized rabble, small numbers of Frankish cavalry were routinely routing much larger cavalry forces of their enemies, even though at this point it seems that their horses were still mostly unarmored or lightly armored.

A classic example of this is the Siege of Antioch, which historians are still somewhat at a loss to explain (I've seen explanations from everything to the discovery of the 'holy lance' to the idea that the European stallions were excited by the Arab mares... but the truth is simpler. Good cavalry with good leadership - plus trouble on the other side)

http://en.wikipedia.org/wiki/Siege_of_Antioch#Battle_of_Antioch

Infantry in this Crusade was also making a mark. The Genoese urban militia, armed with upgraded crossbows designed in their town workshops, proved to be very effective in several sieges in the 'Holy Land', leading to the creation of an instant new industry for the then small fishing town (soon to be major maritime republic) of Genoa - who began to rent out their militia as mercenaries.

Toward the end of this era, European plate armor was becoming increasingly widespread. The Cistercian monks had spread the overshot water wheel all over Europe and with the introduction of the Catalan forge and Barcelona hammer (automated bellows and trip-hammers, powered by water wheels) iron production was way up in both quantity and quality. European tactics and military organization were improving dramatically as well.

By the end of this period, both cavalry and infantry were wearing armor, usually at least partly plate armor. Whoever said that it was normal for most Medieval soldiers to fight without any armor is dead wrong. One of the biggest battlefields so far excavated, at Wisby in 1361, where an army of Swedish peasants was defeated by German mercenaries and due to the heat, were buried with their gear on in mass graves, shows that all of them had armor, mostly mail reinforced with brigandine or coat of plates armor (textile armor with iron plates inside of it) and the only vulnerable spot was their lower legs, which not coincidentally seems to be where most of them were hit.

http://en.wikipedia.org/wiki/Battle_of_Visby

http://media.kunst-fuer-alle.de/img/41/m/41_00077148.jpg

Equipment of Medieval Armies 14th-15th Century
By this point, most both cavalry and infantry were wearing plate armor. Very good plate armor. In fact by the early 1400's the biggest armor supplier in Europe, the city of Milan, was specializing in making steel armor, which was much stronger than the earlier iron armor, and it was also much better fitted and designed, so that fully articulated harnesses could be worn.

Whoever mentioned that bronze isn't much worse than iron was correct, depending on the alloy bronze is a bit (about 5-10%) heavier than iron for the same volume, but not particularly weaker... which is why it was still being used for helmets so long after iron had been discovered. But steel is much better than bronze. And whats more, it was relatively cheap. A proofed Milanese harness was well within the financial means of most free peasants in Central Europe for example. By the mid 1400's the city of Augsburg in Germany was making even better tempered steel armor at a cheaper price.

Infantry was becoming much more dominant on the battlefield, the heyday of heavy cavalry was ending though it was still very effective.

Infantry weapons had vastly improved. The crossbow had reached power that rivaled much larger Roman artillery weapons of the Marian era, with a range of over 300 meters. The longbow had made it's battlefield debut, as had the Mongol and Ottoman composite recurves (more potent weapons of the same family as the Hunnish composite bow which gave the Roman armies so much trouble during the time of Atilla). Firearms were well established and had become primary battlefield weapons during the Hussite wars, organized around war-wagons equipped with guns and supported by cannons, which functioned a lot like tanks.

http://i4.photobucket.com/albums/y110/Nephtys/Miscellaneous/McBride%20Newark%20and%20Others/WA_31.jpg

Primary hand weapons ranged from the two-handed war flail, the two handed maces like the morgenstern (http://en.wikipedia.org/wiki/Morning_star_%28weapon%29) and the godendag (http://en.wikipedia.org/wiki/Godendag), the ahlespeiss (http://en.wikipedia.org/wiki/Ahlspiess), and the lethal halberd, as well as pikes to keep the cavalry away. Sidearms included four foot longswords and kriegmessers, as well as shorter swords, sabers and messers.

Shields were still used but larger ones mostly as static defense for archers, and as portable protection in siege warfare much the way SWAT teams use ballistic shields today (and for the same reason). For one on one combat, the smaller but tougher iron buckler was popular.

Late Medieval cavalry was even more formidable. Both rider and horse were protected. Riders carried very long heavy lances of up to 18' or even more. Their sidearms included longswords and axes, as well as specialized armor-piercing weapons like war-hammers and war-picks.

Advantages for the Romans
The main advantage of a Marian era Roman army would be size. At the time of Augustus there were 28 Roman legions, with an averge of about 4,500 fighting men in each legion. That's over 126,000 men. Which is a massive army by Medieval standards. Now of course that force was spread out all over the Empire, but even 4 or 5 legions massed together for a major emergency would be 20,000 soldiers, which is still very big by the standards of say, the 13th Century. Medieval armies tended to be made up of well equipped and well trained men, knights, urban and rural militias, and professional mercenaries. They were expensive and therefore small forces. The Roman army in the late Republic / Early Empire would be a major threat on this level alone.

Another Roman advantage would be engineering, especially as it relates to siege warfare. Most medieval battles were sieges. In the Early Medieval period siege warfare was crude, engines like trebuchets and torsion spring weapons were poorly developed, and tunneling and sapping wasn't very sophisticated. Roman legions were trained to build very good fortifications, as well as roads, bridges, and anything else they needed to get the job done. By the late Medieval period however this advantage would be less as the geniuses of the Renaissance had absorbed the lessons of Greek and Roman classical engineering and had in fact expanded and improved upon it significantly.

The third factor was discipline. Roman armies had very good discipline particularly after the Marian reform. They would not break and run under the stress of combat, they would not break ranks to loot. This was a major advantage particularly compared to the early medieval armies which were often (though not always) poorly disciplined.

Advantages of the Medieval army
The medieval army would have several advantages. Medieval armies would have much better armor, covering more of their bodies. Their missile weapons (longbows, crossbows, composite bows and guns) would vastly out-range the Roman javelins, and punch right through their scutum shields. This was a major problem for the Marian type Roman armies in Parthia, leading to several catastrophic defeats.

The primary hand weapons of Medieval infantry (flails, two handed maces, halberds) would similarly slice through or crush Roman shields easily (much like the Dacians did with their falces) and their secondary weapons would out-reach the gladius. As for discipline, the Swiss and the Czechs, to cite just two examples of many, clearly rivaled Roman discipline in terms of not breaking or surrendering, though they would sometimes attack without orders.

Medieval cavalry would have been a huge problem for Roman legions. They had a very hard time with Parthian cavalry (and archers) at Carrhae (http://en.wikipedia.org/wiki/Battle_of_Carrhae#The_battle) and of course with the Goths at Adrianople, and I suspect an encounter between a post Marian reform legion and a Medieval army would have had similar results.

http://i4.photobucket.com/albums/y110/Nephtys/Medieval%20European%20Armour/GothicBard.jpg
There is no doubt that late medieval cavalry was more dangerous than 1st Century Parthian cavalry and I don't think a Roman legion could have held up to a major cavalry charge by good cavalry.

G

Matthew
2013-03-31, 11:48 PM
Equipment of Medieval Armies 12th - 14th Century
By this era, medieval armies were notably different from the 'early medieval' or migration era armies which the Romans were familiar with. The main difference was the dominance and power of the heavy cavalry. The invading Crusader armies entering Anatolia and the Middle East in the 11th Century were a shock to the Byzantines for several reasons. They had developed better armor, either due to advances in iron working technology, armor design, or to the textile component or for some other reason we don't know, but the Byzantines reported shock at the effectiveness of the 'Frankish' mail panoply.

The European barbarians had improved technology and tactics for heavy cavalry considerably. This may have hinged mainly on changes to saddle design as well as horse breeding. Or it may have been better stirrups or some other reason. Whatever the cause, the "Frankish" (i.e. European) heavy cavalry caused a major shock to the Byzantines, and more so to the Turks, both of whom noted that in spite of the first Crusade being often (though not always) a poorly organized rabble, small numbers of Frankish cavalry were routinely routing much larger cavalry forces of their enemies, even though at this point it seems that their horses were still mostly unarmored or lightly armored.

A classic example of this is the Siege of Antioch, which historians are still somewhat at a loss to explain (I've seen explanations from everything to the discovery of the 'holy lance' to the idea that the European stallions were excited by the Arab mares... but the truth is simpler. Good cavalry with good leadership - plus trouble on the other side)

Some of this "shock" must have been over-exaggerated by the writers themselves, because the Byzantines were very familiar with the Normans and others prior to the crusades, both from employing them as mercenaries against the Turks and in their wars with Robert Guiscard. Either that, or there was a sudden technological revolution in the space of a few short years. Part of it may be the audience that they were writing for, who might perhaps not have been familiar with contemporary military matters at all, or else stylistic choices (Anna Komena for example was purposefully writing in a very archaic style, sort of neo-classical, which is why she chooses old terminology for describing the western Europeans).

At Antioch, there was a dearth of cavalry on the crusader side when they sallied out, so it is probably also a good example of their worth as foot soldiers.

Galloglaich
2013-04-01, 12:12 AM
The whole edge vs. flat thing was a crazy, crazy crazy debate in HEMA circles, and I think the only person who felt really strong about it was John Clements who created some of the most epic flamewars in internet history over this subject back in the period 2002-2007 or thereabouts. Memes have been made about it, even videos.

I think most people adopt a more flexible position.

He had a point that sharps will bite into each other, but when the quillions are pointed in alignment with the edges, you really have no choice but to parry that way at least partially, and we do have direct evidence or this, namely the hundreds of illustrations in the surviving medieval fencing manuals. But of course, these images can be interpreted different ways.

Some rapier manuals explicitly say to parry with the edge, but of course a rapier is different than a longsword.

Suffice it to say it's an issue which is not easy to prove either way, a lot of energy was spent trying to do so without much benefit to anybody! But in the HEMA world outside of ARMA (which I think is most of the HEMA world at this point, as ARMA has gotten pretty small) it's uncommon to parry exclusively with the flat.

G

rrgg
2013-04-01, 01:41 AM
One issue I have with the whole Rome vs medieval thing is deciding what exactly a proper comparison would be. If you take the Roman empire at its height sure you are talking about an army of up to 250,000 legionaries but compared to the population that boils down to only around one per 200 civilians. Compared to some medieval or dark age kingdoms that could mean the backbone of the Roman army outnumbered by housekarls or even knights alone.

As a general rule though I don't think that the Marian legions would have held many advantages over medieval, late-roman, or even dark age armies. History is generally not a case of "they dun got stupid" and whenever blank hits the fan usually the last thing to go is anything that helps someone defeat their enemies.

Yora
2013-04-01, 07:39 AM
Does anyone know of a website where you can get decent quality sound samples of common guns? Movies and games seem to have a tendency to use sounds that are simply cool to completely random weapons, and I was wondering how they should actually sound.
Youtube has nothing.

warty goblin
2013-04-01, 10:15 AM
Does anyone know of a website where you can get decent quality sound samples of common guns? Movies and games seem to have a tendency to use sounds that are simply cool to completely random weapons, and I was wondering how they should actually sound.
Youtube has nothing.

Guns as a rule don't record or play back particularly well. Sonic booms just don't translate to speakers, and so they always lack that distinctive *crack*.

Rhynn
2013-04-01, 01:24 PM
Movies and games seem to have a tendency to use sounds that are simply cool to completely random weapons, and I was wondering how they should actually sound.

Well, it's more that they usually use completely fake, very very quiet sound effects for guns.

Strange Days was an odd movie to watch because the guns all used different, louder effects than basically every other movie ever (although still not very loud at all, but that's understandable - gunshots will cause hearing damage, especially indoors, so you're not going to want that volume of noise in your movie).

There's plenty of YouTube videos with real guns being really fired, but those are going to lack that crack and be low on volume, too.

fusilier
2013-04-01, 03:15 PM
As a general rule though I don't think that the Marian legions would have held many advantages over medieval, late-roman, or even dark age armies. History is generally not a case of "they dun got stupid" and whenever blank hits the fan usually the last thing to go is anything that helps someone defeat their enemies.

One thing that we must keep in mind is that a roman legionnaire is not merely some guy with a gladius and pilae, just like a modern soldier isn't some guy with an assault rifle and grenades. Training, discipline, tactics, etc., are perhaps even more significant than the technological level of the equipment. While technology is, in my opinion, pretty robust (in the sense that it doesn't usually disappear, especially if it had widespread use and support), those other factors are much weaker and can degrade for any number of reasons. The training and discipline of a Roman Legion would have a lot to do with the institutions that the Roman government provided, and if those collapse even if you know what the tactics are supposed to be without the proper training they can't be effectively employed. Likewise, as rrpg points out, the Roman army was very big (compared to a medieval state) and standardized, this is because the Roman state could support such a big army. Effective tactics used by a big army might be very different than those employed by a much smaller force.

If you combine technological differences with institutional ones, then attempting to recreate roman legions may have been impractical/doomed to failure. Many of the Renaissance era commanders studied Roman tactics very closely, but none seem to have attempted to emulate them.

fusilier
2013-04-01, 03:22 PM
Well, it's more that they usually use completely fake, very very quiet sound effects for guns.

Strange Days was an odd movie to watch because the guns all used different, louder effects than basically every other movie ever (although still not very loud at all, but that's understandable - gunshots will cause hearing damage, especially indoors, so you're not going to want that volume of noise in your movie).

There's plenty of YouTube videos with real guns being really fired, but those are going to lack that crack and be low on volume, too.

A fellow reenactor told me how his neighbor is a foley artist and one day she asked to borrow a variety of his guns -- she wanted to record the various sounds the guns made (cocking, clicking, etc., not firing!). Even though his guns weren't the ones being used in the movie! They don't care, they just want something that "sounds" good.

A common trope in movies and TV is to have the classic "cocking" sound of a gun when one person is threatening another. Recently I was watching a show where somebody turned a readied bolt action rifle toward the protagonist, and there was an inexplicable, loud "click"! :-)

Wardog
2013-04-01, 04:01 PM
The Great Stirrup Controversy is quite a fun read on this subject. It probably was not as big a gap in effectiveness as might be thought, but certainly was an improvement.


According to Osprey's European Medieval Tactics (1) (http://www.ospreypublishing.com/store/European-Medieval-Tactics-%281%29_9781849085038), the stirrup was less tactically significant that commonly supposed. You could still have heavy shock cavalry without stirrups, with the appropriate saddle.

The main advantage of stirrups was apparently strategic/logistical, in that it reduced the strain and constriction on the legs, making riding more comfortable and hence increasing the distance an army could march. (Particularly in colder climates, where restricted blood circulation in the legs would be more debilitating).

Ashtagon
2013-04-01, 04:04 PM
I think you are correct about the Colt revolver, the original being the Colt-paterson: http://en.wikipedia.org/wiki/Colt_paterson

On the Iron vs. Bronze -- I know that was true of early artillery, but I didn't think bronze was superior to iron for edge weapons?

Bronze is far easier to smelt, which was the chief reason it was the first major metal to see industrial use - in the Bronze Age, it wasn't generally practical to forge iron due to the higher temperatures required.

The actual physical properties of bronze were quite varied. Bronze is of course an alloy, and the specific ratios of components had a huge impact on hardness. In addition, whether it was cold-wrought or cast would also affect hardness. Jewellery-quality bronze would typically be softer than iron (all the better to work fine detail into it), but weapons-grade bronze would usually be harder than iron, at least up until Damascus steel forging techniques were invented.


Further reading
http://www.engineeringtoolbox.com/modulus-rigidity-d_946.html
http://www.jewelry-secrets.com/Metals/Metals-Mohs-Scale-Of-Hardness/The-Mohs-Scale-Of-Hardness-For-Metals.html
http://www.allaboutgemstones.com/metal_jewelry_bronze.html

Rhynn
2013-04-01, 10:11 PM
A common trope in movies and TV is to have the classic "cocking" sound of a gun when one person is threatening another. Recently I was watching a show where somebody turned a readied bolt action rifle toward the protagonist, and there was an inexplicable, loud "click"! :-)

And, of course, you have to **** the hammer even with double-action handguns. Just like you have to pump a shotgun that's not been fired yet if you're threatening someone with it. (Or you rack the slide.) A bolt action rifle making the cocking sound is quite something, though...

:smallamused:

It's even got a TVTropes page (http://tvtropes.org/pmwiki/pmwiki.php/Main/DramaticGunCock).

Matthew
2013-04-01, 11:03 PM
According to Osprey's European Medieval Tactics (1) (http://www.ospreypublishing.com/store/European-Medieval-Tactics-%281%29_9781849085038), the stirrup was less tactically significant that commonly supposed. You could still have heavy shock cavalry without stirrups, with the appropriate saddle.

The main advantage of stirrups was apparently strategic/logistical, in that it reduced the strain and constriction on the legs, making riding more comfortable and hence increasing the distance an army could march. (Particularly in colder climates, where restricted blood circulation in the legs would be more debilitating).

Also, it allowed the rider to "rise" or "stand" in the saddle, which we see depicted from time to time with the couched lance or in close combat. Overall, stirrups were an improvement, but incremental rather than revolutionary. Gladly gone is the rather silly notion of the "pole-vaulting" knight through lack of stirrups, though it is still possible to read about it on Wikipedia. :smallbiggrin:

TuggyNE
2013-04-01, 11:14 PM
And, of course, you have to **** the hammer even with double-action handguns. Just like you have to pump a shotgun that's not been fired yet if you're threatening someone with it. (Or you rack the slide.) A bolt action rifle making the cocking sound is quite something, though...

Might I politely suggest you bypass the filter in the mod-approved style? (I.e., "**** the hammer", using black color tags.)

Joran
2013-04-01, 11:31 PM
And, of course, you have to **** the hammer even with double-action handguns. Just like you have to pump a shotgun that's not been fired yet if you're threatening someone with it. (Or you rack the slide.) A bolt action rifle making the cocking sound is quite something, though...

:smallamused:

It's even got a TVTropes page (http://tvtropes.org/pmwiki/pmwiki.php/Main/DramaticGunCock).

At least with some double action/single action revolvers, cocking back the hammer will lighten the trigger pull so the gun will be easier to fire. Hurray, Hollywood is right in an edge case!

Normal audio equipment also does a poor job of capturing the sound of explosions and Hollywood sure loves their fiery explosions. Thanks Mythbusters!

Rhynn
2013-04-02, 01:14 AM
Might I politely suggest you bypass the filter in the mod-approved style? (I.e., "**** the hammer", using black color tags.)

If the powers that be want to filter out common words, I guess that's their prerogative... I didn't even remember the wordfilter existed, and I doubt I will next time, either.

Galloglaich
2013-04-02, 09:09 AM
Also, it allowed the rider to "rise" or "stand" in the saddle, which we see depicted from time to time with the couched lance or in close combat. Overall, stirrups were an improvement, but incremental rather than revolutionary. Gladly gone is the rather silly notion of the "pole-vaulting" knight through lack of stirrups, though it is still possible to read about it on Wikipedia. :smallbiggrin:

There does seem to have been some significant I agree incremental but substantially so, changes between the old Central Asian / Iranian style of heavy cavalry (clibinari / cataphract) and the "Frankish" type which emerged in the 10th-11th Century. One difference which has been suggested to me is that the earlier type of heavy cavalry didn't charge that fast in a canter or a gallop* the way the later did. Probably due to the saddles, stirrups and the breeding / training of the horse. But I think there must be more to it. The horse being the most expensive bit of kit for knights and men-at-arms I suspect that may be really important as well.

* I know that the full gallop wasn't quite as common as we are led to believe in European lance charges either..

My other theory is that whereas cataphracts were very well protected, it was at the expense of mobility, I think with the armor they wore (lamellar and extra plates over mail or yushman / mail and plate ala Byzantine klibanion) they would have a hard time fighting on the ground if they were dismounted, and maybe not that great fighting with shorter weapons from the saddle. By contrast, "Frankish" heavy cavalry of 11th century had a mail and textile panoply which wasn't nearly as encumbering, arguably, or as hot, and may have been more effective in the followup stage of a lance charge as well as during the initial charge.

It's also interesting that whereas 6th Century Sassanid cavalry seems to have used armored horses, in the early days of Feudal European cavalry (10th -11th Century), the horses seem to have often been mostly uprotected, or at any rate horse armor is rare in artwork and doesn't show up that much in equipment rosters and so on. It's only later in the 13th and 14th Century that horse armor seems to come back, peaking with the sophisticated plate barding of the 15th and early 16th, and then fading away again pretty quickly in the early musket / cannon era.

G

Wardog
2013-04-02, 12:05 PM
It's also interesting that whereas 6th Century Sassanid cavalry seems to have used armored horses, in the early days of Feudal European cavalry (10th -11th Century), the horses seem to have often been mostly uprotected, or at any rate horse armor is rare in artwork and doesn't show up that much in equipment rosters and so on. It's only later in the 13th and 14th Century that horse armor seems to come back, peaking with the sophisticated plate barding of the 15th and early 16th, and then fading away again pretty quickly in the early musket / cannon era.

G

Could that simply be due to the Sassanid Empire being large and wealthy (and according to Wikipedia, "the peak of ancient Persian civilization"), whereas early feudal states were not?

Galloglaich
2013-04-02, 01:37 PM
Could very well be, though arguably even by the 11th Century Europe was becomming one of the most successful production zones for iron, hence the ubiquity of the mail body armor for warriors and steel swords and so on. This was mainly due to the revolution of the overshot water wheel, as well as the windmill and some related technologies. Europe was also becomming a major zone for textile manufacturing (horse armor was often made of textiles)

So maybe there was some other tactical reason.

You might even argue that the 'messy' / decentralized state of affairs in Europe contributed to the spread of this type of technology in what came to be called the first Renaissance

http://en.wikipedia.org/wiki/Renaissance_of_the_12th_century

Which, like the more famous second Renaissance, seems to have been centered in the areas with the least effective political control (i.e. Northern Germany, Flanders, the Baltic and Northern Italy).

All that said though I really don't know much about the economy of Persia in the Classical to Medieval period and I think increasingly, I really should because they keep coming up again and again as really important influences on European culture and technology. As well as being kind of a trade and culture bridge to India and China via the Silk Road and maritime trade routes..

G

Yora
2013-04-02, 01:47 PM
The dutch windmill was primarily used for draining fields and not so much for grinding grain. It would work for draining mines as well, if you have places with sufficient and reliable wind.

Also, it allowed the rider to "rise" or "stand" in the saddle, which we see depicted from time to time with the couched lance or in close combat. Overall, stirrups were an improvement, but incremental rather than revolutionary. Gladly gone is the rather silly notion of the "pole-vaulting" knight through lack of stirrups, though it is still possible to read about it on Wikipedia. :smallbiggrin:
I know from experience that stirrups make it a lot easier to sit stable when the horse is galloping, because you can rest all your weight on your feet. A good rider can rest his full weight on his inner upper legs, but in full armor and with a lance and shield in hand, I would guess it becomes a lot harder to keep yourself from sliding down and having your whole weight supported by your crotch. Not only is it uncomfortable, you are also much more likely to slide over the ride or left side with all the shaking that's going on. A horse is basically barrel shaped and you either rest on smooth fur or leather. Sliding of the side is very easy, with stirrups it's almost impossible to happen.

Your lance digging into the ground is the least of your problems. Sideway motion is the real issue.

Matthew
2013-04-02, 08:30 PM
There does seem to have been some significant I agree incremental but substantially so, changes between the old Central Asian / Iranian style of heavy cavalry (clibinari / cataphract) and the "Frankish" type which emerged in the 10th-11th Century. One difference which has been suggested to me is that the earlier type of heavy cavalry didn't charge that fast in a canter or a gallop* the way the later did. Probably due to the saddles, stirrups and the breeding / training of the horse. But I think there must be more to it. The horse being the most expensive bit of kit for knights and men-at-arms I suspect that may be really important as well.

* I know that the full gallop wasn't quite as common as we are led to believe in European lance charges either..

My other theory is that whereas cataphracts were very well protected, it was at the expense of mobility, I think with the armor they wore (lamellar and extra plates over mail or yushman / mail and plate ala Byzantine klibanion) they would have a hard time fighting on the ground if they were dismounted, and maybe not that great fighting with shorter weapons from the saddle. By contrast, "Frankish" heavy cavalry of 11th century had a mail and textile panoply which wasn't nearly as encumbering, arguably, or as hot, and may have been more effective in the followup stage of a lance charge as well as during the initial charge.

It's also interesting that whereas 6th Century Sassanid cavalry seems to have used armored horses, in the early days of Feudal European cavalry (10th -11th Century), the horses seem to have often been mostly uprotected, or at any rate horse armor is rare in artwork and doesn't show up that much in equipment rosters and so on. It's only later in the 13th and 14th Century that horse armor seems to come back, peaking with the sophisticated plate barding of the 15th and early 16th, and then fading away again pretty quickly in the early musket / cannon era.

I suspect horses went largely unarmoured amongst the Franks due to economic considerations, but I think you are correct that tactical value played a role [e.g. cavalry in this period was typically not suffering continuous attack from arrows, unlike Richard's march to Jaffa, and mobility was key]. However, it is worth noting that horse armour is mentioned at least once in twelfth century Arthurian romance without special comment.

The thirteenth century continuation of William of Tyre insists that contemporary armour for knights was too restrictive to allow them to fight effectively on foot, by comparison with their more lightly equipped forebears (Richard the Lionheart and his men in the specific instance). Such a development suggests increasingly specialised "lancers", which would accord with your speculation.



I know from experience that stirrups make it a lot easier to sit stable when the horse is galloping, because you can rest all your weight on your feet. A good rider can rest his full weight on his inner upper legs, but in full armor and with a lance and shield in hand, I would guess it becomes a lot harder to keep yourself from sliding down and having your whole weight supported by your crotch. Not only is it uncomfortable, you are also much more likely to slide over the ride or left side with all the shaking that's going on. A horse is basically barrel shaped and you either rest on smooth fur or leather. Sliding of the side is very easy, with stirrups it's almost impossible to happen.

Your lance digging into the ground is the least of your problems. Sideway motion is the real issue.

Yes, that was the received wisdom before experimental archaeology showed that the military saddle allowed the rider to stay very firmly in place without the need for stirrups.

Animastryfe
2013-04-02, 08:35 PM
How were lances used by heavy cavalry after the initial hit? I imagine that the lance would impale the enemy, who would almost immediately fall down and drag the lance down with him. Would the lance wielder then drop the lance, making the lance a one-hit weapon?

AgentPaper
2013-04-02, 08:40 PM
How were lances used by heavy cavalry after the initial hit? I imagine that the lance would impale the enemy, who would almost immediately fall down and drag the lance down with him. Would the lance wielder then drop the lance, making the lance a one-hit weapon?

From wikipedia with [citation needed], but: "As a secondary weapon, lancers of the period also bore swords, maces or something else suited to close quarter battle, since the lance was often a one-use-per-engagement weapon; assuming the lance survived the initial impact intact, it was (depending the lance) far too long, heavy and slow to be effectively used against opponents in a melee."

Beleriphon
2013-04-02, 08:44 PM
How were lances used by heavy cavalry after the initial hit? I imagine that the lance would impale the enemy, who would almost immediately fall down and drag the lance down with him. Would the lance wielder then drop the lance, making the lance a one-hit weapon?

Not any more than a spear is to infantry. Sure a knight might lose a lance occasionally but generally a lance was a heavy spear designed to pull loose from a foe that was just impaled. The giant lances from tournies weren't combat lances, actual combat lances were made of much more flexible wood and weren't designed to break on impact.

It also depends on what kind of attack were talking about. A heavy cavalry charge that moves right through an infantry line then the weapon is probably kept for more than one attack. If the knights end up in a melee there's a reason they usually had things like maces, hammers and picks.

On the topic of stirrups, I'd imagine they're more useful for leaning to one side or the other by shifting weight to that leg. I figure this would be very useful for melees rather than charges.

AgentPaper
2013-04-02, 08:51 PM
Hm, speaking of tournies, did knights ever charge at each other during combat and try to hit each other with their lances?

Animastryfe
2013-04-02, 08:53 PM
Not any more than a spear is to infantry. Sure a knight might lose a lance occasionally but generally a lance was a heavy spear designed to pull loose from a foe that was just impaled. The giant lances from tournies weren't combat lances, actual combat lances were made of much more flexible wood and weren't designed to break on impact.

It also depends on what kind of attack were talking about. A heavy cavalry charge that moves right through an infantry line then the weapon is probably kept for more than one attack. If the knights end up in a melee there's a reason they usually had things like maces, hammers and picks.

On the topic of stirrups, I'd imagine they're more useful for leaning to one side or the other by shifting weight to that leg. I figure this would be very useful for melees rather than charges.
How would the knight pull the lance free? I assume that the lance would be imbedded quite deeply in the enemy's flesh, and with the body dragging on the ground I cannot picture the mechanics of pulling the lance out.

Beleriphon
2013-04-02, 09:28 PM
How would the knight pull the lance free? I assume that the lance would be imbedded quite deeply in the enemy's flesh, and with the body dragging on the ground I cannot picture the mechanics of pulling the lance out.

You hold on and it rips free by virtue of momentum. All you need is a solid grip.

AgentPaper
2013-04-02, 09:44 PM
You hold on and it rips free by virtue of momentum. All you need is a solid grip.

Is there physical evidence (ie: someone charging a human analogue) for this? Because that sounds completely absurd and more likely to have your arm come off than to rip through a foot of human flesh, bones, and quite possibly plate armor. I mean, perhaps if the lance was short enough, you could let the lance spin around to point behind you, and then simply pull the lance out, but with a longer lance, it's going to hit the ground or the other knight five feet to your right before it can turn all the way around. And even then, it seems like a pretty reliable way to break your wrist and/or dislocate your shoulder even if you do manage to keep the lance.

It seems far more reasonable that a knight would abandon the lance, then either fight in melee, or pull back and pick up a fresh lance from their squire (that's their job, isn't it?). I'm sure they'd be happy to get their lance back after the battle if possible, since they can't be cheap, but it's not worth risking your life over.

Animastryfe
2013-04-02, 10:21 PM
Is there physical evidence (ie: someone charging a human analogue) for this? Because that sounds completely absurd and more likely to have your arm come off than to rip through a foot of human flesh, bones, and quite possibly plate armor. I mean, perhaps if the lance was short enough, you could let the lance spin around to point behind you, and then simply pull the lance out, but with a longer lance, it's going to hit the ground or the other knight five feet to your right before it can turn all the way around. And even then, it seems like a pretty reliable way to break your wrist and/or dislocate your shoulder even if you do manage to keep the lance.


This is what I was thinking. As the impaled enemy hits the ground, the angle of the lance from the horizontal would be much less than negative pi/4, as the length of the lance would usually be longer than the heigh of the horse plus the distance between the horse's back and the arm holding the lance, correct?

fusilier
2013-04-02, 10:45 PM
Might I politely suggest you bypass the filter in the mod-approved style? (I.e., "**** the hammer", using black color tags.)

So one of the theories about why that part of the gun is called a "****" goes back to the action of a sear style matchlock, where the motion of the "jaws" that held the match cord was reminiscent of chicken pecking at the ground.

It works! Thank you!

AgentPaper
2013-04-02, 10:49 PM
This is what I was thinking. As the impaled enemy hits the ground, the angle of the lance from the horizontal would be much less than negative pi/4, as the length of the lance would usually be longer than the heigh of the horse plus the distance between the horse's back and the arm holding the lance, correct?

Assuming the horse is 5 feet tall at the back, and you're holding the the lance 3 feet above that (which is probably generous), then you'd have 8 feet between your shoulder (the pivot point) and the ground. Assuming again that the peasant you're spearing is 4 feet to your side when you hit him, that gives you about 9 feet of clearance. This means that if you're using a lance 9 feet long (which apparently is on the short end), then you can just barely avoid hitting the ground, allowing the lance to spin around and be pulled out.

So, you could concievably retain your lance, IF it's no more than 9 feet, and IF you're on a very big horse, and IF you don't have anyone withing 4-5 feet of you to risk smacking/tripping, and IF your lance doesn't become embedded well enough that you end up dragging a dead body around, and IF you don't break your wrist due to whiplash, and IF you can manage to not have your shoulder dislocate from pulling the lance out, and IF you're heading in a perfectly straight line without running into anyone for a while after hitting the guy, and IF you're not worried about being attacked by someone while you're freeing your lance and bringing it back around in front of you.

There's a lot of ifs in there, and the benefits of keeping a lance after charging seem minimal at best. If you're running down some fool peasant in the middle of nowhere, sure, maybe you might try to do it, but in a pitched battle where you're probably charging into a huge mass of enemy soldiers? How often are you planning to charge in a single battle, anyways?

fusilier
2013-04-02, 10:54 PM
Lances:

I believe General Kearny was stabbed multiple times by the same lancer during the Battle of San Pasquale. On the other hand, I remember a story of someone hunting buffalo with a lance who used the wrong grip, and upon impaling the creature was lifted off his horse and trampled to death. I think the grip was described as underhand, but implied that it was tucked under the arm, the usual grip (when hunting buffalo) being an overhand one, or one where the lance was held away from the body.

Matthew
2013-04-02, 10:55 PM
How were lances used by heavy cavalry after the initial hit? I imagine that the lance would impale the enemy, who would almost immediately fall down and drag the lance down with him. Would the lance wielder then drop the lance, making the lance a one-hit weapon?

Nobody really knows. Precisely what happened during a cavalry charge is one of those things that we cannot accurately reconstruct. There are many theories, but the main thing to bear in mind is that cavalry did not usually fair well against undisturbed decent quality infantry, meaning that direct charges were ineffective and uncommon. Probably, in cases where cavalry charge cavalry or infantry the lance was dropped after impact, but that is not necessarily the case. For example, Polybius tells us that Roman and Greek cavalry had spikes on the butt of their spears to enable them to continue fighting with them when the head broke off, but then they were not what we think of as "heavy cavalry".

Traab
2013-04-02, 11:12 PM
Hmm, is there a wood strong enough to stab someone with at a charge, yet fragile enough to easily snap after its embedded in some poor schmucks abdomen? I ask because that would make the back spike more useful than as a, "In case your lance snaps and you manage to hold onto it you can try to stab someone else after." Or was that spike more to give a soldier extra options to attack with since the main spear head can only face in one direction, and might not be as easy to swing around as it would be to just stab with the other point instead.

Matthew
2013-04-02, 11:17 PM
Ha! No idea. I imagine the end being damage from the strike or hacked off by the enemy, but who knows? Here is the translated text in question:



The cavalry is divided also into ten parts or troops. In each of these, three captains first are chosen; who afterwards appoint three other officers to conduct the rear. The first of the captains commands the whole troop. The other two hold the rank and office of decurions; and all of them are called by that name. In the absence of the first captain, the next in order takes the entire command. The manner in which these troops are armed is at this time the same as that of the Greeks. But anciently it was very different. For, first, they wore no armor upon their bodies; but were covered, in the time of action, with only an undergarment. In this method, they were able indeed to descend from their horses, or leap up again upon them, with greater quickness and facility; but, as they were almost naked, they were too much exposed to danger in all those engagements. The spears also that were in use among them in former times were, in a double respect, very unfit for service. First, as they were of a slender make, and always trembled in the hand, it not only was extremely difficult to direct them with exactness towards the destined mark; but very frequently, even before their points had reached the enemy, the greatest part of them were shaken into pieces by the bare motion of the horses. Add to this, that these spears, not being armed with iron at the lowest end, were formed to strike only with the point, and, when they were broken by this stroke, were afterwards incapable of any farther use.

Their buckler was made of the hide of an ox, and in form was not unlike to those globular dishes which are used in sacrifices. But this was also of too infirm a texture for defense; and, as it was at first not very capable of service, it afterwards became wholly useless, when the substance of it had been softened and relaxed by rain. The Romans, therefore, having observed these defects, soon changed their weapons for the armor of the Greeks. For the Grecian spear, which is firm and stable, not only serves to make the first stroke with the point in just direction and with sure effect; but, with the help of the iron at the opposite end, may, when turned, be employed against the enemy with equal steadiness and force. In the same manner also the Grecian shields, being strong in texture, and capable of being held in a fixed position, are alike serviceable both for attack and for defense. These advantages were soon perceived, and the arms adopted by the cavalry. For the Romans, above all other people, are excellent in admitting foreign customs that are preferable to their own.

Yora
2013-04-03, 04:19 AM
Yes, that was the received wisdom before experimental archaeology showed that the military saddle allowed the rider to stay very firmly in place without the need for stirrups.
Having seen those things, they look like an alternative solution to the same problem.

Spiryt
2013-04-03, 04:32 AM
There's a lot of ifs in there, and the benefits of keeping a lance after charging seem minimal at best. If you're running down some fool peasant in the middle of nowhere, sure, maybe you might try to do it, but in a pitched battle where you're probably charging into a huge mass of enemy soldiers? How often are you planning to charge in a single battle, anyways?

Many, many times?

French charged at least 15 times at Crecy, AFAIR for example.

Obviously, that battle was huge failure from their part, but in more sensibly fought battle, getting back to your lines to regroup, breath a bit, and collect a new lance/ammunition was very important for tactics.

Making 'breaks' during the combat was very common practice, at least if both sides wanted to regroup obviously.

As far as pulling out the lance goes - well, one can stop, or slow down a lot ?(would probably happen naturally during the clash anyway) grab the lance tightly, with both hands, and pull, obviously.

Galloglaich
2013-04-03, 09:59 AM
Yeah I think it's kind of in between. Clearly they could stab people, pull the lance out some kind of way, and stab again. It's also clear that they frequently broke their lances, it was common for the attendant of a knight (valetti) to carry three extra lances at the 'ready' and as mentioned up thread it was normal to wheel back to the friendly lines and re-arm after charges. But could also hold on to them even after heavy impacts.

“By this time the vanguard of the Frankish horsemen had reached me, so I retired before them, turning back my lance in their direction and my eyes toward them lest some one of their horse should prove to quick for me and pierce me with his lance. In front of me were some of our companions, and we were surrounded by gardens with walls as high as a sitting man. My mare hit wit it’s breast one of our companions, so I turned it’s head to the left and applied the spurs to it’s sides, whereupon it leaped over the wall. I so regulated my position until I stood on a level with the Franks. The wall only separated us. One of their horsemen hastened to me, displaying his colors in a green and yellow silk tunic, under which I thought was no coat of mail. I therefore let him alone until he passed me. Then I applied my spurs to my mare, which leapt over the wall, and I smote him with the lance. He bent sideways so much that his head reached the stirrup, his shield and lance fell off his hand, and his helmet off his head. By that time we had reached our infantry. He then resumed his position, erect in the saddle. Having had linked mail under his tunic, my lance did not wound him. His companions caught up to him, all returned together, and the footman recovered his shield, lance, and helmet.”

-An Arab-Syrian Gentleman and Warrior in the period of the Crusades. Usamah Ibn-Minqidh, 12th Century AD

Seems like they could hold on to the lance even after a pretty heavy strike.

Also they seem to have done two types of lane charges against infantry, one where they ride up, accelerate a little usually into a canter, and make thrusts into the enemy as they wheel aside. This is what was done against intact infantry formations which still had good cohesion. The second was the all out charge directly into the enemy formation which was usually only done when it showed signs of breaking, was being attacked on the flank or hadn't gotten organized yet, though it also did also occur that good heavy cavalry could break infantry in a single charge, often at the cost of many horses.



G

Galloglaich
2013-04-03, 10:01 AM
Hm, speaking of tournies, did knights ever charge at each other during combat and try to hit each other with their lances?

Yes routinely.

G

Rhynn
2013-04-03, 11:33 AM
Holy crap, mail will stand up to a lance charge? That's impressive if accurate.

AgentPaper
2013-04-03, 12:09 PM
Holy crap, mail will stand up to a lance charge? That's impressive if accurate.

The description is a bit weird, but it sounds less like a direct charge, and more like him coming up behind and to the side of him and poking with his lance. Since both horses are moving in about the same direction, there's less momentum in play, and add the bit where the other guy basically falls off his horse (intentionally?) to avoid the full impact, and there's probably not all that much force to the thrust. He says himself that he only tried it because he thought the guy had no armor on at all, so a weak thrust could still make a pretty decent wound, which in those days could be just as deadly as a full impalement in many cases.

Spiryt
2013-04-03, 12:32 PM
Dunno if anyone has some different translation, because translation can make huge difference in case of completely different language, and also ancient one.


But translated like that, it leaves little doubt that impact was powerful.

Usama spurred his horse, and the smite was so violent that the dude was almost unhorsed, his helmet felt off his head, and his weapons out of his hands.

Spears, also wielded from horseback, were most common threats on battlefields, so it's not in any way surprising that quality mail was made to protect against them.

This article (http://www.myarmoury.com/feature_mail.html) has more mentions of uses of lances against mail. With citations, so all of those can probably be found in the Internet with a bit of trying.

AgentPaper
2013-04-03, 12:49 PM
But translated like that, it leaves little doubt that impact was powerful.

Sorry, didn't mean to imply that it was a weak blow, except in comparison with a full charge with both horses hurtling directly at each other at full speed.

Galloglaich
2013-04-03, 01:58 PM
I think it was a formidable strike.

Usamah does describe other incidents (and I thnk in most cases, head to head lance charges which might double the amount of KE in the impact) where sometimes the armor protects, sometimes the lance goes right through. Usamah himself wore a jazeraint consisting of two mail coats, one frankish, one Arabic, plus textile components (and I think rabbit fur? If I remmber correctly)

Some of these guys probably had stiffeners and metal plates under their armor by Usamahs time.


The thirteenth century continuation of William of Tyre insists that contemporary armour for knights was too restrictive to allow them to fight effectively on foot, by comparison with their more lightly equipped forebears (Richard the Lionheart and his men in the specific instance). Such a development suggests increasingly specialised "lancers", which would accord with your speculation.

Yes because I think that 13th century transitional / kind of piecemeal armor was probably a bit more like the 6th Century Sassanid stuff, than either the earlier mail, which is fairly easy to get around in, or the later fully articulated plate harness which had become very well put together so as not to impede movement much.

I think in the 13th - mid 14th Century you had more distinct panoplies which were worn by cavalry vs. heavy infantry, not just the shoes but leg protection period, such as you saw at Wisby. Which also matches I think the 6th Century or the 1st.

G

warty goblin
2013-04-03, 03:09 PM
I think it was a formidable strike.

Usamah does describe other incidents (and I thnk in most cases, head to head lance charges which might double the amount of KE in the impact) where sometimes the armor protects, sometimes the lance goes right through. Usamah himself wore a jazeraint consisting of two mail coats, one frankish, one Arabic, plus textile components (and I think rabbit fur? If I remmber correctly)

G
A head to head impact would do a lot more than double the kinetic energy. Compared to striking a stationary target from the back of a moving horse, it would quadruble the KE. Compared hitting a target riding along the same vector as lancer's own mount, it would be an even greater increase.

Spiryt
2013-04-03, 03:29 PM
A head to head impact would do a lot more than double the kinetic energy. Compared to striking a stationary target from the back of a moving horse, it would quadruble the KE. Compared hitting a target riding along the same vector as lancer's own mount, it would be an even greater increase.

Overall, all around energies would without a doubt be great, but all of this wouldn't do all that much to energies that actually can get used/get transfered by

shoulder, elbow, wrist, lance

system.

So in general math is tricky here.

AgentPaper
2013-04-03, 03:44 PM
Here's a question: How much power did an arrow have? I've heard conflicting stories about them, for example one that said that plate armor made you basically invulnerable, and others saying they were almost as powerful as bullets (but of course much harder to aim), including a story of an arrow hitting a Knight's leg, and going through the outer plate, his leg, the inner plate, and then a foot into his horse before stopping.

Spiryt
2013-04-03, 04:23 PM
Well, for start, 'power' is somehow unclear term here.

Power, physically, is a rate at which energy is transfered, used etc. and in arrows case it depends on much more things than arrow and it's motion itself.

As far as sheer kinetic energy goes, arrows generally don't even come close to having bullets energy.

Smallest, slowest modern pistol bullets have KE of at least ~ 300 J, generally, right after leaving the barrel.

Arrows from really heavy poundage traditional reflexives composites will have ~ 170 J at most.

Then one have to remember that overall energy is not quite as important as the way it's being 'used', which depends on a lot of things.

As far as penetrating ferrous plate supported by human body and some cloth, quite a lot of tests had been done. Most of them were somehow lacking, some barely scientific though.

Galloglaich
2013-04-03, 07:08 PM
Right, so to add to what Spiryt said, because of it's shape and composition (pointy vs round and usually hardened iron or steel vs. lead) arrows require less energy to go through things than bullets do.

Dr. Alan Williams, who is kind of the definitive expert on armor today due to his exhaustive tests on antique armor, determined that an arrow needed (if I remember correctly) about 1/4 to 1/10 as many joules as a bullet does to penetrate armor.

G

kardar233
2013-04-03, 11:54 PM
I was thinking about making a laser into a personal melee weapon the other day. Now, this would be for a 40K game but I prefer to ground things in real science whenever possible, so that's why I'm here.

I'm wondering if anyone knows of a preferably non-rigid material with a high refractive index that decreases when pressure is applied. I'm imagining a weapon made up of an infrared laser that fires into what's effectively an optical fibre. This fibre's index of refraction would change when pressure is applied to it, sending infrared laser light streaming out towards whatever made contact.

Is this at all feasible?

Brother Oni
2013-04-04, 02:31 AM
As far as penetrating ferrous plate supported by human body and some cloth, quite a lot of tests had been done. Most of them were somehow lacking, some barely scientific though.

To support Spiryt's point, here's quite a good test of arrows against various armour types: English Longbow Testing circa 1400 (http://www.currentmiddleages.org/artsci/docs/Champ_Bane_Archery-Testing.pdf).

The short of it is that penetration depends on the arrow and the poundage of the bow and even quite thin plate (1.2 mm) is decent at stopping arrows from a 75 lb selfbow at 10 yds (or a 110 lb longbow at 250 yds).

One issue with the test is that the angle of penetration is optimal so shooting at an angle may result in less penetration than expected due to the physics of armour penetration.

Another test I remember tried 1.2 mm, 2 mm and 3 mm plate, with only partial penetration against the 2 mm and no penetration on the 3 mm at optimal angle (90 degrees). This would imply that 2mm plate would be effective against an arrow striking at an angle less than ~67 degrees so indirect fire could possibly cause less casualties against well armoured targets than expected.


Dr. Alan Williams, who is kind of the definitive expert on armor today due to his exhaustive tests on antique armor, determined that an arrow needed (if I remember correctly) about 1/4 to 1/10 as many joules as a bullet does to penetrate armor.

Do you have a link to anything he's done? I'm interested in what sort of tests were performed.

AgentPaper
2013-04-04, 02:42 AM
Any suggestions for good books to read to learn more about how medieval armies were organized? IE: What kinds of soldiers there were, where they came from, what equipment they used, where THAT came from, where they lived, how they were paid, what tactics were employed, et cetera.

TuggyNE
2013-04-04, 02:59 AM
Probably kind of a stupid question, but how were visors on helmets fastened to make sure they wouldn't get popped open in the middle of combat?

fusilier
2013-04-04, 04:16 AM
Right, so to add to what Spiryt said, because of it's shape and composition (pointy vs round and usually hardened iron or steel vs. lead) arrows require less energy to go through things than bullets do.

Dr. Alan Williams, who is kind of the definitive expert on armor today due to his exhaustive tests on antique armor, determined that an arrow needed (if I remember correctly) about 1/4 to 1/10 as many joules as a bullet does to penetrate armor.

G

For reasons that are not entirely clear to me, when talking about projectiles the standard framework is "energy" -- and for even more obscure reasons this framework carries over to armor penetration. As hinted at above, it's force that penetrates armor, and more specifically force applied to an area (pressure). Which means that the more important factors to consider are momentum, shape, and maybe material (soft lead carries more mass but deforms more easily than a harder material). So assuming the same amount of momentum a narrow, stiff projectile, like an arrow, will develop more pressure than a round musket ball.

Rhynn
2013-04-04, 06:36 AM
Probably kind of a stupid question, but how were visors on helmets fastened to make sure they wouldn't get popped open in the middle of combat?

It probably varied - I've seen some visored bascinets with a leather strap & buckle on the "chin" to keep the visor down. Some might just have had a fairly stiff "joint" (or whatever you call the connection). But AFAIK pulling off your opponent's helmet or lifting their visor was a technique in harnischfechten (which involves a lot of grappling, and often the use of a dagger to actually kill your opponent). It may be that there was no optimal solution, because being able to open your own visor (without taking off your gauntlets or whatever you've got on) was a big advantage - it gets hot and hard to breathe under those, I understand.

Edit:
Here (http://www.getdressedforbattle.co.uk/acatalog/klappvisor-bacinet.jpg) (jpg) is an example of a fastening system - a chain fastening. Where it's authentic is anyone's guess.

Here (http://media-cache-ec4.pinterest.com/192x/c9/71/c6/c971c67ebde0f032b46f7f2c0228c072.jpg) (jpg) is another - a strap all the way around the back of the helmet.

Both seem really fiddly to be opening by yourself with gauntlets or even heavy gloves on, but I've never tried, so what do I know?

Galloglaich
2013-04-04, 09:18 AM
Any suggestions for good books to read to learn more about how medieval armies were organized? IE: What kinds of soldiers there were, where they came from, what equipment they used, where THAT came from, where they lived, how they were paid, what tactics were employed, et cetera.

The Osprey military books are probably your best bet. They are kind of the cliff notes of military organization going back to the Bronze Age or before. You just have to pick your period and place. Norman knights, pre-Edo Samurai, Greek Hoplites, whatever. Each one is about $15 and most are available on Amazon. Some you can also find online as PDF's.

They are usually pretty well illustrated and reasonably accurate - more than you'll usually find on History channel or BBC anyway, and they cover all the things you asked about above, kit, organization, tactics and so on, usually with pretty good maps and diagrams as well as the drawings of typical soldiers. Easy to read and they usually have decent bibliographies in the back for further (more serious) reading. I think these are the most accessible entry into this kind of thing for most people.

There are also of course forums like this one, All Empires, Myarmoury, RomanArmyTalk, HEMA Alliance, Schola Gladiatoria and others where you can find answers to specific questions.

G

Galloglaich
2013-04-04, 10:21 AM
Thought some folks might find this interesting:

"In 1601, an artisan in Nordlingen named Hans Schwarz was arrested by the local council because of a weapons violation. The problem was not that Schwarz had kept or used an illicit weapon; rather, his crime was that he did not own a sword. Schwarz was only one of a number of local householders arrested in that year for failing to keep sufficient stores of arms and armor in their homes. The men were given 14 days to 'honourably arm themselves'. In other towns, presentation of proper arms was a requirement for marriage."

Source:
http://www.amazon.com/Martial-Ethic-Early-Modern-Germany/dp/0230576567/ref=sr_1_2?ie=UTF8&qid=1364946213&sr=8-2&keywords=ann+tlusty

Page 1

G

fusilier
2013-04-04, 04:24 PM
The Osprey military books are probably your best bet. They are kind of the cliff notes of military organization going back to the Bronze Age or before. You just have to pick your period and place. Norman knights, pre-Edo Samurai, Greek Hoplites, whatever. Each one is about $15 and most are available on Amazon. Some you can also find online as PDF's.

They are usually pretty well illustrated and reasonably accurate - more than you'll usually find on History channel or BBC anyway, and they cover all the things you asked about above, kit, organization, tactics and so on, usually with pretty good maps and diagrams as well as the drawings of typical soldiers. Easy to read and they usually have decent bibliographies in the back for further (more serious) reading. I think these are the most accessible entry into this kind of thing for most people.

There are also of course forums like this one, All Empires, Myarmoury, RomanArmyTalk, HEMA Alliance, Schola Gladiatoria and others where you can find answers to specific questions.

G

I wanted to second this. The Opsrey books provide good introductions and are affordable and easily accessible. There's a pair of more recent ones (Elite series I think), that specifically covers Medieval tactics. However, they do make mistakes, and accuracy varies from book to book. Nevertheless, they provide a good overview, and sometimes work as a springboard for more detailed study (if desired). Another place to look at would be wargaming websites, theminiaturespage.com, for example. They will be concerned with precisely the issues you describe. In that vein, there are some books by Ian Heath; they are a little bit dated now (no worse than a lot of the Osprey books), but they give a good general treatment of the various armies of the Middle Ages, and also supply illustrations and describe soldiers in more detail (useful for miniature wargamers).

Armies of the Middle Ages (volumes one and two).

TuggyNE
2013-04-04, 08:49 PM
It probably varied - I've seen some visored bascinets with a leather strap & buckle on the "chin" to keep the visor down. Some might just have had a fairly stiff "joint" (or whatever you call the connection). But AFAIK pulling off your opponent's helmet or lifting their visor was a technique in harnischfechten (which involves a lot of grappling, and often the use of a dagger to actually kill your opponent). It may be that there was no optimal solution, because being able to open your own visor (without taking off your gauntlets or whatever you've got on) was a big advantage - it gets hot and hard to breathe under those, I understand.

Edit:
Here (http://www.getdressedforbattle.co.uk/acatalog/klappvisor-bacinet.jpg) (jpg) is an example of a fastening system - a chain fastening. Where it's authentic is anyone's guess.

Here (http://media-cache-ec4.pinterest.com/192x/c9/71/c6/c971c67ebde0f032b46f7f2c0228c072.jpg) (jpg) is another - a strap all the way around the back of the helmet.

Both seem really fiddly to be opening by yourself with gauntlets or even heavy gloves on, but I've never tried, so what do I know?

Interesting, thanks. :smallsmile:

I have seen a few helmets that didn't have separate visors, presumably for safety reasons.

Rhynn
2013-04-04, 10:39 PM
Interesting, thanks. :smallsmile:

I have seen a few helmets that didn't have separate visors, presumably for safety reasons.

There's many types of helmets like that, yes. Some Valsgarde and Vendel helmets have a full face visor that doesn't move, like the Sutton Hoo helmet. Heaumes (great helms) have no separate visor. There's visored sallets with no hinges on the visor.

Matthew
2013-04-05, 03:47 AM
Yes because I think that 13th century transitional / kind of piecemeal armor was probably a bit more like the 6th Century Sassanid stuff, than either the earlier mail, which is fairly easy to get around in, or the later fully articulated plate harness which had become very well put together so as not to impede movement much.

I think in the 13th - mid 14th Century you had more distinct panoplies which were worn by cavalry vs. heavy infantry, not just the shoes but leg protection period, such as you saw at Wisby. Which also matches I think the 6th Century or the 1st.

Very likely. It seems like a very dangerous specialisation on and off the battlefield, but I guess it might also accord with the reduction in the number of "knights" relative to mounted sergeants and the like, which would make it more understandable.

Galloglaich
2013-04-05, 04:31 PM
Very likely. It seems like a very dangerous specialisation on and off the battlefield, but I guess it might also accord with the reduction in the number of "knights" relative to mounted sergeants and the like, which would make it more understandable.

If you are talking about the leg armor, it's because (I think) it's just hard to run or walk for a long time with leg armor on, especially from the knees on down.


There's many types of helmets like that, yes. Some Valsgarde and Vendel helmets have a full face visor that doesn't move, like the Sutton Hoo helmet. Heaumes (great helms) have no separate visor. There's visored sallets with no hinges on the visor.

Face plates or face masks go way back, Roman cavalry wore them,

http://www.livius.org/a/germany/kalkriese/kalkriese_mask.JPG

The Persians

http://media-cache-ec3.pinterest.com/550x/6d/d6/b4/6dd6b4e8fe561e1734b8729b23a891da.jpg

And the Russians, the Byzantines, the Vikings and so on. They remained common into the 11th Century, perhaps coincidentally around the time when the heavy cavalry rose to prominence.

With heavy cavalry, you see the full faced heaume and the great helm, which was really only worn for cavalry charges.

http://upload.wikimedia.org/wikipedia/commons/thumb/6/60/Topfhelm_DHM.jpg/180px-Topfhelm_DHM.jpg

This is an actual 'helm'

Underneath these, they also wore lighter iron caps called bascinets, cervielle

http://upload.wikimedia.org/wikipedia/commons/thumb/2/23/Maciejowski_Bible_Navin.gif/220px-Maciejowski_Bible_Navin.gif

http://en.wikipedia.org/wiki/Cervelliere

These are 'helm-ettes'

Once the fighting started, they typically fought with an open-faced helmet. Like these bascinets

http://i038.radikal.ru/0811/39/26aca635cd23.jpg

But having two helmets wasn't that efficient, and an open faced helmet was kind of dangerous in a fight (though it helped a lot with allowing you to see and breathe). So later they started adding hinged nasals to them, like this

http://upload.wikimedia.org/wikipedia/commons/thumb/6/62/05-207.png/150px-05-207.png

and then visors, which made the old 'helm' obsolete.

http://upload.wikimedia.org/wikipedia/commons/thumb/8/8f/05-183.png/200px-05-183.png

Another route was the sallet, which started as just a cap used by the infantry

http://www.christies.com/lotfinderimages/D53565/a_very_rare_sallet_almost_certainly_italian_third_ quarter_of_the_15th_d5356506h.jpg

then they added a vision slit in it so you could pull it down lower over your face when, say, somebody was shooting arrows at you

http://images.metmuseum.org/CRDImages/aa/web-highlight/DP22326.jpg


http://www.christies.com/lotfinderimages/D55098/a_rare_german_sallet_circa_1470_d5509813h.jpg

Then the neck protection (gorget) began to be extended to include a seperate chin protector (bevor) which helped cover the lower part of your face

http://www.artic.edu/aic/collections/citi/images/standard/WebLarge/WebImg_000102/141633_868781.jpg

and then they added a visor to the helmet and combined the whole thing together into very complete protection.

http://upload.wikimedia.org/wikipedia/commons/thumb/c/c7/Sallet_helmet%2C_Southern_Germany%2C_1480-1490_-_Higgins_Armory_Museum_-_DSC05461.JPG/450px-Sallet_helmet%2C_Southern_Germany%2C_1480-1490_-_Higgins_Armory_Museum_-_DSC05461.JPG




I think all this correlated with the gradual increase of the threat from high-velocity missiles on the European battlefield.

G

Matthew
2013-04-07, 08:10 AM
If you are talking about the leg armor, it's because (I think) it's just hard to run or walk for a long time with leg armor on, especially from the knees on down.

In this case, the text does not limit itself to the encumbrance of leg armour, but suggests that contemporary armour is so restrictive that a dismounted knight is virtually helpless. Here is the passage in question:



He and his knights and sergeants went lightly armed. At that time hardly anyone had a bacinet, shoulder pieces, pointed coif, grieves or a helm with a visor unless he were a king, count or a great lord. Because they were lightly armed, if by some chance a sergeant lost his horse he could manage on foot, may God be merciful! But now their armour is so tight and heavy that if a knight falls from his horse he can do nothing to help himself.

Of course, the translation may be lacking in accuracy, but that is what I was thinking of particularly.

Yora
2013-04-07, 08:59 AM
I love the look of sallets, but they appear to allow almost no vision at all.

Rhynn
2013-04-07, 01:08 PM
I love the look of sallets, but they appear to allow almost no vision at all.

How do they allow less vision than visored bascinets or heaumes? They've got the same general size eye-slit as any other closed helm, IMO.

Compare...

Sallet (http://fc08.deviantart.net/fs70/f/2010/100/9/2/Sallet_armet_by_Ugo_Serrano.jpg), visored, with bevor.

Armet (http://upload.wikimedia.org/wikipedia/commons/f/fa/Heaume-Morges-4.jpg)
Heaume / Great Helm (http://4.bp.blogspot.com/-nuJHFGTaIVQ/ThYv0jJZOwI/AAAAAAAAAmU/5cW_1yJoklg/s400/crusaders_great_helm_400.jpg) ("sugar loaf great helm")
Hounskull bascinet (http://images.metmuseum.org/CRDImages/aa/web-highlight/sfeah04-3-235CRs4.jpg)
Klappvisor bascinet (http://www.thorkil.pl/platnerstwo/helmy/pozne/szwajcaria.jpg)
Close helmet (http://www.artic.edu/aic/collections/citi/images/standard/WebLarge/WebImg_000033/65574_277335.jpg)

Obviously, they all restrict vision to some degree (and it varies more by individual helmet than by type, between these closed/full helmed types), but that's because sticking a dagger in someone's eye (or getting a lucky arrow into it) is a great way to kill them.

The visored bascinet looks to me like one of the easiest models to open up when you need to see.


In this case, the text does not limit itself to the encumbrance of leg armour, but suggests that contemporary armour is so restrictive that a dismounted knight is virtually helpless. Here is the passage in question:


Of course, the translation may be lacking in accuracy, but that is what I was thinking of particularly.

What armor could that be referring to? I was under the impression that even full gothic harness isn't so heavy that you can't, for instance, lift yourself up from the ground.

I can't find the video I wanted (older gentleman flinging himself off a horse in armor and getting up just fine), but here's one (http://www.youtube.com/watch?v=xm11yAXeegg)...

Spiryt
2013-04-07, 01:17 PM
Judging by period, it could be some full mail with some clunky early coat of plates, perhaps with a lot of padding and leg armor, that would make moving on your own rather tiring and sluggish.

I would say that this text is obviously a hyperbole, like with many medieval accounts, but it was certainly possibility for very encumbering armor in 13th century.

As far as I recall, full armor that famous 'Kings Mirror" suggest sounds pretty damn bulky, multiple layers of linen both under and over the mail...

Rhynn
2013-04-07, 03:07 PM
Ah, that actually lines up with something that I noticed in the rules of HârnMaster... that damn quilt armor is heavy, and having it on your legs is a bit crazy.

Given how thick and heavy real textile armor apparently/probably was, I can definitely imagine that quilted trews or leggings of some sort would be very hard to move in, especially with some mail chausses on top.

Straybow
2013-04-07, 08:24 PM
My understanding was that you don't want a sword razor-sharp because it will dull too quickly.

Then I read that the edge-to-edge combat you see in movies wasn't actually how they did things, because smacking sword-blades like that would actually chip or break the weapon in short order. Yes, it does chip the edge. The weapon is a tool, and it suffers some abuse in service. Good quality steel stands much abuse and the edge can be worked and ground many times. You'll replace it for aesthetics before it will wear out.

In the "dark ages" steel was expensive to make and a relatively rare skill. The Vikings preferred to block attacks with the shield rather than the sword to preserve the blade. The near-parallel edge blade is not very well balanced. With the superior balance of a tapered blade a sword can cross and disengage quicker for more effective defense. Steel making had advanced both in average quality and in availability, so that high quality swords were less expensive and readily available.

As for Clements' position, he's speaking specifically about longsword techniques. It simply doesn't apply to one-handed swordsmanship. There are some situations where a parry on the flat is possible, but reaching out to block with the true edge before the blow reaches full strength is a more sure defense than twisting the wrist to deflect with the flat close to the body.

Hawkfrost000
2013-04-07, 09:08 PM
I love the look of sallets, but they appear to allow almost no vision at all.

They give you more vision than you would think.

My sallet gives me about 80% my normal peripheral vision. The bit that throws you off is that you cant see down to where your feet are. Its not that much of a problem in combat, but it makes climbing stairs a pain.

DM

Rhynn
2013-04-07, 11:35 PM
They give you more vision than you would think.

My sallet gives me about 80% my normal peripheral vision. The bit that throws you off is that you cant see down to where your feet are. Its not that much of a problem in combat, but it makes climbing stairs a pain.

DM

This seems to be common with helmets that protect the face - they limit ownward vision a lot more than peripheral. Including more somewhat "open" helmts like hoplite helms (and barbutas, I expect). I expect this could get to be a problem in a battlefield with wounded lying all over the place, which may be one reason to open your visor - the charge is done, the fighting is breaking up, you need to see down so some stubborn bugger doesn't grab your legs, trip you, and crawl over you with a dagger, or just drive a sword into you from below. (Hoplite helmets were made to be pushed up on your head to allow better vision; incidentally, the design of sallets makes me think the visorless closed sallets may have been used similarly - tilt it back and up on your head to expose your face, with the long back protecting your neck from arrows from behind.)